Está en la página 1de 250

UNED · UCR · MEP · ITCR · UNA

Problemas de preparación para las olimpiadas


costarricenses de matemática

Recopilación y edición:
Comisión de Olimpiadas
Costarricenses de Matemáticas
–OLCOMA–

EUNED
EDITORIAL UNIVERSIDAD ESTATAL A DISTANCIA
2
Presentación

El proyecto olimpiadas es una actividad académica que intenta realizar un aporte en la


creación de medios que permitan hacer atractivo el estudio de las matemáticas y despertar en
los jóvenes no sólo el espı́ritu de investigación sino una sana competitividad. Su propósito
es permitir a los estudiantes de secundaria desarrollar el interés y habilidades por esta
disciplina, ası́ como proporcionar a los docentes un recurso más que fortalezca su quehacer
académico.
El banco de problemas que aparece en este folleto constituye una recopilación de ejer-
cicios de olimpiadas anteriores, realizadas tanto en nuestro paı́s como en otros paı́ses donde
se llevan a cabo este tipo de certámenes ası́ como una colección de nuevos problemas
aportados por profesionales en el campo.
El propósito de estos ejercicios es que sirvan de base para la preparación de los es-
tudiantes que participan en las distintas eliminatorias, de los diferentes niveles, de las
Olimpiadas Costarricenses de Matemática. Esperamos que tanto los estudiantes como los
profesores que los preparan obtengan el máximo provecho del material.
En la primera parte de este trabajo se propone una lista de ejercicios; son más de cien
ejercicios para cada uno de los tres niveles A, B y C en que se divide la competencia.
Estos ejercicios están propuestos en las modalidades de selección y desarrollo. Debemos
recordar que las eliminatorias de la Olimpiada Nacional contemplan estas dos modalidades:
la primera está constituida solamente por ejercicios de selección, la segunda tiene parte de
selección y parte de desarrollo, la tercera (final) está conformada únicamente por problemas
de desarrollo. Estos ejercicios que aquı́ se proponen tienen diferentes grados de dificultad
con el propósito de que sirvan de entrenamiento para las tres eliminatorias. Debemos agregar
que la clasificación de los ejercicios por niveles está hecha tomando en cuenta la temática de
la que tratan y la dificultad de los mismos; sin embargo, esto no excluye que los estudiantes
puedan resolver, también, muchos de los ejercicios que corresponden a niveles diferentes del
suyo.
En la segunda parte del material se proporcionan las seis pruebas de la Olimpiada
Nacional del año 2000; las tres eliminatorias de los dos ciclos en que se dividió esta com-
petencia. Esta es otra buena cantidad adicional de problemas que puede ayudar en el
entrenamiento.
Se incluye un esquema de solución a los ejercicios planteados, tanto de los problemas
propuestos en la primera parte como de los problemas de las seis pruebas de la Olimpiada
del año 2000. Cabe destacar que las soluciones oficiales que se presentan no son los únicos

3
4

caminos para llegar a la respuesta. Conociendo el interés de los estudiantes participantes


en estos eventos académicos competitivos y sabiendo la capacidad de estos jóvenes, estamos
seguros de que los mismos podrán obtener otras soluciones interesantes a los problemas
planteados. Como notará el lector, los problemas que aquı́ aparecen no son ejercicios rutina-
rios a los que se les aplica directamente los conocimientos que se adquieren en secundaria,
más bien requieren de una buena dosis de ingenio para ser resueltos. Como en todos los
aspectos del aprendizaje de las matemáticas, el esfuerzo individual y el enfrentamiento so-
litario con los problemas son importantes, pero también es muy importante la discusión con
los compañeros y los profesores.
Hemos agregado también un pequeño resumen que contiene algunos conceptos y resul-
tados que pueden ser útiles en la resolución de este tipo de ejercicios. Algunos de estos
conceptos no son necesariamente parte de los programas de matemáticas de la enseñanza
media, pero son básicos y de relativamente fácil comprensión.
Al final hemos incluido los temas que abarca cada nivel. Esta temática está relacionada
con los programas oficiales del Ministerio de Educación, pero no los siguen en forma estricta,
dada la manera en que está dividida la competencia. Como pueden ver, algunos temas son
prácticamente los mismos en los tres niveles, en estos casos, desde luego, lo que hace la
diferencia es la profundidad de los temas y el grado de dificultad de los ejercicios.
La Comisión de Olimpiadas Costarricenses de Matemáticas espera que este material sea
de utilidad en la preparación previa a la competición y agradece su participación en las
olimpiadas costarricenses de matemática.
Contenido

Presentación 3

Ejercicios propuestos 7
NIVEL A 7
NIVEL B 27
NIVEL C 45

Eliminatorias del año 2000 67


Primera eliminatoria 67
III ciclo (7◦ , 8◦ , 9◦ ) 67
IV ciclo (10◦ , 11◦ , 12◦ ) 73
Segunda eliminatoria 80
III ciclo (7◦ , 8◦ , 9◦ ) 80
IV ciclo (10◦ , 11◦ , 12◦ ) 83
Tercera eliminatoria (final) 87
III ciclo (7◦ , 8◦ , 9◦ ) 87
IV ciclo (10◦ , 11◦ , 12◦ ) 88

Solución de los ejercicios propuestos 91


NIVEL A 91
NIVEL B 116
NIVEL C 155

Solución de las eliminatorias del año 2000 205


Primera eliminatoria 205
III ciclo 205
IV ciclo 210
Segunda eliminatoria 217
III ciclo 217
IV ciclo 221
Tercera eliminatoria (final) 226
III ciclo 226
IV ciclo 229
6

Conceptos y resultados útiles 235

Simbologı́a 245

Temario de la Olimpiada 247


Ejercicios propuestos

NIVEL A
SELECCION
1. Cuando el número 111222333444555666777888999 se divide entre 111, se obtiene un
número cuya cantidad de dı́gitos es
a) 8 c) 17
b) 9 d) 25
2. En la figura, los cuadrados ABC D y EF GH son congruentes, AB = 10 cm y G es el
centro del cuadrado ABC D.

D C

A B
H F

Entonces, el área de la región del plano cubierta por la figura es


a) 175 cm2 c) 100 cm2
b) 150 cm2 d) 75 cm2
3. Juan tiene con Carlos el mismo parentesco que Antonio tiene con el hijo de Carlos.
Carlos tiene con Antonio el mismo parentesco que Antonio tiene con Juan. De las
siguientes proposiciones, puede ser cierto que:
a) Carlos es el nieto de Juan
b) Carlos es el padre de Juan
c) Juan es el nieto de Carlos
d) Juan es le padre de Carlos

7
8

4. Sean b, p, q, x, y, z números naturales con b 6= 1 y tales que p = bx , q = by ,


b4 = (py qx )z , entonces el producto xyz es igual a
a) 4 c) 2
b) 3 d) 1

5. La siguiente figura consta de siete cuadrados iguales, el área de esta figura es 112
cm2 .

Entonces el perı́metro de la figura es


a) 64 cm c) 48 cm
b) 88 cm d) 28 cm

1 −1
6. Si 3x 4x − 1 = 11 , entonces
3 3
a) 0 ≤ x ≤ 4 c) 2 <x≤2
3 3
b) 4 <x≤ 2 d) x > 2

7. El porcentaje de aciertos de un jugador de baloncesto en un partido fue exactamente


65 %, entonces el menor número de lanzamientos que el jugador debió realizar en ese
partido fue
a) 20 c) 65
b) 30 d) 100

8. Un Hotel tiene planta baja (piso 0) y 1000 pisos. De la planta baja parten 5 ascensores.
El ascensor
A para en todos los pisos
B para en los pisos 0, 5, 10, 15, ...
C para en los pisos 0, 7, 14, 21, ...
D para en los pisos 0, 17, 34, 51, ...
E para en los pisos 0, 23, 46, 69, ...
El número de pisos en los que paran exactamente 4 ascensores es igual a
a) 5 c) 4
b) 8 d) 2
9

9. El número de parejas de números naturales p y q que verifican que el mı́nimo común


múltiplo de p y q es igual a 80, es
a) 9 c) 27
b) 3 d) 54

10. El señor Villanueva, el señor Becerra y el señor Espinoza viven en la casa de huéspedes
de Nana Pancha. Uno de ellos es panadero, el otro es taxista y el tercero es bombero.
Sabiendo que:

El señor Villanueva y el señor Becerra juegan ajedrez todas las noches


El señor Becerra y el señor Espinoza van juntos a los juegos de beisbol
El taxista colecciona monedas, el bombero soldaditos de plomo y el panadero,
sellos postales
El taxista nunca ha ido a un juego de beisbol
El señor Espinoza nunca ha oı́do hablar de sellos certificados

Entonces, en lo que trabaja el señor Becerra


a) es de panadero
b) es de taxista
c) es de bombero
d) no es posible determinarlo

11. En la figura, ABC D es un rectángulo con AB = 20, C B = 16, M y N son puntos


medios de DC y BC respectivamente.

M
D C

A B

El área del cuadrilátero ANC M es igual a


a) 80 c) 160
b) 120 d) 240

12. Si la expresión ab − ab se puede descomponer en dos factores cuya suma es igual a


b + a , entonces esos factores son
a b

a) 1+ b
a ya−b
ab c) a+b
a y a−b
b

b) b−a
a y a−b
b d) a+b
b y a−b
a
10

13. De acuerdo con la información que se proporciona en la figura adjunta el segmento


de mayor longitud es

B
55◦
A 70◦
60◦ D

60◦
C

a) BD c) AC
b) BC d) C D

14. Sea m = 313 + 513 , el menor número primo p que divide a m es


a) p = 3 c) p = 5
b) p = 2 d) p = 13

15. Los señores Trujillo, Lara, Bolı́var y Sucre son de los lugares llamados Trujillo, Lara,
Bolı́var y Sucre; pero en ningún caso el apellido coincide con el nombre del lugar de
nacimiento. El nacido en Trujillo no tiene el mismo apellido que el nombre del lugar
de nacimiento del señor Bolı́var. El nacido en Lara no es el señor Sucre, ni tiene como
apellido el nombre del lugar de nacimiento del señor Lara. Entonces podemos afirmar
que el que nació en Sucre es el señor
a) Lara c) Bolı́var
b) Trujillo d) Sucre

16. Se tienen cuatro números enteros consecutivos ordenados de menor a mayor. Si al


cubo del cuarto número se le resta el cubo del tercero, a este resultado se le suma el
cubo del segundo y a esto se le resta el cubo del primero, entonces, sobre el número
obtenido podemos asegurar que
a) es divisible por 2 pero no por 3
b) es divisible por 6
c) es divisible por 3 pero no por 2
d) no es divisible ni por 2 ni por 3

17. Los ángulos de un triángulo están en razón 2 : 3 : 4, entonces la suma de las medidas
de los dos ángulos menores es igual a
a) 80◦ c) 100◦
b) 90◦ d) 120◦

18. Varios bailarines están bailando en cı́rculo acomodados uno frente a otro, es decir,
diametralmente opuestos como se muestra en la figura.
11

1
20

53

Si los numeramos consecutivamente comenzando con el 1 y si además sabemos que


el baiları́n número 20 está exactamente enfrente del número 53, entonces el número
de bailarines que hay es
a) 58 c) 64
b) 100 d) 66

19. Un cubo de madera de 4 cm de lado está pintado en toda su superficie exterior de


color azul. Realizando cortes horizontales y verticales se obtienen 64 cubitos de 1 cm
de lado. El número total de cubitos que no tienen ninguna de sus caras pintadas de
azul es igual a
a) 24 c) 16
b) 8 d) 32

20. Sean a y b números naturales con a > b. Entonces del entero positivo m definido
por m = (2a + 1)2 − (2b + 1)2 se puede afirmar que es divisible por
a) 3 c) 16
b) 8 d) 9

21. Sean a, b, m números reales con a + m 6= 0. La condición que debe cumplirse para
que se verifique la igualdad

a2 − m2 + 2ab + b2 a+b+m
2 2
=
a − m + ab + mb a+m

es que
a) a + b − m 6= 0 c) a − b + m 6= 0
b) a − b − m = 0 d) a + b + m = 0

22. En la figura, ABC D es un cuadrado de lado 6 cm y C E = DE = 5 cm.


12

C D

B A

Entonces la medida de AE es

a) 10
√ cm c) √115 cm
b) 101 cm d) 109 cm
23. Si a, b, c son números tales que a + b = 2c, entonces el valor de la expresión
" b c−a # c1
2a−b 2b−c
c−b
2c+b

es
a) 1 c) 4
b) 2 d) 8
24. Oscar, Carlos, Antonio y Rubén son candidatos a ocupar un cargo. Los requisitos son:
astucia, alta inteligencia y firmeza. Solamente uno de ellos reúne todos los requisitos
para ser elegido. Se sabe que:

i) Cada uno de ellos posee, al menos, uno de los requisitos.


ii) Solamente tres de ellos son astutos, solamente dos son altamente inteligentes
y solamente uno es firme.
iii) Oscar y Carlos tienen igual grado de inteligencia. Carlos y Antonio son igual-
mente astutos. Antonio y Rubén no son, ambos, astutos.

El elegido es
a) Oscar c) Carlos
b) Antonio d) Rubén
25. La cantidad de números de cuatro cifras que podemos formar con la condición de
que la suma de los cuadrados de las cifras de los extremos sea 13 y la suma de los
cuadrados de las cifras del medio sea 85 es
13

a) 2 c) 4
b) 3 d) 8

26. La siguiente distribución de números enteros positivos está formada de tal manera
que: 1 es el padre de 2, 3 y 4; 2 es el padre de 5, 6 y 7; 3 es el padre de 8, 9 y 10; 4
es el padre de 11, 12 y 13; 5 es el padre de 14, 15 y 16; y ası́ sucesivamente

1
2 3 4
5 6 7 8 9 10 11 12 13
14 ...

Entonces, el padre de 2000 es


a) 2001 c) 665
b) 667 d) 1996

27. En un restaurante, un hombre encarga que reserven una mesa para cenar varias
personas que son un padre, una madre, un tı́o, una tı́a, un hijo, una hija y dos primos.
El número mı́nimo de personas que pueden asistir al restaurante, para que se satisfaga
el enunciado, es
a) 8 c) 4
b) 6 d) 3

28. Considere un triángulo ABC tal que AB = BC = 10 cm, AC = 16 cm. Los segmentos
BD y AE son medianas del triángulo ABC , trazadas sobre los lados AC y BC res-
pectivamente. El punto F es punto de intersección de los segmentos AE y BD tal que
BF = 23 BD. El área del triángulo ADF es
a) 24 cm2 c) 16 cm2
b) 8 cm2 d) 48 cm2

29. En la figura, ABC D es un cuadrado, 4ABE es isósceles; C F = F B.

E
D C

A B

La medida del ángulo EF B es igual a


a) 45◦ c) 60◦
b) 135◦ d) 125◦
14

30. En la siguiente tabla:


1440 720 240 60 x
el valor de x es
a) 12 c) 6
b) 40 d) 8
31. Las longitudes de los lados de un triángulo son b + 1, 7 − b y 4b − 2. El número de
valores de b para los cuales el triángulo es isósceles es
a) 0 c) 2
b) 1 d) 3

32. Se escribe 1998 = (n − 1)nn (10n + c). Si n y c son enteros positivos entonces c es
igual a
a) 7 c) 9
b) 3 d) 37
33. En la figura, el ángulo C OB mide 120◦ y el ángulo C OD mide la mitad del ángulo
BOA.

B
C

D A
O

Entonces, la medida de ∠BOA es


a) 90◦ c) 20◦
b) 60◦ d) 40◦
34. Si N es un número de cinco dı́gitos de la forma 3a42b (con a y b dı́gitos), entonces
el número de maneras en que se puede elegir a y b para que N sea divisible por 6
es igual a
a) 2 c) 17
b) 19 d) 6

35. En un cuadrado ABC D de lado 1, E es el punto medio de la diagonal BD y F es el


punto medio de ED. Entonces el área de 4C F D es
3 1
a) 8 c) 2
1 1
b) 12 d) 8
15

36. La siguiente figura se puede doblar de manera que se forme un cubo.

D E

B C

x A

Al formar dicho cubo la letra que corresponde a la cara opuesta de la cara marcada
con x es
a) B c) D
b) C d) E

37. En la figura, ABEF es un rectángulo y 4C DE es isósceles. AB = 100 cm; AF es el


triple de AB, BC es el doble de AB y el perı́metro de la figura es 9, 41 m.

B E
C

A F

Entonces C D es igual a
a) 1, 41 m c) 0, 41 m
b) 2, 41 m d) 3, 41 m

38. La suma de todos los dı́gitos del número 1099 − 99 es


a) 873 c) 879
b) 874 d) 899

39. En la siguiente figura, los lados grandes son iguales entre sı́ y los lados pequeños
son iguales entre sı́. Además, los lados pequeños miden la mitad de los lados grandes.
Todos los ángulos son rectos y el área de la figura es 200.
16

Entonces el perı́metro de la figura es


a) 20 c) 60
b) 40 d) 80

40. En la siguiente figura, el número de caminos que hay para ir de la casilla 1 a la


casilla 7 si solo se permite moverse de una casilla a otra adyacente marcada con un
número mayor es

1 3 5 7

2 4 6

a) 8 c) 12
b) 11 d) 13

41. En la figura, el cuadrilátero ABC D es un rectángulo; M y N son los puntos medios de


los lados AD y BC respectivamente. P y Q son los respectivos puntos de intersección
de AC con BM y con ND.

M
A D

P
Q

B C
N

Si AD mide 5 cm y AB mide 3 cm, entonces el área del cuadrilátero MPQD es


a) 9 cm2 c) 7, 5 cm2
b) 5, 5 cm2 d) 3, 75 cm2

42. Una señora tiene tres hijas. El producto de las edades de la madre y sus hijas es
16555. Entonces la diferencia entre la edad de la hija mayor y la edad de la hija menor
es igual a
a) 6 c) 5
b) 12 d) 32

43. El lado AC de un triángulo ABC se divide en 8 partes iguales. Siete segmentos de


recta paralelos a BC se dibujan desde los puntos de división.
17

C A

Si BC = 10, entonces la suma de las longitudes de los 7 segmentos es igual a


a) 35 c) 40
b) 45 d) 32

44. Con vértices en los puntos de la figura,

el número de cuadriláteros que se puede dibujar es


a) 4 c) 24
b) 16 d) 36

45. Se tiene una fila de 5 sillas numeradas del 1 al 5. Suponga que usted está sentado en
alguna de esas sillas. Un movimiento consta de pasarse a una de las sillas que estén
a su lado. Si usted está en la silla 1, solo puede pasarse a la silla 2, análogamente,
si usted está en la silla 5, solo puede pasarse a la 4, pero si está en cualquier otra
silla usted tiene dos posibilidades para pasarse. Suponga que usted está sentado
inicialmente en la silla 1, luego realiza 19 movimientos, después elimina la silla 1 y
la silla 5 y, finalmente, hace 99 movimientos más. ¿En qué silla terminará sentado?
a) 2 c) 4
b) 3 d) no se puede determinar

46. El número de triángulos que hay en la siguiente figura es

a) 22 c) 18
b) 20 d) 14
18

47. Se tiene que completar la siguiente cuadrı́cula con los números del 1 al 5, de tal forma
que cada número aparezca únicamente una vez en cada columna y en cada renglón.

3 4 1 5
2
2 3
1 5
4

Entonces, el número que va en el centro de la cuadrı́cula es


a) 1 c) 4
b) 2 d) 5
48. En una clase hay 25 alumnos. Entre ellos, 17 alumnos son ciclistas, 13 nadadores y 8
futbolistas. Ningún alumno hace tres deportes. Los ciclistas, nadadores y fultbolistas
sacaron 90 en matemáticas. Seis alumnos de la clase sacaron 60 en matemáticas.
Entonces, el número de nadadores que juegan fútbol es
a) 2 c) 6
b) 4 d) 10

49. En la figura, ABC D es un paralelogramo. P es un punto de la diagonal AC . Trazamos


por P paralelas a los lados del paralelogramo. Estas paralelas intersecan a los lados
del paralelogramo en los puntos indicados en la figura.

R
A B

P
N Q
D C
M

Sabiendo que (ABC D) = 40 cm2 , entonces (RQMN) es igual a


a) 40 cm2 c) 20 cm2
b) 10 cm2 d) 18 cm2
50. Para subir de una primera a una segunda planta hay que subir en total 10 gradas.
Una persona puede subir las gradas de una en una o de dos en dos o combinando
estas posibilidades sin regla alguna (nunca de tres en tres o más gradas a la vez). El
número de maneras que dicha persona puede subir de la primera a la segunda planta,
pisando obligatoriamente la sexta grada, es
a) 19 c) 18
b) 13 d) 65
19

b
51. Sea y = a + , x 6= 0, con a y b constantes. Si y = 1 cuando x = −1 y y = 5
x
cuando x = −5, entonces a + b es igual a
a) −1 c) 11
b) 0 d) 10

52. En la figura, ABC D es un paralelogramo de área 10 cm2 , AB = 3 cm y BC = 5


cm. E, F y G son puntos en los lados AB, BC y AD respectivamente, tales que
AE = BF = AG = 2 cm. Sea H el punto de intersección del segmento C D con la
lı́nea paralela a EF que pasa por G.

2 F 3
B C
E H

A D
2 G 3

Entonces el área del cuadrilátero EF HG es


a) 4 cm2 c) 5 cm2
b) 4, 5 cm2 d) 5, 5 cm2

DESARROLLO
1. En un colegio hay clubes de jardinerı́a, teatro y pintura. De 93 alumnos de sétimo año
de ese colegio, 7 están al menos en el club de jardinerı́a, 27 están al menos en el club
de teatro, 40 al menos en el club de pintura, 7 al menos en jardinerı́a y teatro a la
vez, 4 al menos en jardinerı́a y pintura, 19 al menos en teatro y pintura y, finalmente,
hay 4 que están en los tres clubes. Determine cuántos estudiantes están solamente
en el club de jardinerı́a, cuántos solamente en el de teatro, cúantos solamente en el
de pintura y cuántos no están en ningún club.

2. Un hombre distribuyó dinero entre sus hijos de la siguiente manera: primero le dio
al menor C// 1000 más 101 de lo que restaba, luego dio al segundo C // 2000 más 101
// 3000 más 101 de lo que hasta ese momento
del restante, después al tercero le dio C
quedaba y ası́ sucesivamente hasta llegar al último hijo. Al final cada hijo recibió la
misma cantidad de dinero. ¿Cuántos hijos tiene el hombre y cuánto dinero repartió?

3. Pruebe no existe ningún número entero positivo n tal que N = n2 + 1 sea divisible
por 3.

4. En el alfabeto de la tribu UAU hay solo dos letras: U y A. Además esta lengua posee
las siguiente propiedades: si de una palabra quitamos UA (es decir la letras U y A que
están juntas), entonces el significado de la palabra no varı́a; el sentido de la palabra
20

tampoco varı́a al adicionar en cualquier lugar de la palabra la combinación de letras


AU ó UUAA. ¿Usando solamente estas operaciones se puede afirmar que las palabras
UAA y AUU tienen el mismo significado?

5. Pruebe que si n es un entero positivo par, entonces el número N = n3 + 20n es


divisible por 48.

6. Dados los números reales positivos x, y, z, x 0 , y0 , z 0 tales que


x y z
0
= 0 = 0,
x y z

demostrar que
p √ p √
(x + y + z) (x 0 + y0 + z 0 ) = xx 0 + yy0 + zz 0 .

7. En la siguiente suma, cada letra representa un dı́gito distinto; letras iguales repre-
sentan el mismo valor:

A M L P T +
A O M A
L D
M E T
C O M I I T

Sabiendo, además, que se verifican las igualdades: M = 23 A; T = 23 M; M + I = A;


E < P, determine qué palabras determinan los números 053683979; 6942694319.

8. Sean ab y dc fracciones irreducibles positivas cuya suma es un número entero. Probar


que b = d.

9. Un grillo salta en lı́nea recta, la primera vez que saltó, su salto fue de 1 cm en alguna
dirección (hacia adelante o hacia atrás), la segunda vez saltó 2 cm en cualquier
dirección (hacia adelante o hacia atrás), la tercera vez saltó 3 cm y ası́ sucesivamente.
Pruebe que el grillo no puede volver a su posición inicial en 1985 saltos realizados
de la manera indicada.

10. Utilizando exclusivamente los dı́gitos 2 y a se forma el siguiente número de 90 cifras:

2a22a222a2222a . . . 22 . . . 2a.

Si este número es múltiplo de 9, ¿cuáles son los posibles valores de a?

11. Cada integrante de un grupo de 10 niños es amigo de exactamente 7 niños del grupo
(la amistad es mutua). Pruebe que no es posible dividir al grupo en tres equipos de
tal manera que en cada uno de los tres equipos no haya un par de amigos.
21

12. En un tablero de 10×10, una ficha se desplaza de acuerdo con la jugada del “camello”
la cual consiste en que la ficha se desplaza inicialmente a una casilla vecina (dos
casillas son vecinas si comparten un lado) y luego se desplaza tres casillas más en
sentido perpendicular. ¿Se puede, aplicando varias veces la jugada del camello, llevar
una ficha de alguna casilla inicial a una casilla vecina?

13. Desde un punto D sobre la hipotenusa de un triángulo rectángulo ABC , se trazan


perpendiculares DE y DF a los lados C A y AB respectivamente. Determine el punto
D para el cual EF tiene longitud mı́nima.

14. En una reunión de 10 personas hay exactamente 19 pares de personas que se conocı́an
entre sı́. Pruebe que existe una persona que conocı́a como máximo a 3 personas.

15. Se tiene un tablero de 25 × 25 casillas. En cada casilla está escrito alguno de los
números 1, 2, 3, . . ., 25. Además, en dos casillas cualesquiera, simétricas con respecto
a la diagonal principal, están escritos números iguales; en ninguna fila hay escritos
dos números iguales; en ninguna columna hay escritos dos números iguales. Probar
que todos los números en la diagonal principal son diferentes.

16. Sean a, b, c, d, e números naturales consecutivos tales que a + b + c + d + e es un


cubo perfecto y b + c + d es un cuadrado perfecto. Calcular el mı́nimo valor posible
de c.

17. En cada escalón de una escalera de 10 peldaños hay una rana. Cada una de ellas
puede, de un salto, colocarse en otro escalón, pero cuando lo hace, al mismo tiempo,
otra rana saltará la misma cantidad de escalones en sentido opuesto: una sube y otra
baja. ¿Conseguirán las ranas colocarse todas juntas en el mismo escalón?

18. El entero positivo N tiene 1994 cifras. De éstas, 14 son iguales a 0 y los números de
veces que aparecen las demás cifras:

1, 2, 3, 4, 5, 6, 7, 8, 9

están en la razón
1:2:3:4:5:6:7:8:9
respectivamente. Demostrar que N no es un cuadrado perfecto.

19. En un libro de 2108 páginas se tuvieron que reescribir todos los números de las
páginas. ¿Cuántos ochos se reescribieron?

20. Hallar el menor número natural terminado en 88, divisible por 88 y con la suma de
sus cifras igual a 88.

21. Las piezas de un rompecabezas rectangular son 9 cuadrados de lados 1, 4, 7, 8, 9, 10,


14, 15 y 18. ¿Cómo deben ubicarse las nueve piezas para armar el rompecabezas?
22

22. Encuentre (si es que existe) el mayor entero positivo n menor que 2300 tal que
n = p6 q, donde p y q son números primos y q > 10.

23. En la siguiente figura, para llegar del punto A al punto B, solamente se pueden
recorrer caminos en la dirección que indican las flechas. ¿Cuántos de estos caminos
distintos se pueden recorrer?

A 2 3 4 5 6 7 8 9 B

24. Sean a y b números reales tales que

2a2 + 2b2 = 5ab.


a+b
Determine los posibles valores para .
a−b
25. Se dan en el plano unos cuantos puntos rojos y unos cuantos puntos azules. Algunos
de los puntos están unidos por segmentos. Un punto se llama “punto especial” si más
de la mitad de los puntos unidos a él tienen color diferente al suyo. En cada paso se
elige un punto especial y se le cambia el color. Pruebe que al cabo de varios pasos
no queda ningún punto especial.

26. Un terreno triangular limita con tres terrenos cuadrados, cada uno de los cuales tiene
un lado común con el triángulo. Las superficies de los terrenos cuadrados son 505, 233
y 52 hectáreas. Determine la superficie del terreno triangular.

27. Cierta familia tiene hijos e hijas; cada hija tiene el mismo número de hermanas que
de hermanos y cada hijo tiene el doble de hermanas que de hermanos, ¿cuántas hijas
y cuántos hijos hay en esta familia?

28. Tres diarios cubren la información de una carrera de solamente tres participantes:
X , Y , Z . Extraemos dos afirmaciones de cada uno de ellos, una es falsa y la otra
verdadera:

El ganador no fue X
Diario A:
El ganador no fue Y

Y llegó último
Diario B:
X llegó antes que Z

Y llegó antes que Z
Diario C:
Z llegó antes que X
¿Es posible asegurar en qué orden llegaron los participantes? Justifique.
23

29. En la figura, ABC D es un trapecio de bases AB = 10 y C D = 6. La altura h mide


4. Sea P el punto medio del lado AD y Q el punto medio de PB. Determinar el área
del triángulo PQC .

D C

Q
A B

30. Sea Sn = 1 − 2 + 3 − 4 + 5 + · · · + (−1)n−1 n, para n = 1, 2, 3, .... Calcule el valor de


S57 + S69 − S60 .

31. En el rectángulo ABC D de la figura; M, N, P y Q son los puntos medios de los lados.

M
A B

Q N

D C
P

Si el área del triángulo sombreado es 1 cm2 ; determine el área del rectángulo ABC D.

32. Determine el valor numérico de la diferencia b − a, con b > a, si las medidas son
dadas en la figura, en la que x es una constante positiva mayor que 2.

x−1 x+1

a b
A x C

33. Sobre una mesa se tienen 1999 fichas que son rojas de un lado y negras del otro
(no se especifica cuántas hay con el lado rojo hacia arriba o con el lado negro hacia
arriba). Dos personas juegan alternadamente. Cada persona en su turno hace una de
las siguientes dos cosas:
24

a) Retirar cualquier número de fichas, con la condición de que todas las fichas retiradas
tengan el mismo color hacia arriba.
b) Voltear cualquier número de fichas, con la condición de que todas las fichas vol-
teadas tengan el mismo color hacia arriba.
Gana el que retira la última ficha. ¿Cuál jugador puede asegurar que ganará, el primero
en jugar o el segundo?

34. Encuentre todos los enteros positivos a, b tales que

a + b−1
= 13 y a + b ≤ 80.
a−1 + b

35. En la siguiente figura, ABC D es un cuadrado y DEF un triángulo equilátero con AC


paralelo a EF . Si DG es la prolongación de DE, determine la medida del ángulo
DGC .

E F
D

A C
G

36. Con 40 fósforos se forma la siguiente figura

Esta figura tiene 16 cuadrados de 1 × 1, 9 cuadrados de 2 × 2, 4 cuadrados de 3 × 3


y un cuadrado de 4 × 4. Determinar el mı́nimo número de fósforos que hay que quitar
para que la figura resultante no tenga ningún cuadrado de ningún tamaño.

37. En un triángulo isósceles ABC con AB = AC se toman D, E y F puntos sobre los


lados BC , C A y AB, respectivamente, de manera que el triángulo DEF es equilátero.
25

Si a = m∠BDF , b = m∠EFA y c = m∠DEC , pruebe que


b+c
a= .
2
38. La tabla

se va a llenar con números, de la siguiente manera:


La primera fila se completa con los números del 1 al 10 en ese orden. La segunda
fila se completa con los números del 1 al 10 en cualquier orden. En cada casilla de
la tercera fila se escribe la suma de los dos números escritos en las dos casillas que
quedan arriba de ella. ¿Hay alguna forma de completar la segunda fila de modo que
las cifras de las unidades de los números de la tercera fila sean todas distintas?
39. Encontrar todos los números de dos cifras tales que al sumarles el número escrito con
las mismas cifras pero en orden inverso, dan un cuadrado perfecto.
40. En una cuadrı́cula de 8 × 8 se han escogido arbitrariamente 10 cuadritos y se han
marcado los centros de éstos. El lado de cada cuadrito mide 1. Demuestre que existen
al
√ menos dos puntos marcados que están separados una distancia menor o igual que
2, o que existe al menos un punto a una distancia 12 de una orilla de la cuadrı́cula.

41. En la siguiente figura AB = AD = 130 y BEDC es un cuadrado cuya área es igual
a la de 4AEB. Determinar el área del cuadrado BEDC .

E
A C

42. Pruebe que la suma de cuatro números enteros positivos consecutivos no puede ser
un cuadrado perfecto.
43. Sea un número natural de cuatro dı́gitos, P = abcd (a, b, c, d son dı́gitos, con a 6= 0),
si se invierten sus cifras se obtiene el número P 0 = dcba y además P + P 0 = 6435.
¿Cuántos números P satisfacen esta propiedad?
44. Determine tres números enteros positivos tales que la resta del tercero menos el
segundo es 2 y la resta del segundo menos el primero también es 2 y además la suma
de los cuadrados de los tres números es igual a un número de cuatro cifras iguales.
26

45. Sea P un número entre 1 y 16000 tal que es a la vez un cubo y el doble de un
cuadrado. Determine todos los valores de P.

46. Si a, b, c, d son dı́gitos tales que 0 ≤ a < b < c < d, ¿cuántos números de la forma
1a1b1c1d1 son múltiplos de 33?

47. Determine para qué enteros positivos j el número 22 + 25 + 2j es un cuadrado perfecto.

48. Sea ABC un triángulo tal que ]ABC es agudo y m]ABC = 2 · m]AC B; se traza la
altura correspondiente al vértice A que corta a BC en D. Sea E en la prolongación
del lado AB tal que BE = BD. La recta que pasa por E y D corta al lado AC en F .
DF
Determine el valor de .
AC

49. Un número de tres cifras es equilibrado si una de sus cifras es el promedio de las
otras dos, por ejemplo el número 258 es equilibrado pues 5 = 2+8
2 . ¿Cuántos números
equilibrados de tres cifras hay?

50. Pruebe que si n es un entero positivo entonces los números n(n + 1)(2n + 1) y
n(n + 1)(4n + 5)son divisibles por 6.

51. Decidir si es posible o no distribuir los 16 números del 1 al 16 en los triangulitos de


la figura, de modo que la diferencia entre los números colocados en los triangulitos
vecinos valga 1 ó 2 (triangulitos vecinos son los que tienen un lado en común).

52. Sea ABC un triángulo escaleno de área 1999. Sea A1 un punto del lado BC y sean

→ ← → ←→ ←→ ←→
B1 y C1 puntos sobre las rectas AC y AB respectivamente, tales que AA1 , BB1 y CC1
son paralelas. Determine el área del triángulo A1 B1 C1 .

53. Escribir el número 1999 como la suma de números enteros positivos, de tal manera
que el producto de dichos números sea el mayor posible.

54. Los números del 1 al 12 se colocan, sin repetir, en los cı́rculos del siguiente arreglo
triangular:
27

i) Muestre que no existe una forma de acomodarlos de modo que las sumas de los
números que están en cada uno de los lados del triángulo sea 27.
ii) Pruebe que sı́ existe un acomodo en que, en cada uno de los lados del triángulo,
la suma sea igual a 28.

NIVEL B
SELECCION
5 sen A
1. Sea A un ángulo agudo tal que tan A = 12 . El valor numérico de la expresión
1 + cos A
es
a) 2/3 c) 1/5
b) 5/13 d) 5

2. N es un número de tres dı́gitos tal que al restarle 8 el resultado es divisible entre 8,


si a N se le agrega 9 el resultado es divisible por 9 y si a N se le resta 7, el resultado
es divisible por 7. El residuo que se obtiene al dividir N entre 5 es
a) 4 c) 2
b) 3 d) 1

3. Si A y B son cifras, distintas de cero, entonces el número de cifras (no necesariamente


diferentes) de la suma
9876 + A32 + B1
es
a) 4 c) 6
b) 5 d) faltan datos

4. El mayor número de cuatro dı́gitos que aparece en la progresión aritmética

1, 4, 7, 10, 13, 16, . . .

es
a) 9998 c) 9997
b) 9999 d) 9996
28

5. En la figura, el paralelogramo ABC D está formado por cuatro triángulos equiláteros


de lado 1.

D C

A B

Entonces, la longitud de la diagonal AC es



a) 3√ c) 7
b) 5 d) 4

6. Un grupo de personas salió a almorzar a un restaurante. Se gastaron C k 7 200, el gasto


se repartió por partes iguales entre todas las personas del grupo, pero posteriormen-
te se decidió que tres personas no pagarı́an por lo que cada uno de los restantes
pagó C
k 400 más. El número de personas que habı́a en el grupo es
a) 6 c) 7
b) 8 d) 9

7. ¿Cuántos números hay entre 9992 y 10002 , sin incluir estos dos números?
a) 999 c) 1998
b) 1000 d) 1999
8. En la figura, las dos rectas l y m son paralelas, entonces x es igual a

E D
l
40 ◦

x C
100◦
m
A B

a) 120◦ c) 140◦
b) 130◦ d) 150◦
9. El entero positivo 403 puede ser escrito en la forma 403 = p + q, con p y q números
primos tales que p < q, de
a) una manera c) cinco maneras
b) dos maneras d) cuatro maneras

10. En el triángulo de la figura adjunta se tiene que AH⊥BC , BC = 6m, AH = 4 m.


29

M N

C B
R H Q

Entonces el cuadrilátero MNQR es un cuadrado de lado


a) 4, 16 m c) 2, 8 m
b) 2, 4 m d) 4 m

11. El número de enteros positivos comprendidos entre 74 y 47 , que son cuadrados perfectos
es igual a
a) 76 c) 80
b) 78 d) 82

12. Se tiene un trapecio ABC D donde BC es la base menor, tal que BC = 10 cm, C D = 19
cm y las medidas de los ángulos A, B, C , son respectivamente 30◦ , 150◦ , 120◦ . Entonces
AD es igual a
a) 48 cm c) 28 cm
b) 38 cm d) 24 cm

13. Se quiere alumbrar el perı́metro de un terreno de forma trapezoidal cuyos lados miden
140 m, 133 m, 210 m y 182 m. Se desea que en cada uno de los vértices del terreno
quede un poste. Además, que dos postes consecutivos guarden la misma distancia y
que dicha distancia sea la mayor posible. El número de postes que se necesitan para
alumbrar el terreno es
a) 95 c) 92
b) 94 d) 91

14. El número de maneras en que se puede descomponer el número 1999 como la suma
de un número impar positivo más el cuadrado de un número par positivo, es
a) 10 c) 26
b) 20 d) 22

15. Cierto vehı́culo viaja a 60 km/h si va cuesta arriba, a 90 km/h si va cuesta abajo y a
72 km/h en los demás casos. Si el vehı́culo tarda 5 horas para ir de A a B y 4 horas
para regresar de B a A, entonces la longitud del camino entre A y B es
30

a) 300 km c) 324 km
b) 360 km d) 348 km

16. En la sucesión 1, 3, 2, . . ., cada término, después de los dos primeros, es igual a la


diferencia del precedente y el precedente del precedente. La suma de los primeros
2001 términos de la sucesión es
a) 0 c) 4
b) 1 d) 6

17. El número de 4 cifras 8xy9 es un cuadrado perfecto. Entonces x + y es igual a


a) 10 c) 1
b) 9 d) 5

18. Sea n ∈ N, n > 1. Un número real equivalente a


r
n 12n (4n + 1)
8n · 3n + 6n
es
√ q
2
a) n
4 c) n
3
b) 2 d) 3

19. En la figura, los triángulos BF C y BAE son triángulos equiláteros.

E F

A C

La medida, en grados, del ángulo AOC , es


a) 60◦ c) 120◦
b) 135◦ d) 45◦

20. Se divide un rectángulo en rectángulos más pequeños, como se muestra en la figura.


31

1 2
3 4
A B

x 16

D
C

Las áreas de los rectángulos pequeños son las indicadas en la figura. El área x del
rectángulo ABC D es
a) x = 7 c) x = 6
b) x = 8 d) x = 9

21. Dado el sistema 


 x+y−1

 x−y+1 =a
,

 y−x−1
 =b
x+y+1
entonces x + y es igual a
2b − ab − 1 ba + a + 2
a) c)
ab + 1 1 − ab
1 − ab ba + 1
b) d)
ab + 1 1 − ab
22. En la figura, ABC D es un cuadrado cortado por dos segmentos perpendiculares entre
sı́: EF y GH.

E
A B
H

D C
F

Si EF = 10 cm, entonces HG mide


a) 15 cm c) 20 cm
b) 10 cm d) 25 cm
32

23. En una fiesta habı́a distintos tipos de cajas de galletas de tal modo que los asistentes
pudieron establecer las siguiente conclusiones:
i. De cada caja de galletas comieron exactamente 3 personas.
ii. Cada persona escogió galletas de exactamente 2 cajas distintas.
iii. Por cada par de cajas hubo exactamente una persona que comió de ambas.
El mı́nimo número de personas que comieron galletas es
a) 6 c) 8
b) 4 d) 12

24. Efectuando el producto 999 . . . 9 × 555 . . . 5 (donde el primer número tiene 95 nueves
y el segundo número tiene 95 cincos), se obtiene un número cuya suma de las cifras
es igual a
a) 846 c) 855
b) 945 d) 954

25. Fernando pensaba vender sus postales en 1000 colones. Después de vender 8 postales
perdió la cuarta parte de las que le quedaban y solo pudo obtener 850 colones en
la venta total. Si vendió todas las postales al mismo precio, entonces el número de
postales que tenı́a es
a) 85 c) 24
b) 50 d) 20

26. Un número de dos cifras es el doble del producto de sus cifras. La suma de las cifras
de dicho número es
a) 6 c) 8
b) 7 d) 9
 2
2
27. La suma de los dı́gitos en base 10 del número 101999n +2 + 1 , donde n es un entero
positivo es
a) 4 c) 1999n2
b) 4n d) 1999n2 + 2

28. En la figura, ABDC es un rectángulo.

A B

C D
33

El porcentaje del área del rectángulo que corresponde al área de la región sombreada
es
a) 20 c) 25
b) 30 d) faltan datos
29. Se comienza con el número 1. Una “operación” consiste en multiplicar el número por
3 y sumarle 5. La cifra de las unidades del resultado obtenido después de aplicar la
operación 1999 veces es
a) 1 c) 8
b) 2 d) 9
30. La cantidad de números enteros positivos n que satisfacen la desigualdad
2 n 11
< <
5 17 13
es
a) 6 c) 8
b) 10 d) 5
31. Se escriben los números 5, 7, 11, 13, 17, 19 y 23, uno en cada uno de los circulitos de
la siguiente figura, de manera que la suma de los tres números en cada lı́nea sea el
mismo número primo.

El número que queda en el circulito del centro de la figura es


a) 7 c) 13
b) 11 d) 17

32. En la siguiente figura, los segmentos AY y BX son perpendiculares a los segmentos


BC y AC respectivamente.

T X

B C
Y
34

Si m∠ABC = 50◦ y m∠BAC = 60◦ entonces la medida de ∠BT Y es igual a


a) 60◦ c) 80◦
b) 70◦ d) 50◦
1
 1

33. Si 1 + n 1− m = 1, entonces m es igual a
a) n − 1 c) 2n

b) n + 1 d) n2 + 1
34. Si los números a, b, c satisfacen las siguientes igualdades:
1 1 1
+ + = 1,
a b c
1 1 1 1
− + = ,
a b c 3
1 1 1
+ − = 0,
a b c
entonces a + 2b + 3c es igual a
a) 6 c) 18
b) 12 d) 24
35. Si A y B son números naturales tales que
A B 31
+ =
7 5 35
entonces A es igual a
a) 1 c) 3
b) 2 d) 4
36. En un triángulo equilátero X Y Z se dividen los lados en tres partes iguales. Llamemos
a las divisiones A, B, C , D, E, F como se muestra en la figura.

A F

B E

Y Z
C D

Si el área de 4X Y Z es igual a 18, entonces el área de la región sombreada es igual


a
a) 12 c) 9
b) 10 d) 8
35

37. La suma de cinco números enteros es 146. Si m es el mayor de los cinco múmeros,
entonces el menor valor que puede tener m es
a) 35 c) 27
b) 41 d) 30
38. En la siguiente figura se tiene que
AE 2
AD = 2DB y = .
EC 3

A C
E

Si el área de 4ADE es 8, entonces el área de 4ABC es


a) 20 c) 28
b) 24 d) 30

39. El entero positivo más pequeño n que satisface la ecuación n3 + 2n2 = b, donde b es
el cuadrado de un número impar, es
a) b − 1 c) 15
b) 7 d) b − 5
40. La cantidad de números a, menores que 1500 y tales que (5a + 1)(3a + 2) es divisible
por 15 es
a) 98 c) 100
b) 99 d) 101
41. El número entero positivo equivalente al número natural

n = 65743 · 5438 + 34257 · 4562


+65743 · 4562 + 34257 · 5438

es
a) 109 c) 34257 · 104
b) 65743 · 104 d) 1020

42. Se dan los números ar, ar 2 , ar 3 , . . ., ar 10 (donde a y r son números reales no nulos).
Si la suma de esos números es 18 y la suma de los recı́procos es 6, entonces el
producto de los números es igual a
36

a) 216 c) 81
b) 94 d) 243

43. Un rectángulo se divide en nueve subrectángulos, como muestra la figura. En el interior


de algunos de los rectángulos está escrito su perı́metro.

12 4 6

Entonces el perı́metro del rectángulo grande es igual a


a) 56 c) 72
b) 36 d) 28

n2 + n − 1
44. Si n es un entero positivo entonces la fracción es irreducible para
n2 + 2n
a) ocho valores de n
b) cuatro valores de n
c) diez valores de n
d) infinitos valores de n

45. En un rectángulo que mide 120 de largo y 100 de ancho se traza un triángulo en su
interior como en el dibujo.

B h

Si la base del triángulo es b = 125, entonces la altura h del triángulo mide


a) 96 c) 60
b) 90 d) 75
37

46. Se tiene un número x de tres cifras, x = abc, donde a, b, c son cifras diferentes entre
sı́ y diferentes de cero y un número y = bca. La resta x − y es un número de dos
cifras que es un cuadrado perfecto; entonces la cantidad de números x que cumplen
estas condiciones es igual a
a) 7 c) 11
b) 9 d) 13
47. En las siguientes expresiones, a, b, c representan números reales distintos de cero:
xy xz zy
= a, = b, = c.
x+y x+z y+z

Suponiendo que las expresiones están bien definidas, entonces x en términos de a, b,


c es igual a
2abc
a) abc
ab+ac+bc c) ab+ac−bc
2abc 2abc
b) ab+ac+bc d) ac+bc−ab

48. Considere la secuencia 1, 2, 2, 3, 3, 3, 4, 4, 4, 4, 5, 5, 5, 5, 5, ... cuyos términos son


enteros positivos consecutivos en orden creciente y el entero n aparece n veces. Al
dividir entre 5 el término número 1993 de esta secuencia, se obtiene como resto
a) 0 c) 3
b) 4 d) 2
49. En la siguiente figura se tiene que AB = BD = 2, m]DBC = 30◦

30◦
A C
B

Entonces AD es igual a
√ √ √ √
a) √6 + 2 c) √6+ 3
b) 2 3 d) 3 2
50. Sean a, b, c números reales positivos tales que
b < a < c < a + b y defı́nase
√ √ √ √ √ √
c− a a+b− a a− a−b
A= c−a , B= b , C= b , entonces podemos asegurar que
a) C < A < B c) A < B < C
b) B < A < C d) C < B < A
38

51. Si las medidas de las medianas en un triángulo rectángulo,


√ trazadas a partir de
los vértices de los ángulos agudos, miden 5 cm y 40 cm; entonces la medida en
centı́metros de la hipotenusa del triángulo rectángulo es
√ √
a) 2 13 c) 13
b) 20 d) 52
a
52. La suma de todas las fracciones de la forma tales que a y b son enteros positivos,
b
menores o iguales que 1000, con a ≤ b es
a) 250 750 c) 190 850
b) 320 840 d) 380 940

DESARROLLO
1. Si A es un número real, el sı́mbolo [A] se llama la parte entera de A y corresponde
al mayor número entero que es menor o igual que A. Determine todos los números
enteros x, tales que hx i hx i
+ = 15.
2 3
2. Determinar los valores reales del parámetro m de manera que el sistema

mx + 2y = a
7x + 5y = b

tenga una solución en números enteros (x, y) para cualesquiera sean los valores de
los enteros a y b.

3. Pruebe que si la suma k + m + n de tres números enteros positivos es divisible por


6 entonces k 3 + m3 + n3 también es divisible por 6.

4. Un polinomio P(x) tiene coeficientes enteros y para cierto número entero a se verifica
que
P(a) = P(a + 1) = P(a + 2) = 1,
¿existe algún entero k tal que P(k) = 8?

5. Sea T un triángulo cuyos lados son enteros consecutivos y cuya área es un entero.
Muestre que una de sus alturas divide al triángulo en dos triángulos rectángulos
pitagóricos, es decir, con lados enteros. Además muestre que esta altura divide a la
base en dos segmentos cuyas longitudes difieren en 4 (salvo en el caso del triángulo
3 − 4 − 5).

6. Un escolar gastó cierta suma de dinero para comprar un cuaderno, un lapicero y un


lápiz. Si el cuaderno hubiera costado 15 de lo que costó, si el lapicero hubiera costado
la mitad de lo que costó y si el lápiz hubiera costado 25 de lo que costó, entonces el
gasto total habrı́a sido de 80 colones. Si el cuaderno hubiera costado la mitad de lo
39

que costó, el lapicero la cuarta parte de lo que costó y el lápiz la tercera parte de lo
que costó, entonces el gasto habrı́a sido de 120 colones. Determine cuál fue el gasto
de la compra y determine si el cuaderno es más caro que el lapicero o al revés.

7. De la ciudad A parten simultáneamente una motocicleta, una bicicleta y un automóvil,


hacia una ciudad B. Cuando el automóvil llega a B, regresa a A y de regreso se
encuentra a la motocicleta a x kilómetros de B y luego se encuentra a la bicicleta a y
kilómetros de B. Cuando la motocicleta llega a B regresa y encuentra a la bicicleta a z
kilómetros de B. Determine, en términos de x, y, z, la distancia entre ambas ciudades,
suponiendo que cada vehı́culo viajaba a velocidad constante.

8. Determinar todas las parejas de números reales (x, y) que satisfacen



 2
 2x = y

 1 + x2

 2y2

 =x
1 + y2

9. En un paralelogramo ABC D se tiene que AB = 10, BC = 5 y el ángulo agudo entre


las diagonales mide 60◦ . Determine la altura sobre el lado AB.

10. Dado que

0 = (x + y)2 + (x + 3y)2
−10(x + 3y) − 4(x + y) + 29,

determine el valor de la expresión 2y2 + xy.

11. Sean x, y números reales tales que


 p  q 
x + x2 + 1 y + y2 + 1 = 1.

Calcule el valor numérico de x + y.

12. Tres recipientes A, B y C tienen inicialmente los siguientes contenidos: A, 2 litros de


aceite de soya; B, 3 litros de aceite de maı́z y C , 4 litros de aceite de girasol. Luego
se pasa 1 litro del recipiente A al recipiente B; después se mezcla bien y se pasa 1
litro del recipiente B al recipiente C. Este se mezcla bien y se pasa 1 litro de C al
recipiente B. Finalmente, bien mezclado el contenido de B se pasa 1 litro de este al
recipiente A. Después de completada la operación, ¿qué porcentaje de cada aceite hay
en el recipiente A?

13. En una reunión hay 31 hombres y 31 mujeres sentados alrededor de una mesa redonda.
Pruebe que los vecinos de al menos una persona son dos hombres.
40

14. Sea k un número entero mayor o igual que 1, tal que k(k+1)
2 es un cuadrado perfecto
N 2 , con N < 100. Determine todos los k que satisfagan las condiciones dadas.

15. En un triángulo ABC , se tiene que BD 0 y BE 0 trisecan al ángulo B, y C D 00 y C E 00


trisecan al ángulo C . Sea D la intersección de BD 0 con C D 00 y E la intersección de
BE 0 con C E 00 , con E más cercano que D al lado BC . Pruebe que los ángulos BDE y
EDC son iguales.

16. Se considera la ecuación

x 2 − (5m + a) x + 6m2 + 5m − 4 = 0,

donde a y m son parámetros (la incógnita es x). Determine qué valores debe tomar
a para que la ecuación tenga soluciones reales para cualquiera que sea el valor del
parámetro m.

17. Demostrar que para cualquier número entero positivo n es válida la desigualdad
1 1 1 1
+ + ··· + 2
< .
9 25 (2n + 1) 4

18. En el plano se traza un polı́gono con a + b vértices; a de los vértices se designan con
la letra A y los otros b vértices se designan con la letra B. Sobre cada uno de los
lados del polı́gono se hace lo siguiente: si los dos vértices del lado están denotados
con la letra A, sobre ese lado escribe el número 2; si los dos vértices del lado están
denotados con la letra B, sobre ese lado escribimos el número 12 ; si uno de los vértices
del lado corresponde a la letra A y el otro corresponde a la letra B, sobre ese lado
escribimos el número 1. Determine el producto de todos los números escritos.

19. Se tiene una cuadrı́cula de 4 × 4 y se quiere llegar del cuadrito inferior izquierde al
cuadrito superior derecho. Si solamente se puede ir hacia arriba, o hacia la derecha
o en diagonal hacia arriba y a la derecha, ¿de cuántas maneras se puede hacer?

20. Determine todos los números enteros n para los cuales 2n3 − 1 es múltiplo de 1999.

21. Sea AL la bisectriz del ángulo A de un triángulo acutángulo ABC . Sean M un punto
en el lado AB y N un punto en el lado AC de modo que m∠MLA = m∠ABC y
m∠NLA = m∠AC B. Si D es el punto de intersección de AL y MN, pruebe que
AL3 = AB · AC · AD.

22. Sea ABC un triángulo y D, E, F puntos sobre los lados BC , AC y AB respectivamente,


tales que AD, BE y C F concurren en un punto G. Trazamos DF , DE y por G una
paralela a BC que corta a DF y DE en H e I respectivamente. Pruebe que HG = GI.

23. Se tienen 10 números enteros positivos no necesariamente diferentes. Se realizan las


siguientes operaciones: se descarta el primero y se suman los nueve restantes; luego
el que se descarta es el segundo y se suman los nueve restantes; se continúa de esa
41

manera hasta que finalmente se descarta el último y se suman los otros nueve. De
esta forma se obtiene solo nueve resultados diferentes que son: 86, 87, 88, 89, 90, 91,
92, 93 y 96. Hallar los diez números iniciales.

24. Suponga que los números reales x, y, z, t satisfacen las siguientes condiciones:

x+y+z+t =0
x 2 + y2 + z 2 + t 2 = 1

Pruebe que −1 ≤ xy + yz + zt + tx ≤ 0.

25. En un trapecio ABC D de bases AB y C D, M es el punto medio de DA. Si BC = a,


MC = b y el ángulo MC B mide 150◦ , hallar el área del trapecio ABC D en función
de a y b.

26. Sea a un entero positivo impar mayor que 17, tal que 3a − 2 es un cuadrado perfecto.
Demostrar que existen enteros positivos distintos b y c tales que a + b, a + c, b + c
y a + b + c son cuatro cuadrados perfectos.

27. Siendo a, b, c, d números reales, probar que

48a2 + 16b2 + 4c2 + d2 ≥ 32ab + 16ac + 8ad.

28. En un 4ABC se tiene BC = 2, AB > AC √. Se traza la1altura AH, la mediana AM y la


bisectriz interior AL. Se dan ML = 2 − 3 y MH = 2 .

a) Calcular (AB)2 − (AC )2 y la razón de los lados AB y AC .


b) Hallar las longitudes AB, AC y AM.
c) Calcular los ángulos del triángulo ABC .

299998 − 1
29. Pruebe que es un número entero.
3
30. Determine todos los números primos que se pueden escribir como una suma de dos
números primos y que también se pueden escribir como una diferencia de dos números
primos.

31. Se tienen tres mezclas compuestas de tres elementos A, B y C . La primera mezcla


consta sólo de los elementos A y B en razón de peso 3 : 5, la segunda mezcla contiene
solamente los elementos B y C en razón de peso 1 : 2, en la tercera mezcla aparecen
solo los elementos A y C en razón de peso 2 : 3. ¿En qué proporción se deben tomar
estas mezclas para que la mezcla obtenida contenga los ingredientes A, B y C en
razón de peso 3 : 5 : 2?
42

32. Sean a, b, c números racionales no nulos tales que a1 + b1 + c1 = abc. Demuestre que
el número    
a2 b2 + 1 b2 c2 + 1 c2 a2 + 1

es el cuadrado de un número racional.

33. Sea ABC un triángulo equilátero de lado a, D punto medio de BC , E un punto en


AB, de modo que DE es perpendicular a AB, F un punto en AC , de modo que DF es
perpendicular a AC . Calcular el área de 4DEF .

34. Sean x, y, z números reales tales que:


(y − z)2 + (z − x)2 + (x − y)2 = (y + z − 2x)2 + (z + x − 2y)2 + (x + y − 2z)2
Probar que x = y = z.

35. Sea ABC un triángulo rectángulo, con ángulo recto en C ; D y E son puntos sobre
la hipotenusa tales que C D = m, C E = n y se verifica que m2 + n√ 2 = 1. Además
3
BD = DE = EA. Comprobar que la hipotenusa de 4ABC es AB = 5 5.

36. Sean a1 , a2 , . . ., an números enteros tales que:


i) a1 · a2 · · · · · an = n
ii) a1 + a2 + · · · + an = 0
Pruebe que 4 divide a n.

37. Determinar todos los enteros n ≥ 1 para los cuales es posible construir un rectángulo
de lados 15 y n, con piezas congruentes a:

Nota: las piezas no deben superponerse ni dejar huecos; los cuadritos de las piezas
son de lado 1.

38. Observe la siguiente sucesión de fracciones:

1 1 1 1 1
, , , , , ...
5 45 117 221 357

Determine una ley de formación para esta sucesión y calcule la suma de los 13
primeros términos.
43

39. En la siguiente figura, ABC D es un paralelogramo y DX = BY . Si el perı́metro del


triángulo BC E es a + 2b, el perı́metro del triángulo C DX es b − 2a (b > 2a) y el
perı́metro del triángulo C F Y es p, determine p en términos de a y b.

X
D
F C

A E
B
Y

40. Hallar un número menor que 1000 tal que el cubo de la suma de sus cifras es igual
a su cuadrado.

41. Se da un cuadrado de lado a y un triángulo equilátero de lado a, como en la figura.


Determine el área de la región sombreada.

42. Pedro y Cecilia participan en un juego con las siguientes reglas: Pedro elige un número
entero positivo a y Cecilia le gana si encuentra un número entero positivo b, primo
con a, tal que en la descomposición en factores primos de a3 + b3 aparecen por lo
menos tres factores primos distintos. Demostrar que Cecilia siempre puede ganar.

43. Encuentre las soluciones enteras positivas del sistema

w + x = yz
y + z = wx

44. Un grupo de 15 amigos que están sentados en fila inician el siguiente juego. El primero
de ellos entrega al segundo cierta cantidad de dinero y éste al recibirla duplica la
cantidad recibida y le agrega d colones más, se la entrega al siguiente quien a su
vez hace lo mismo y ası́ sucesivamente. Si el primer amigo entregó C k 5 y el décimo
recibió C
k 1 917, 50, ¿cuánto dinero recibe el último?
44

45. El ángulo A de un triángulo isósceles ABC mide 25 de un ángulo recto, siendo con-
gruentes sus ángulos B y C . La bisectriz del ángulo C corta al lado opuesto en el
punto D. Calcule las medidas de los ángulos del triángulo BC D. Exprese la medida a
del lado BC en función de la medida b del lado AC , sin que en la expresión aparezcan
razones trigonométricas.
p2 + q 2
46. Pruebe que si p y q son números primos tales que es entero, entonces p = q.
p+q
47. Encontrar todos los números naturales de tres dı́gitos abc (a 6= 0), tales que

a2 + b2 + c 2

es divisor de 26.

48. Si a, b, c son números positivos tales que a + b + c = 2, pruebe que

a2 b2 c2
+ + ≥ 1.
a+b b+c c+a

49. Durante cierto año (no bisiesto), una pequeña librerı́a que abrı́a los siete dı́as de la
semana vendió en total 600 libros. Cada dı́a vendió al menos un libro; pruebe que
existe un perı́odo de dı́as consecutivos en el que vendieron 120 libros en total durante
ese perı́odo.

50. Determine todas las secuencias finitas de números naturales consecutivos que sumen
2000.

51. Sea ABC D un cuadrado y sea M punto medio de BC y N el punto medio de AB. Sea
P la intersección de AM con DN, demuestre que el ángulo ∠APN es recto.

52. Sea xm el número que resulta de multiplicar las cifras del entero positivo m. Halle la
suma de todos los xm tales que m tiene n cifras.

53. Pruebe que si a y b son tales que la suma de sus cuadrados es un cuadrado perfecto
entonces, existe un c tal que la suma de los cuadrados de a, b y c también es un
cuadrado perfecto.

54. Sea ABC un triángulo tal que AB = 11, AC = 10, BC = 9. Se marca un punto M
en el lado AB y un punto N en el lado AC de manera que el perı́metro del triángulo
AMN sea igual al perı́metro del cuadrilátero MNC B y el área del triángulo AMN
1 1
sea igual al área del cuadrilátero MNC B, calcule el valor de + .
AM AN
55. Por el punto medio de la hipotenusa de un triángulo rectángulo se traza una recta
que corta al cateto mayor con un ángulo de 45◦ . Si la hipotenusa mide h, calcule,
en términos de h, la suma de los cuadrados de los segmentos determinados de esta
manera en ese cateto.
45

56. Determine los números naturales a, b, c tales que:

a3 − b3 − c3 = 3abc y a2 = 2(b + c)

m 5 √
57. Si m y n son enteros positivos tales que m = 3n−1, demostrar que + < 14.
n 4mn
58. En la siguiente figura se tiene que AP = PQ = QB y AM = MN = NC , determinar
el área de 4DMN en términos del área de 4ABC .

A
M
P
D N
Q

B C

59. En el triángulo acutángulo ABC , mb y ma denotan las medianas de los lados BA y


C A respectivamente. Suponga que ma ⊥mb . Muestre que cot B + cot C ≥ 32 .

60. Se escriben los números 1, 2, 3, ..., 20 en la pizarra. Se permite borrar cualesquiera


dos números a, b y en lugar de ellos escribir el número ab + a + b, ¿qué número
quedará escrito después de 19 de estas operaciones?

61. Pruebe que para todo número entero positivo n, se tiene que el número 11 . . 11}22
| .{z . . 22} 5
| .{z
n veces n+1 veces
es un cuadrado perfecto.

62. Los sı́mbolos (a, b, ..., g) y [a, b, ..., g] denotan respectivamente el máximo común
divisor y el mı́nimo común múltiplo de los enteros positivos a, b, ..., g. Por ejemplo:
(3, 6, 18) = 3, [6, 15] = 30. Pruebe que

[a, b, c]2 (a, b, c)2


=
[a, b] · [b, c] · [c, a] (a, b) · (b, c) · (c, a)

NIVEL C
SELECCION
1. La función f es tal que, para cada número real x, se cumple que

f (x) + f (x − 1) = x 2 .

Si f (1) = 1, entonces f (10) es igual a


a) 10 c) 55
b) 1 d) 385
46

2. Al simplificar la expresión

1 1
p √ +p √
x+2 x−1 x−2 x−1

para 1 < x < 2, se obtiene como resultado


2 2
a) √ c)
x−1 2−x

x−1 2−x
b) d)
x+2 x+1
3. Existe un número de dos cifras tal que si se le agrega 1 el resultado es un cuadrado
y si se le agrega 1 a su mitad se obtiene también un cuadrado. La suma de las cifras
de ese número es
a) 13 c) 9
b) 10 d) 12

4. En la figura, ABC DEF es un hexágono regular y C es un cı́rculo con centro en B.

F A

B H
E
P

D C

La razón del área sombreada entre el área del hexágono es


1 3
a) 3 c) 4
2 4
b) 3 d) 5

5. La cantidad de números naturales de cuatro dı́gitos de la forma X 2Y Z (donde X , Y ,


Z son cifras, X 6= 0) que son divisibles por 30 es
a) 51 c) 40
b) 30 d) 21

6. En la figura:
m]ABC + m]BC A = 5m]BAC ;

O es el punto de intersección de las bisectrices de los ángulos internos de 4ABC .


47

A G

Entonces la medida de ]BOC es igual a


a) 135◦ c) 105◦
b) 115◦ d) 150◦
7. Si F (x) es la suma de los dı́gitos impares del número x, entonces el resultado de la
suma
F (1970) + F (1971) + · · · + F (2001)
es
a) 1970 c) 769
b) 2001 d) 536

8. En un prisma rectangular, las áreas de tres de sus caras laterales son 72 cm2 , 32 cm2 ,
144 cm2 . Si las medidas de los lados son números enteros, entonces uno de los lados
del prisma mide
a) 16 cm c) 18 cm
b) 2 cm d) 6 cm

3
a
9. Si logab a = 4, entonces logab √ es igual a
b
15 17
a) 6 c) 8
17 19
b) 6 d) 8

10. Existen dos triángulos, uno con medidas 10, 10, 12 y otro con medidas 10, 10, x tales
que tienen igual área. Entonces
a) 8 ≤ x < 12 c) 14 < x ≤ 17
b) 12 < x ≤ 14 d) 17 < x ≤ 20
11. En la tabla que se muestra abajo, n es el número de la casilla en la cual, por primera
vez, el número de la fila inferior es mayor que el número en la fila superior:

1 2 3 4 ··· n
1000 1004 1008 1012 ··· ···
20 27 34 41 ··· ···

La suma de las cifras de n es igual a


48

a) 10 c) 12
b) 11 d) 13

12. En el cuadrado ABC D, M es el punto medio de C D, MR = 4 · RA y AD = 10.

D A
R

C B

Entonces, la distancia de R al lado C B es


a) 9 c) 7, 5
b) 8 d) 7

13. Si k es un número positivo y f es una función tal que para todo número positivo n
 2
√n
se cumple f n + 1 = k, entonces para todo número positivo m, la expresión
√ 12
  
9 + m2 m
f es igual a
m2
√ √
a) k c) k k
b) 2k d) k 2

14. En la siguiente figura, C es un punto en la recta m tal que BC < AB; además, si D


es el punto intersección de AB con m, entonces BD 6= AB.

A B m

El número de puntos E que podemos encontrar en la recta m tal que el triángulo


ABE es isósceles es
a) 5 c) 3
b) 4 d) infinito

15. El triángulo ABC es equilátero


√ y sus lados AC y BC son tangentes al cı́rculo cuyo
centro es O y cuyo radio es 3.
49

O C

El área del cuadrilátero AOBC es:



a) 2 √3 c) 2π√
b) π 3 d) 3 3

16. El número de soluciones enteras de la ecuación 23+x + 23−x = 65 es


a) 3 c) 1
b) 2 d) 0
17. En la siguiente figura, la suma u + v + w es igual a

a) 3u c) 360◦
b) 180◦ d) no se puede saber
18. En la siguiente figura, si el área del hexágono regular es H, entonces el área del
triángulo ABC es

B
A

C
50

a) H
2 c) H
6

b) H
4 d) H
8

19. El número de tripletas de números reales (x, y, z) que satsifacen el sistema de ecua-
ciones

x+y=2
xy − z 2 = 1

es
a) 0 c) 2
b) 1 d) 3

20. Los lados de un hexágono equiángulo miden x, y, 10, 6, 12 y 14, en ese orden. El
perı́metro del hexágono es

a) 60 c) 84
b) 72 d) 96

21. Sea N = 1 + 2 + 3 + · · · + 1011 . El número de veces que aparece el factor 2 en la


descomposición prima de N es

a) 8 c) 10
b) 9 d) 11

22. El número de tripletas de números enteros positivos (r, n, x), con x > 1, n > 1, que
satisfacen la ecuación x 2n+1 = 2r + 1 es
a) 0 c) 3
b) 2 d) infinito

23. En una conferencia internacional se reunen 15 delegados de África, América, Asia y


Europa. Cada continente envı́a un número diferente de delegados y cada uno está re-
presentado, por lo menos, por un delegado. América y Asia envı́an un total de 6
delegados, Asia y Europa envı́an un total de 7 delegados. El continente que envı́a 4
delegados es

a) Asia c) América
b) Europa d) Africa

24. Sea ABC un triángulo isósceles con AB = AC = 17 cm y P un punto cualquiera del


lado BC diferente de los puntos extremos. Por P se trazan: una paralela a AC que
corta a AB en Q y una paralela a AB que corta a AC en R.
51

B C
P

El perı́metro del cuadrilátero AQPR es


a) 34 cm c) 51 cm
b) 17 cm d) 144 cm

25. En una cuadrı́cula 3 × 3 se colocan, de alguna manera, los números del 1 al 9. A


cada segmento interior de longitud 1 de la cuadrı́cula, se le asigna el número que
resulta de sumar los dos números de los cuadrados que tienen al segmento en común,
por ejemplo, en la cuadrı́cula adjunta, al segmento entre los dos puntos se asigna el
número 13 = 6 + 7.

6 7

Sea S la suma de los doce números asignados a los segmentos interiores. Entre todas
las formas posibles de colocar los nueve números en las cuadrı́culas, el valor máximo
de S es
a) 224 c) 180
b) 128 d) 134

26. El número de primos p tales que 2p + 1 es un cubo es


a) uno c) tres
b) dos d) infinito

27. Si r, s, t son números reales positivos tales que t


x+r =2y t
r−x = 3, entonces se tiene
que
a) r < t < x c) x < r < t
b) r < x < t d) t < x < r
52

28. La colección infinita de números

1, 2, 4, 5, 7, 9, 10, 12, 14, 16, ...,

se forma de la siguiente manera: se coloca el primer impar (1), luego los dos siguientes
pares (2 y 4), luego los tres impares siguientes (5, 7 y 9), luego los cuatro pares que
le siguen al último impar colocado, luego los cinco impares que le siguen al último
par colocado y ası́ sucesivamente. El número par más cercano a 2000 que aparece en
esta colección es
a) 2002 c) 1996
b) 2026 d) 2030

29. Sea ABC un triángulo y sea P un punto en el segmento AB. Sea D un punto en una
paralela a AB que pasa por C tal que AB = C D. Entonces podemos asegurar que
a) AC = PD c) (ABC ) = (C DP)
b) m]C AB = m]PDC d) PC = C B
p √
30. El número 2 + 3 es igual a
√ r r
3 1 3 √
a) + c) + 2
√2 √2 2
6 2 √ √
b) + d) 2 + 3
2 2
31. En la siguiente figura ADC B es un cuadrado de lado 1, 4C MN es equilátero.

D C

A B
N

El área (C MN) es igual a


3 1
a) 8 c) 2
√ 1

b) 2 3 − 1 d) 2 3

32. En una fiesta habı́a distintos tipos de cajas de galletas de tal modo que los asistentes
pudieron establecer las siguientes conclusiones:

i) De cada caja de galletas comieron exactamente tres personas.


ii) Cada persona escojió galletas de exactamente dos cajas distintas.
53

iii) Por cada par de cajas hubo exactamente una persona que comió de ambas.

El número mı́nimo de personas que comieron galletas es


a) 6 c) 8
b) 4 d) 10

33. En la figura adjunta el cuadrilátero AC DE es un trapecio tal que ED = 15 cm,


AC = 24 cm y la altura es 12 cm.

E D

A C
B

Sabiendo que B es el punto medio del lado AC , el área del cuadrilátero OBC D es
a) 234 cm2 c) 32 cm2
16
b) 3 cm2 d) 112 cm2

34. En un cı́rculo de radio R se inscribe un triángulo equilátero, dentro del triángulo un


cı́rculo y dentro de éste, otro triángulo equilátero, y ası́ sucesivamente, entonces el
área del cı́rculo número n es igual a
√  n−1
πR n 3 1
a) c) πR 2
2 4
 n−1
3 πR n
b) πR 2 d)
4 2

35. La suma de los dı́gitos del número 22n+1 · 52n+3 − 1 (con n ∈ N) es igual a
a) 4n + 3 c) 18n + 15
b) 2n + 4 d) 18n + 9

36. Sean a, b, c, d números reales distintos tales que a y b son las raı́ces de la ecuación
cuadrática x 2 − 3xc − 8d = 0; c y d son las raı́ces de x 2 − 3ax − 8b = 0. Entonces
a + b + c + d es igual a
a) 64 c) 32
b) 96 d) 80

37. En la figura adjunta BC une los centros de los cı́rculos. AB es perpendicular a BC .


BC = 8 y AC = 10. Entonces el perı́metro del cı́rculo pequeño es igual a
54

B C

a) 2π c) 6π
b) 4π d) 8π

38. Se tienen dos ángulos A y B, ambos entre 0◦ y 90◦ . Si


q
3
sin A − sin B = 2,
q
1
cos A + cos B = 2,

entonces A + B es igual a
a) 60◦ c) 120◦
b) 90◦ d) 45◦

39. En la figura, ABC D es un cuadrado y C EF es un triángulo equilátero de área 3
cm2 , entonces el área del cuadrado es

D C
E

A B
F

2

a) 4 cm√ c) 1 + 3 cm2
b) 2 + 3 cm2 d) 2 cm2

40. En la siguiente figura se dan tres semicircunferencias mutuamente tangentes, C D y DA


son diámetros de las circunferencias menores, el punto B está en la semicircunferencia
mayor y el segmento BD es perpendicular al diámetro C A.
55

C A
D

Entonces el área sombreada es igual a


a) π
4 CB · AB c) π2 BD 2
b) π
2 CB · AB d) π4 BD 2
41. La cantidad de enteros positivos de la forma 4a8b2 (a, b son cifras) que son divisibles
entre 72 es igual a
a) 2 c) 4
b) 6 d) 13
42. La suma
sin 1◦ sin 1◦ sin 1◦
S= + + + · · · +
cos 0◦ · cos 1◦ cos 1◦ · cos 2◦ cos 1998◦ · cos 1999◦
es igual a
a) tan 0◦ c) tan 1998◦ − tan 1999◦
b) tan 1999◦ d) 2 tan 1999◦

43. En la figura, la recta T es tangente al cı́rculo y paralela al segmento DE.

A R T

D E

Si AD = 6, AE = 5, C E = 7, entonces BD es igual a
a) 3, 5 c) 5
b) 4 d) 5, 5

44. En una circunferencia se consideran cuatro puntos distintos A, B, C , D tales que AD


es diámetro. Se traza la recta L tangente a la circunferencia en D. Sea P el punto
56


→ ←

intersección de L con AB y Q el punto intersección de L con AC . Si AB = 46, 08,
AC = 28, 8 y BP = 3, 92, entonces la medida del segmento C Q es igual a
a) 31, 2 c) 51, 2
b) 41, 2 d) 61, 2
45. La siguiente figura muestra dos cuadrados de lado 1 cm, donde AMPQ se ha obtenido
de ABC D al girar este cuadrado 45◦ sobre el vértice A.

C
B

M
Q

45◦
A D

Entonces el área sombreada, en centı́metros cuadrados, es


√ √
a) 2√− 2 c) 2 −√ 1
b) 2 + 1 d) 3 + 2 2

46. En la figura, el área del cı́rculo mayor es 1 m2 . El cı́rculo menor es tangente inter-
namente al cı́rculo mayor y también es tangente a los lados del ángulo inscrito que
mide 60◦ .

A 60

Entonces el área del cı́rculo menor es


a) 94 c) 29
b) 94 π d) 29 π

47. El valor de n ∈ N, para el cual 932 − 65 · 930 + 3n , es cuadrado perfecto, se encuentra


en
57

a) ]0,10[ c) ]23,54[
b) ]11,17[ d) ]61,67[
48. Dado ABC D un cuadrilátero, constrúyase el cuadrilátero A1 B1 C1 D1 que une los puntos
medios de AB, BC , C D y DA; el cuadrilátero A2 B2 C2 D2 que une los puntos medios de
A1 B1 , B1 C1 , C1 D1 y D1 A1 y ası́ sucesivamente. Entonces el área (An Bn Cn Dn ) es igual
a
(ABC D) (ABC D)
a) c)
n (1/2)n
(ABC D) (ABC D)
b) √ d)
n 2n

DESARROLLO
1. Sea f una función de R en R definida por
q q 
1 2 2
f (x) = (x + 1) − (x − 1)
2x
si x 6= 0 y f (0) = 1. Determinar el ámbito de f .

2. Considere 109 enteros a1 , . . ., a109 tales que 0 < a1 < a2 < . . . < a109 < 1999.
Muestre que entre los valores

di = ai+1 − ai , i = 1, . . . , 108,

hay un valor que se repite 4 o más veces .


Encuentre un ejemplo de enteros

0 < a1 < a2 < . . . < a109 ≤ 1999

donde ninguna diferencia di = ai+1 − ai se repita más de tres veces.

3. Demuestre que para cualquier número real x, es válida la desigualdad

4 sin 3x + 5 ≥ 4 cos 2x + 5 sin x.

4. Se tienen dos secuencias de números

a) 3, 7, 11, 15, . . . , 1147


b) 2, 9, 16, 23, . . . , 2004

Determine cuántos números pertenecen a la vez a ambas secuencias.

5. En una circunferencia está inscrito un rectángulo ABC D, con AB = a. Sea K P el


diámetro de la circunferencia que es paralelo al lado AB. Si la medida de ∠BK C es
α, calcule el radio de la circunferencia en términos de a y de α.
58

6. Resuelva el sistema de ecuaciones


 √ √ √
x − y = x + xy
(x + y)2 = 2(x − y)2

7. Se tiene un cuadrado ABC D, una recta pasa por el centro de este cuadrado e interseca
al lado AB en un punto N, tal que NB
AN
= 21 . En esta recta se toma un punto arbitrario
M que se encuentra dentro del cuadrado. Probar que las distancias de M a los lados
AB, AD, BC , C D, tomados en el orden indicado, forman una progresión aritmética.

8. Determine los valores de x (en términos de a) que resuelven la ecuación


 
1
loga (ax) · logx (ax) = loga2 ,
a

donde a > 0, a 6= 1.

9. El radio de la base de un cono circular recto es R y su altura es H. Determine, cuál


de los cilindros inscritos en este cono tiene la mayor superficie lateral.

10. Determine todas las parejas (x, y) de números reales tales que

y = x 3 − 3x
x = y3 − 3y

11. Sean a y b números reales y sea

1
f (x) = .
ax + b

Determine las condiciones que deben cumplir a y b para que haya tres números reales
distintos x1 , x2 , x3 tales que f (x1 ) = x2 , f (x2 ) = x3 , f (x3 ) = x1 .

12. Sea ABC D un cuadrado de lado 1 y sean E, F , G, H puntos sobre los lados AB, BC ,
C D, DA respectivamente.
√ Muestre que el perı́metro del cuadrilátero EF GH es mayor
o igual a 2 2.

13. Sean a y b números reales tales que a + b = 2. Pruebe que a4 + b4 ≥ 2.

14. Determinar todos los números enteros x tales que existen enteros p y q con

x = 3p + 1
x = 4q + 2
59

15. Encontrar todos los números reales x1 , x2 , . . ., x1999 tales que

1 + x12 = 2x2 ,
1 + x22 = 2x3 ,
···
2
1+ x1998 = 2x1999 ,
2
1+ x1999 = 2x1 .

16. Seis circunferencias tienen un punto en común. Pruebe que hay una de ellas que
contiene al centro de otra de las circunferencias.

17. Determinar todos los valores del parámetro a de manera que el sistema
 2
x + y2 = z
x+y+z =a

tenga solución única.

18. Considere los números del 1 al 14 y un tetraedro, se desea asignar uno de esos números
a cada una de las 4 caras, un número a cada uno de los 4 vértice y un número a cada
una de las seis aristas del tetraedro de manera que el número asignado a cada arista
sea igual al promedio de los números asignados a los vértices que la determinan e
igual al promedio de los números asignados a las caras que la comparten. Determine
si es posible o no realizar lo indicado.

19. Una circunferencia C tiene radio 5 y dos cuerdas AB y C D, que no se intersecan ni


son paralelas y AB = 8, C D = 6. Sea M el punto medio de AB y Q la intersección,
←→ ← →
en el interior de C, de DB y AC . Pruebe que QM⊥DC .

20. Determinar todos los enteros no negativos x, y, z tales que 2x + 3y = z 2 .

21. Un trapecio inscrito en una circunferencia de radio r tiene tres lados de longitud s
y el cuarto de longitud r + s, con s < r. Determine las medidas de los ángulos del
trapecio.

22. Pruebe que el número 1 se puede escribir de una infinidad de maneras distintas en
la forma
1 1 1
1= + + ··· + ,
5 a1 an
donde n y a1 , a2 , . . . , an son enteros positivos y 5 < a1 < a2 < . . . < an .

23. Un número es suertudo si al sumar los cuadrados de sus cifras y repetir esta operación
suficientes veces obtenemos el número 1. Por ejemplo 1900 es suertudo: la suma de
los cuadrados de las cifras de 1900 es 82, la suma de los cuadrados de las cifras de
60

82 es 68, la suma de los cuadrados de las cifras de 68 es 100 y, finalmente, la suma


de los cuadrados de las cifras de 100 es 1.
Determine una infinidad de parejas de números enteros consecutivos, donde ambos
números sean suertudos.

24. En un tablero de 5 × 9, se realiza el siguiente juego. Inicialmente, un número de discos


se colocan aleatoriamente en algunas de las casillas, no más de un disco por casilla.
En cada turno, todos los discos se mueven de acuerdo con las siguientes reglas:

a) Cada disco se puede mover una casilla arriba, o una abajo, o una a la izquierda
o una a la derecha.
b) Si un disco se mueve arriba o abajo en un turno, dicho disco se debe mover a la
izquierda o a la derecha en el turno siguiente, y viceversa: si un disco se mueve
a la izquierda o a la derecha en un turno, debe morverse hacia arriba o hacia
abajo en el turno siguiente.
c) Al final de cada turno, ninguna casilla debe contener 2 o más discos.

El juego finaliza si es imposible completar otro turno. Probar que si inicialmente se


colocan 33 discos en el tablero, entonces el juego finalizará después de cierto número
de turnos. Probar también que se puede ubicar 32 discos en el tablero de tal manera
que el juego se pueda continuar indefinidamente.

25. (a) Pruebe la identidad


cos 3x = 4 cos3 x − 3 cos x

(b) Haciendo uso de la fórmula en (a), pruebe que



−1 + 5
sin 18 =

.
4

26. Pruebe que si a es un entero positivo que no tiene factores comunes con 240, entonces
240 divide a a4 − 1.

27. Sean a, b y c números reales no nulos, con suma no nula, tales que
1 1 1 1
+ + = .
a b c a+b+c

Pruebe que también se verifica que


1 1 1 1
+ + = .
a1999 b1999 c1999 a1999 + b1999 + c1999

28. Determinar todos los números enteros positivos x, y, z, tales que x < y < z y, además,
xy + 1 yz + 1 zx + 1
, , sean enteros.
z x y
61

29. Encuentre los valores de x, y, z tales que


 √
 x − √yz = 42
y − xz = 6
 √
z − xy = −30

30. En un triángulo ABC , los segmentos BN, BL y BM son, respectivamente (y en ese


orden de izquierda a derecha), altura, bisectriz del ángulo ABC y mediana del triángulo.
Sabiendo que los ángulos ABN, NBL, LBM, MBC son congruentes, determine las
medidas de los ángulos internos del triángulo.
31. Calcular el valor de N, donde N es igual a
(104 + 324)(224 + 324)(344 + 324)(464 + 324)(584 + 324)
(44 + 324)(164 + 324)(284 + 324)(404 + 324)(524 + 324)

32. En un triángulo isósceles su base mide a y sus lados congruentes miden b, el ángulo
no congruente mide 20◦ . Probar que
a3 + b3 = 3ab2 .

33. Para un triángulo ABC con circuncentro O, considere H la intersección de las alturas
BE y C F y M el punto de intersección de AO con C H. Pruebe que si F M = EH,
entonces el triángulo es isósceles.
34. Determine todas la tripletas (x, y, z) de números reales que son solución del sistema

 3 3 3
 x +y +z =8


 2
x + y2 + z 2 = 22

 1 1 1

 z
 + + =−
x y z xy

35. Suponga que f es una función tal que


 
2x + 29
2f (x) + 3f = 100x + 80.
x−2
Calcule f (3).
36. Pruebe que si se seleccionan siete enteros positivos distintos de entre los números 1,
2, ..., 126, existirán dos de ellos x e y tales que 1 < yx ≤ 2.
37. Sea a una constante real. Dado el siguiente sistema de ecuaciones:

x sin θ + y cos θ = 2a sin 2θ
x cos θ − y sin θ = a cos 2θ

determine una relación entre x e y, en la que no aparezca θ.


62

38. Sea f una función definida en el conjunto de los números reales tal que existe k ∈
R − {0} fijo con
a) f (x + y) − f (x − y) = k4 xy, para todo x, y ∈ R
b) f (xy) = kf (x)f (y), para todo x, y ∈ R
Determine f (x).
39. Determine todos los números reales x tales que
3
1 + sin3 x + cos3 x = sin 2x.
2
40. Determine todas las funciones f , definidas en el conjunto de los números enteros no
negativos, tales que para cualesquiera x, y; con x ≥ y, se satisface
f (x + y) + f (x − y) = f (3x).

41. Dado a > 0, resolver el sistema de ecuaciones (incógnitas x1 , x2 , . . ., xn ):



x1 − x2 = a;

x2 − x3 = a;
...;

xn−1 − xn = a;

xn − x1 = a.

42. Sea el polinomio


f (x) = x n + a1 x n−1 + a2 x n−2 + · · · + an−1 x + an ,
donde a1 , . . ., an son números enteros. Suponga que existen cuatro números enteros
diferentes a, b, c, d tales que
f (a) = f (b) = f (c) = f (d) = 5.
Pruebe que no existe ningún número entero k tal que f (k) = 8.
43. Considere un triángulo isósceles. Si los radios de las circunferencias circunscrita e
inscrita al triángulo miden, respectivamente, 9 cm y 4 cm, halle la distancia entre los
centros de las dos circunferencias.
44. Sea f (x) = 125x 4 − 5logx 5 . Determine para qué valores en el dominio de f se tiene
que f (x) ≤ 0.
45. Encuentre todos los enteros que se escriben como
1 2 9
+ + ··· +
a1 a2 a9
donde a1 , a2 , . . ., a9 son dı́gitos distintos de 0 que pueden repetirse.
63

46. Sea f una función tal que existe un número real a con
q
1
f (x + a) = + f (x) − [f (x)]2 .
2

Pruebe que f (x + 2a) = f (x).

47. Sea n un número entero positivo par. Determine todas las tripletas de números reales
(x, y, z) tales que
x n y + yn z + z n x = xyn + yz n + zx n

48. Sea ABC DE un pentágono convexo (las diagonales quedan dentro del pentágono).
Sean P, Q, R y S los baricentros de los triángulos ABE, BC E, C DE y DAE respec-
tivamente. Demostrar que PQRS es un paralelogramo y que su área es igual a 92 del
área del cuadrilátero ABC D.

49. Sea ABC un triángulo acutángulo. C1 y C2 son circunferencias que tienen a los lados
AB y C A como diámetros, respectivamente. C2 corta al lado AB en el punto F (con
F 6= A) y C1 corta al lado C A en el punto E (con E 6= A). Además, BE corta a C2 en
P y C F corta a C1 en Q. Demostrar que AP = AQ.

50. Sean ABC D un cuadrado de lado 1, P, Q puntos sobre los lados BC , C D respecti-
vamente, con m∠PAQ = 45◦ y E, F los lados de intersección de PQ con AB y AD
respectivamente.
a) Muestre que PQ es tangente a la circunferencia de centro A y radio 1.

b) Muestre que AE + AF ≥ 2 2.

51. En la figura se tiene que ABC


√D es un cuadrado. El radio del cı́rculo menor es 1 y
el del cı́rculo mayor es 1 + 2. Los cı́rculos son concéntricos. Calcule el área del
cuadrado.

C
B

A
D

52. Encontrar un número N de cinco cifras diferentes y no nulas que sea igual a la suma
de todos los números de tres cifras distintas que se pueden formar con las cinco cifras
de N.
64

53. Hay cierto número n (con n > 80) de personas a la orilla de un rı́o. Sus pesos en
kilogramos son a1 ≥ a2 ≥ ... ≥ an−1 ≥ an . Estas personas deben atravesar el rı́o y
solo disponen de un bote; en el bote caben a lo más dos personas y el peso máximo
que puede llevar en cada viaje es an +a80 . En cada viaje tanto de ida como de regreso
el bote debe ser conducido por alguna de las n personas. Si el bote debe atravesar
el rı́o al menos 355 veces, ¿cuántas personas hay?

54. En un tablero de 8×8 están escritos ordenadamente los números del 1 al 64; como se
ve en la figura.
1 2 3 4 5 6 7 8
9 10 11 12 13 14 15 16
17 18 19 20 21 22 23 24
25 26 27 28 29 30 31 32
33 34 35 36 37 38 39 40
41 42 43 44 45 46 47 48
49 50 51 52 53 54 55 56
57 56 57 58 59 60 61 64
Luego se coloca un signo + o un − delante de cada número de manera que al final
queden 4 números positivos y cuatro números negativos en cada fila y queden cuatro
números positivos y cuatro negativos en cada columna. Se suman los 64 números
ası́ obtenidos. Determine todos los posibles resultados de esta suma.

55. En la siguiente figura, ABC D es un paralelogramo tal que AB = 1, BC = 2 y ∠ABC


es obtuso. También BMDN es un paralelogramo tal que MB⊥BC y ND⊥DA. Si los
dos paralelogramos indicados son semejantes, determine el área de ABC D.

B
C

A
D

56. Sean A y B dos puntos cualesquiera dentro del cı́rculo, C. Determine las condiciones
más generales bajo las cuales se pueda trazar un triángulo rectángulo, inscrito al
cı́rculo, tales que A y B están sobre los catetos del triángulo.

57. Sea el polinomio

f (x) = x 5 + a1 x 4 + a2 x 3 + a3 x 2 + a4 x − 7
65

donde a1 , ..., a4 son números enteros. Suponga que existen cinco enteros diferentes a,
b, c, d, e tales que f (a) = f (b) = f (c) = f (d) = f (e) = 5. Hallar el valor numérico de
166(abcde) + 8.

58. Muestre que si n ≥ 1, entonces la suma de todos los divisores de n es menor o igual

que n n.

59. Probar que


1 1 1 1
3< √ √ +√ √ +√ √ + ··· + √ √
2+ 5 8 + 11 14 + 17 596 + 599

60. Si en un triángulo ABC , tenemos que


sin A + sin B + sin C √
= 3 (1)
cos A + cos B + cos C
entonces al menos uno de los ángulos mide 60◦ .

61. Encontrar todas las funciones f de R en R tales que, para cualesquiera números
reales x1 , ..., x2000 , se tiene

f (x1 ) · f (x2 ) · · · · · f (x2000 ) = f (x1 ) + f (x1 + x2 ) + · · · + f (x1 + x2 + · · · + x2000 )

62. Suponga que f es una función que cumple f (x + y) = f (xy) para todo número real
estrictamente positivo. Pruebe que f es una función constante.

63. Demuestre que en un polı́gono regular la suma de las distancias de un punto sobre
cualquiera de los lados a los demás lados es constante.

64. Dado untriángulo 4ABC ,con ortocentro O. Entonces si R está sobre la perpendicular
a AB por O y es tal que la distancia de R a AB es igual que la distancia de O a
AB; P está sobre laperpendicular a BC por O y es tal que la distancia de P a BC
es igual que la distancia de P a BC y Q está sobre la perpendicular a AC por O y
es tal que la distancia de Q a AC es igual que la distancia de Q a AC . Demuestre
que O es el incentro del 4PQR.

65. Una función f (n) está definida para todos los números enteros positivos y toma valores
no negativos. Además, para todo par de enteros m, n, se tiene que
f (m + n) − f (m) − f (n) es igual a 0 o a 1,
f (2) = 0, f (3) > 0 y f (9999) = 3333.
Pruebe que para todo entero k, con 1 ≤ k ≤ 3333, se tiene que f (3k) = k.

66. Pruebe que para cualesquiera números reales a, b, p, q con a ≥ 0, b ≥ 0, p > q > 0,
se cumple la desigualdad

(ap + bp )1/p ≤ (aq + bq )1/q .


66

67. Sean O1 , O2 , O3 los vértices de un triángulo equilátero de lado d. Se dibujan cir-


cunferencias congruentes de radio r (con d2 < r < d) con centros en cada uno
de los tres vértices, como lo muestra la figura. A, B, C son los puntos de inter-
sección entre los cı́rculos que quedan en el interior de 4O1 O2 O3 . D, E, F son los
puntos de intersección de los tres cı́rculos en el exterior de 4O1 O2 O3 . Pruebe que
4O1 O2 O3 ∼ 4ABC ∼ 4DEF .

O3

E D

C
A B

O1 O2

F
Eliminatorias del año 2000

Primera eliminatoria
III ciclo (7◦ ,8◦ ,9◦ )
1. Se tienen tres cofres: uno de oro, otro de plata y el último de plomo. Cada uno de
ellos contiene la siguiente inscripción:

Plata Oro Plomo


El retrato El retrato El retrato
no está en está en este no está en el
este cofre cofre cofre de oro

Si solamente una de las inscripciones es verdadera podemos afirmar que:


(a) El retrato está en el cofre de plomo
(b) El retrato está en el cofre de plata
(c) El retrato está en el cofre de oro
(d) No es posible determinar en cual está
2. Hay cuatro botes en una de las orillas de un rı́o, sus nombres son Ocho, Cuatro, Dos y
Uno, porque esa es la cantidad de horas que tarda cada uno de ellos en cruzar el rı́o.
Se puede atar un bote a otro, pero no más de uno, y entonces el tiempo que tardan
en cruzar es el del más lento de los dos botes. Un solo marinero debe llevar todos los
botes a la otra orilla. La menor cantidad de tiempo que se necesita para completar el
traslado es
(a) 12 horas (c) 16 horas
(b) 15 horas (d) 14 horas
1 1
3. Si 2a = 5b = 10 entonces + es igual a
a b
1
(a) (c) 10
10
1 1
(b) + (d) 1
2 5

67
68

4. El jefe de unos bandidos decı́a a sus hombres: “hemos robado unas piezas de tela. Si
cada uno de nosotros toma seis piezas quedarán cinco piezas. Pero si cada uno de
nosotros quiere siete piezas, nos faltarán ocho piezas”. El número de piezas robadas
es
(a) 78 piezas (c) 83 piezas
(b) 15 piezas (d) 18 piezas.
5. Marı́a tiene 24 años. Tiene dos veces la edad que Ana tenı́a cuando Marı́a tenı́a la
misma edad que Ana tiene ahora. La edad de Ana está entre
(a) 10 y 12 años (c) 16 y 18 años
(b) 13 y 15 años (d) 19 y 21 años.
6. Una asamblea está constituida por profesores, estudiantes y administrativos. Si el
número de profesores en la asamblea es de 570, el número de estudiantes constituye
un quinto del total de la asamblea y el número de admimistrativos constituye un sexto
del total de la asamblea, entonces el total de asambleistas es
(a) 900 (c) 521
(b) 697 (d) 540
7. El número más grande que se puede escribir usando cuatro treses y potencias es
3
(a) 3333 (c) 333
33
(b) 33 (d) 3333 .

8. Si en la figura adjunta los segmentos AC y EF son paralelos y m(∠ABG) = m(∠BGE).

B
A C

D
E F

Se tiene
a) m∠EGF = 12 m∠BF E
b) m∠GEF = 12 m∠BF E
c) m∠GEF = 12 m∠EF G
d) m∠GF E = 21 m∠EGF
9. El número 555 555 puede descomponerse como producto de dos factores de tres dı́gitos
en
(a) una manera (c) tres maneras
(b) dos maneras (d) ninguna manera
69

10. Si el número 1a1b1c1d1 con a, b, c y d dı́gitos distintos no cero, es divisible por 11,
la cantidad de valores distintos que pueden tomar a, b, c y d es
(a) 240 (c) 10
(b) 40 (d) 160

11. Si el promedio de tres números es 85 y el promedio de otros dos es 95, entonces el


promedio de los cinco es
(a) 88 (c) 90
(b) 89 (d) 91

12. Cada movimiento en un juego consiste en invertir dos flechas adyacentes, la posición
inicial es ↑↑↑↓↓↓ y la posición final es ↑↓↑↓↑↓ el número mı́nimo de movimientos
para llegar a la posición final es
(a) 1 (c) 4
(b) 2 (d) 3

13. La suma de los dos últimos dı́gitos de 72000 es igual a


(a) 1 (c) 13
(b) 7 (d) 11

4
14. La cifra que ocupa el lugar 2000 en la expresión decimal del número racional es
101
(a) 9 (c) 0
(b) 6 (d) 3

15. Con seis varillas se construye una estructura como se muestra en la figura adjunta

Las tres varillas exteriores son congruentes entre sı́ y las tres varillas interiores son
congruentes entre sı́.
Se desea pintar cada varilla de un solo color de modo que en cada punto de unión,
las tres varillas que llegan tengan distinto color. Las varillas solo se pueden pintar de
azul, blanco, rojo o verde. El número de maneras en que se puede pintar la pieza es
igual a
(a) 4 (c) 16
(b) 12 (d) 15
70

16. La cantidad de números que hay del 1 al 1000 que pueden escribirse en la forma ab
con a, b enteros mayores que 1, es
(a) 43 (c) 49
(b) 40 (d) 50
17. En la selva, la hiena miente los lunes, martes y miércoles; la zorra miente los jue-
ves, viernes y sábado. En los dı́as que no mienten, ellas dicen la verdad. Un dı́a se
encontraron la hiena y la zorra y sostuvieron un diálogo
Hiena: –Hola zorra!
ayer yo mentı́
Zorra: –Hola hiena!
Yo También mentı́ ayer.
Entonces el dı́a en que sucedió este encuentro es:
(a) lunes (c) jueves
(b) martes (d) nunca pudo suceder
18. Llegan 4 niños a una fiesta y hay 6 gorros; tres verdes y tres rojos. A cada niño se le
coloca su gorro respectivo con los ojos vendados, se sientan a una mesa circular de
manera que cada niño ve los gorros de los otros tres. Empezando por el niño número
uno y en el sentido contrario a las manecillas del reloj a cada niño se le hace la
pregunta ¿Sabes ya de qué color es tu gorro? y todos escuchan la respuesta hasta
que alguien contesta afirmativamente. Además el primer niño dice que no. El primero
de estos niños que contesta afirmativamente, es
(a) ninguno (c) el tercero
(b) el segundo (d) el cuarto
19. En el paı́s de la maravillas hay 10 duendes encantados, numerados del uno al diez, que
cambian de color entre rojo y verde. En la primera semana todos son rojos, la segunda
semana los múltiplos de dos cambian a verde, la tercera semana los múltiplos de 3
cambian al color rojo, y ası́ alternadamente hasta que la décima semana los múltiplos
de 10 cambian al color verde. De los diez duendes, los que quedaron pintados de rojo
al final son los numerados por
(a) impares (c) pares
(b) divisores de 10 (d) múltiplos de 10.
20. Construimos una secuencia de triángulos isósceles empezando con AB = BC , luego
BC = C D, y ası́ sucesivamente.

D
B

A C
71

Si la m∠BAC = 17◦ , el número de triángulos isósceles que podemos dibujar es


(a) 2 (c) 4
(b) 3 (d) 5
2000
21. El número de enteros positivos n que hacen que la expresión sea entera es
n + 19
(a) 15 (c) 12
(b) 20 (d) 13
22. Los enteros mayores que uno se ordenan en cinco columnas como se indica a conti-
nuación:

A B C D E
2 3 4 5
9 8 7 6
10 11 12 13
17 16 15 14
... ... ... ... ...

El entero 1993 queda en la columna


(a) E (c) B
(b) A (d) C
23. Un rectángulo con perı́metro 176 cm está dividido en cinco rectángulos congruentes
como se muestran en la figura

El perı́metro de cada uno de los rectángulos congruentes es igual a


(a) 72 cm (c) 40 cm
(b) 80 cm (d) 88 cm
24. Susana tiene un quinto de la edad de su madre. La edad de su madre es menor que
cien y cuando es dividida por dos, por tres, por cuatro, por seis y por ocho, el residuo
es uno. Cuando es dividida por cinco el residuo es cero. La suma de las edades de
Susana y de su madre es igual a
(a) 30 (c) 36
(b) 24 (d) 42
72

25. Si x, y son números tales que 2x 2 − x + 14 = 20 y x 2 − x2 + 7 = y, entonces 5y2 − 3


es igual a
(a) 402 (c) 497
(b) 396 (d) 452

26. En la figura se tiene: ∠AOB es obtuso, OC ⊥ OA, OD es bisectriz de ∠AOB, OE es


bisectriz de ∠BOC .
A
D

C
E

O B

Entonces el ángulo ∠DOE mide


(a) 25◦ (c) 45◦
(b) 30◦ (d) 40◦
27. Se colocan los números enteros positivos en la siguiente manera

fila 1 1 2 3 4
fila 2 5 6 7 8
fila 3 9 10 11 12
fila 4 13 14 15 16
... ... ... ... ...

Entonces la suma de los términos en la fila 50 igual a


(a) 986 (c) 810
(b) 794 (d) 576
28. En cierto año, el mes de enero tuvo exactamente cuatro martes y cuatro sábados. Ese
año el dı́a 23 de enero cayó en
(a) lunes (c) miércoles
(b) domingo (d) jueves
29. Un número entero M, diferente de cero, es el cuadrado de un cuadrado y tiene a 6
M
como factor, entonces el menor valor de es
6
(a) 216 (c) 36
(b) 1296 (d) 6
1
30. La suma 2 + 23 + · · · + 1999
2000 + 12 + 13 + · · · + 1
2000 es igual a
3998
(a) 2000 (c) 2000
3500
(b) 1000 (d) 1999
73

IV ciclo (10◦ ,11◦ ,12◦ )


a c
1. Sean a, b, c, d números números reales positivos tales que = , entonces
b d
√ abcd √ abcd
a) abcd = c) ac + bd =
2 2
√ ad + bc √ ad + bc
b) abcd = d) ac + bd =
2 2
2. Se tienen tres cofres, uno de oro, otro de plata y el último de bronce, cada uno de
ellos tiene una inscripción:

Plata Oro Plomo


El retrato El retrato El retrato
no está en está en este no está en el
este cofre cofre cofre de oro

Si solamente una de las inscripciones es verdadera podemos afirmar que


a) El retrato está en el cofre de plomo
b) El retrato está en el cofre de plata
c) El retrato está en el cofre de oro
d) No es posible determinar en qué cofre está el retrato.
3. En la figura, ABC D es un cuadrado de lado 4 cm, E es el punto medio de BC .

E
B C

F
A D

El triángulo F ED tiene área igual a 7 cm2 , entonces la medida de AF es igual a


a) 1 cm c) 1, 5 cm
b) 2 cm d) 0, 5 cm
1 1
4. Si 2a = 5b = 10 entonces + es igual a
a b
1
a) c) 10
10
1 1
b) + d) 1
2 5
74

5. El número de parejas de enteros positivos impares que tienen suma 1998 es igual a
a) 503 c) 499
b) 500 d) 999
6. El total de números de cuatro cifras que son divisibles por 30 es igual a
a) 333 c) 300
b) 297 d) 321
7. En el siguiente arreglo cuadrangular

67 a 43
b c d
e 73 x

La suma de los elementos en las filas, las columnas y las diagonales es una constante.
El valor de x es
a) x = 37 c) x = 31
b) x = 13 d) x = 7
8. Si el valor de la expresión
    
1 1 1
1− 1− ··· 1 − 2
4 9 n

es 2−1 + 2000−1 , entonces el valor de n es


a) 1000 c) 1001
b) 2000 d) 2001
9. En la figura, los dos cuadrados tienen el mismo centro, la razón entre el lado del
2
cuadrado menor y el lado del cuadrado mayor es .
5

Entonces la razón entre el área sombreada y el área del cuadrado mayor es


1 23
a) c)
10 100
1 21
b) d)
8 100
75

10. La cifra que ocupa el lugar número 2000, después de la coma, en la expresión decimal
4
del número racional es
101
a) 9 c) 0
b) 6 d) 3

11. Construimos una secuencia de triángulos isósceles empezando con AB = BC , luego


BC = C D, y ası́ sucesivamente tal como lo muestra la figura.

D
B

A C

Si m]BAC = 17◦ , el número de triángulos isósceles que podemos dibujar es

a) 2 c) 4
b) 3 d) 5

12. Sea un número de tres dı́gitos diferentes P = abc, donde a, b y c pueden ser 1 o un
número primo y P es múltiplo de a, b y c. La cantidad de números P que cumplen
estas condiciones es
a) 6 c) 12
b) 10 d) 15

13. Hay cuatro botes en una de las orillas del rı́o, su nombres son Ocho, Cuatro, Dos
y Uno, porque esa es la cantidad de horas que tarda cada uno de ellos en cruzar
el rı́o. Se puede atar un bote a otro, pero no más de uno, y entonces el tiempo que
tardan en cruzar es igual al del más lento de los dos botes que van atados. Un solo
marinero debe llevar todos los botes a la otra orilla. La menor cantidad de tiempo que
se necesita para completar el traslado es

a) 12 horas c) 16 horas
b) 15 horas d) 14 horas

14. En la figura adjunta, la longitud del segmento AC es


76

60◦

4 4

C
3 3

D B

√ √ √ √
a) 2 3 − 5 c) 2 3 + 5

4 3
b) 3 d)
3
15. En la figura, los cuadrados ABIJ, BC GH, C DEF , son adyacentes; los puntos F , H, J
están alineados y AB = 4, BC = 7, C D = x.

F E

H G

J I

A 4 B 7 C x D

Entonces x es igual a
33
a) c) 10
4
49
b) d) 11
4
1 1 1
16. La cantidad de pares de números enteros positivos (n, m) que satisfacen + =
n m 3
es igual a
a) 4 c) 1
b) 6 d) 3

17. Sea ABC un triángulo, D el punto medio de BC , M el punto medio de AD y N un


punto sobre la prolongación de AD, tal que DN = DM. Entonces, de las siguientes
afirmaciones:
I) AB = NB II) AD = MN III) C M = NB
77

Son ciertas
a) todas c) Solo I y III
b) Solo I y II d) Solo II y III

18. En la suma A = (12 + 1) + (22 + 2) + (32 + 3) + · · · + (19982 + 1998), el dı́gito de las


unidades es
a) 0 c) 4
b) 2 d) 8

19. Los botones de un teléfono están colocados como lo muestra la figura. Si los botones
están a un centı́metro de distancia uno de otro, midiendo de centro a centro.

1 2 3

4 5 6

7 8 9

Cuando se marca el número 5927018, la distancia (en cm) que viaja el dedo es
√ √ √
a) √5(3 + √2) + 2 √2
b) 2√ 5(1 +
√ 2) +√2 2
c) 2√ 5 +√ 2(3 + √5)
d) 5 + 2(2 + 3 5)

20. La cantidad de enteros positivos menores que 1998 que tienen un número impar de
divisores positivos es
a) 30 c) 44
b) 45 d) 50

21. Sea P un número entero de tres dı́gitos tal que la diferencia de P con el número
que se obtiene al intercambiar las unidades y las decenas es un cuadrado perfecto
no cero, entonces el número de valores posibles para P es
a) 144 c) 32
b) 135 d) 16

22. Si E es el punto medio del lado AC del triángulo rectángulo ABC de la figura.
78

60◦ 30◦
B C
D

Entonces la razón entre los perı́metros de los triángulos rectángulos DEC y ABC , es

1 3
a) c)
3 4
2 1
b) d)
5 2
23. Al factorizar un número entero positivo n se obtiene n = pα1 1 pα2 2 , con p1 , p2 primos
diferentes y α1 , α2 enteros positivos. Sabemos que n2 tiene 143 divisores positivos,
entonces n3 tiene
a) 341 divisores po- c) 304 divisores po-
sitivos sitivos
b) 143 divisores po- d) 430 divisores po-
sitivos sitivos

24. En un cubo de lado 2, M, N, P, Q son puntos medios de los lados mostrados en la


figura.

M N

La distancia máxima entre un punto de MN y otro de PQ es igual a


√ √
3 6
a) c)
2 2
r
3 √
b) d) 6
2
25. La cantidad de parejas de números enteros positivos (x, y) que satisfacen 2x + 5y =
2000 es igual a
79

a) 99 c) 199
b) 149 d) 249

26. Se tienen 2000 piezas rectangulares de 2 cm de ancho y 3 cm de largo y con ellas


se arman cuadrados (sin superposiciones ni huecos). La mayor cantidad de cuadrados
no congruentes que se pueden tener al mismo tiempo es
a) 10 c) 12
b) 15 d) 9

27. Los niños A, B y C toman 13 dulces de una mesa, al final, A dijo: “tomé dos dulces
más que B”, B dijo: “tomé la mitad de dulces que A y 5 menos que C ”, y finalmente C
dijo: “tomé un número par de dulces”. Si sabemos que a lo sumo uno de ellos mentı́a,
el mentiroso es
a) A c) B
b) C d) ninguno

28. Se tiene cierta cantidad de piedras, cada una de ellas pesa un kilogramo o menos
y si se separan en dos grupos, de cualquier forma, siempre quedará un grupo en el
que el peso total de las piedras es menor o igual que 1 kilogramo. Entonces el peso
máximo del total de las piedras es
a) 3 kg c) 2,5 kg
b) 2 kg d) 3,5 kg

29. En la figura 4APD es un triángulo rectángulo.

A 5
B

P
16

D C
12

Si el perı́metro de 4APD es menor que 41, entonces (APD) es igual a



a) 40 c) 4 √
281
b) 61 d) 50 2

30. En la pantalla de una calculadora inicialmente se tiene un 1. Si se apreta la tecla A


el número en la pantalla se multiplica por 3; si se apreta la tecla B, el número en la
80

pantalla se disminuye en 1. Utilizando alguna secuencia de A y B hay que llegar a


obtener un 97 en la pantalla. El menor número de teclas que se debe apretar es
a) 20 c) 11
b) 15 d) 8

Segunda eliminatoria
III ciclo (7◦ ,8◦ ,9◦ )
SELECCIÓN

1. En una cartulina hay impresas cuatro afirmaciones:

1. En esta cartulina exactamente una afirmación es falsa.


2. En esta cartulina exactamente dos afirmaciones son falsas.
3. En esta cartulina exactamente tres afirmaciones son falsas.
4. En esta cartulina exactamente cuatro afirmaciones son falsas.

La cantidad de afirmaciones falsas que hay en la cartulina es


a) 4 c) 2
b) 3 d) 1

2. La cantidad de números de 10 dı́gitos que contienen solamente ceros y unos y que


además son divisibles por 9 es
a) 8 c) 10
b) 9 d) 11

3. Si la mitad de 5p es 3u, entonces la tercera parte de 10p es


a) 3u c) u
b) 6u d) 4u

4. Cuatro amigos, Juan, Julia, Jorge y Jacinta fueron a almorzar. El menú ofrece ensalada
mixta o ensalada rusa, carne de pollo, pescado, res o cerdo y de postre gelatina,
helado o flan. Cada uno comerı́a una ensalada, un tipo de carne y un postre. A Juan
no le gusta la ensalada rusa ni el pollo; a Julia no le gusta la ensalada mixta ni el
helado; a Jorge no le gusta ni el pescado ni la gelatina y a Jacinta no le gusta ni la
carne de res ni la de cerdo. Quien tiene menor número de posibilidades para formar
su almuerzo es
a) Juan c) Jorge
b) Julia d) Jacinta

5. En la siguiente figura, m]BAC = α, m]BPC = m]BQC = 90◦ .


81

P
H

B C

Entonces m]BHC es igual


a) 3α c) 180◦ − α
b) α d) 2α

6. Daniel y Raquel disponen de 3 y 5 litros de leche, respectivamente, y deciden com-


partirla con Miguel, por lo que la reunen y la dividen en partes iguales. Si Miguel
paga 800 colones por su parte de leche y deben repartirlos entre Daniel y Raquel de
manera que a cada uno le corresponda según su aporte de leche a la parte de Miguel
entonces les corresponden:
a) 350 a Daniel y 450 a Raquel c) 200 a Daniel y 600 a Raquel
b) 300 a Daniel y 500 a Raquel d) 100 a Daniel y 700 a Raquel

7. Se necesita pesar 14 kilos de frijoles pero solamente se dispone de una pesa de 3


kilos y otra de 19 kilos. Si cada uso de la pesa de 19 kilos cuesta 5 colones y cada
uso de la pesa de tres kilos cuesta un colón el costo mı́nimo para pesar los frijoles es
a) 18 colones c) 14 colones
b) 16 colones d) 12 colones

8. La suma de tres números es 205, la razón entre el primer número y el segundo es 52


y la razón entre el segundo número y el tercero es 34 , entonces la suma de los dos
primeros números es igual a
a) 100 c) 30
b) 75 d) 105

9. El valor de
1 · 2 · 4 + 2 · 4 · 8 + 3 · 6 · 12 + · · · + 100 · 200 · 400
1 · 3 · 9 + 2 · 6 · 18 + 3 · 9 · 27 + · · · + 100 · 300 · 900
es
1 2
a) 729 c) 3
1 8
b) 27 d) 27

10. En la figura adjunta ABC D es un cuadrado de lado a y C E = 2a.


82

A D
F

E
B
C

Entonces el área del triángulo ADF es igual a


a) 16 a2 c) 23 a2
b) 13 a2 d) 14 a2
11. Tanto al numerador como al denominador de una fracción ab se le suma el denominador
de ab ; a la fracción resultante se le resta ab y, de este modo se obtiene como resultado
la misma fracción ab . Entonces ab cumple que
1 1 1
a) 6 < a
b < 4 c) 2 < a
b <1
1 1 3
b) 4 < a
b < 2 d) 1 < a
b < 2

12. La suma de las cifras del número que se obtiene al desarrollar 102000 − 2000, es igual
a
a) 17 978 c) 17 972
b) 17 968 d) 17 962

13. En la figura adjunta ABC D es un cuadrilátero con AD√paralelo a BC , la diagonal AC


es perpendicular al lado C D, m]BAC = 30◦ , AC = 4 3 y AB = BC .

B C

30◦
A D

Entonces el área de ABC D es igual a


√ √
a) 12√3 c) 16√ 3
b) 24 3 d) 8 3
14. El número de parejas (x, y) de números enteros positivos tales que x y y son primos,
3 −y3
x 6= y y x xy es un número entero es
a) 0 c) 4
b) 2 d) infinito
83

15. Si ABC es un triángulo tal que su área es 18, AB = 5 y las medianas trazadas desde
los vértices A y B son perpendiculares entre sı́, entonces el valor de la suma AC + BC
es
√ √ √ √
a) 73 + 2 13 c) 12 73 + 2 13
√ √ √ √
b) 115 3 + 13 d) 225 3 + 13

DESARROLLO

1. Un tubo contiene cinco bolas numeradas del uno al cinco. Tiene una abertura A en un
extremo, una abertura B en el otro extremo y una abertura C en el medio en la parte
superior, tal como se muestra en la siguiente figura.

B
A 1 2 3 4 5

Una jugada consiste en sacar la última bola por la abertura B e introducirla por A,
desplazando las demás bolas hacia la derecha o introducirla por C, desplazando las
dos últimas bolas hacia la derecha. Por ejemplo, si en algún momento las bolas se
encuentran en la siguiente posición 14352 hay dos jugadas posibles: una es sacar la
bola 2 por B e introducirla por A, quedando la nueva posición ası́: 21435 y la otra
es introducir la bola 2 por C, quedando la nueva posición ası́: 14235. Si inicialmente
las bolas se encuentran en la siguiente posición 12345 (como aparecen en la figura
anterior), determine el mı́nimo número de jugadas necesario para llegar a la posición
25413.

2. Determine todos los números x tales que x es un número entero positivo impar de
tres cifras tal que cada una de sus cifras es divisible por tres y, además, exactamente
tres de los divisores de x son números primos.

3. Sea ABC D un trapecio con bases AB y C D. Las diagonales AC y BD se cortan en P


←→
y las prolongaciones de los lados AD y BC se cortan en Q. Pruebe que PQ corta a
las bases en sus puntos medios.

IV ciclo (10◦ ,11◦ ,12◦ )


SELECCIÓN

1. Si un triángulo está inscrito en un semicı́rculo de radio r, el área máxima que puede


tener es
a) 12 r 2 c) 32 r 2
b) 2r 2 d) r 2
84

2. La expresión  
4999 1 1 1
− + + ··· +
2000 2·3 3·4 1999 · 2000
es igual a
1
a) 0 c) 2000
3
b) 2 d) 2000
√ √ √
3. Si x, y son reales positivos, entonces una expresión equivalente con y + xy + x
es:
√ p √ √ p √
a) y + p y + xy + 2x y c) y + x + xy + 2x x
√ √ √ p √
b) x + y + xy + 2y x d) x + x + xy + 2y y

4. El siguiente arreglo de números se construye de la siguiente manera: la primera fila


está formada por las potencias de 3, desde 30 hasta 399 . A partir de la segunda fila,
cada término es la resta del que está en la fila anterior a la derecha del término
menos el que está arriba a la izquierda.
1 3 9 27 81 · · · 398 399
2 6 18 54 ··· 399 − 398
4 12 36 ··· ··· ···
8 24 ··· ···
··· ··· ···
El último término de la fila número 51 es igual a
a) 251 · 349 c) 251 · 350
b) 250 · 350 d) 250 · 349

5. En la figura, AB es diámetro de la circunferencia, m]ADC = α, m]DAB = β y


m]DAC = 75◦ .

A
75◦
β
D α

C
B

Entonces β es igual a
a) α + 25◦ c) α − 15◦
b) α − 25◦ d) α + 15◦
85

6. Sea p un número de cuatro cifras, multiplique al número p por 59, tome las últimas
cuatro cifras de este nuevo número y multiplique por 3; finalmente tome las últimas
cuatro cifras de este otro número y multiplique por 113. Si el resultado final es 2112,
entonces el número original es
a) 2211 c) 2112
b) 1122 d) 2121
7. Una persona sabe un secreto y decide contárselo a p número de personas, a su vez
cada una de estas p personas decide contárselo a p + 1 personas y cada una de
las últimas que lo saben a su vez lo cuenta a p + 2 personas. Si nadie cuenta el
secreto a una persona que ya lo sabı́a, entonces, el número de personas que terminan
conociendo el secreto es
a) p(p + 1)(p + 2) c) (p + 1)(p2 + 2p + 3)
b) (p + 1)(2p + 3) d) (p + 1)(p2 + 3p + 1)
8. Se reparten 10 confites entre Hugo, Eduardo, Mario y Elizabeth, de manera que
cada uno reciba al menos un confite, Elizabeth reciba exactamente tres confites y
Eduardo reciba a lo sumo dos confites. El número de maneras en que puede hacerse
la repartición es
a) 11 c) 8
b) 10 d) 9

9. Sean x, y, z números enteros positivos con x, y impares y sea T = 3x + (y − 1)2 z.


Entonces, al dividir T entre 4 se obtiene como residuo
a) 0 c) 2
b) 1 d) 3

10. Si (1 − 14 )(1 − 19 ) . . . (1 − k12 ) es el racional m


n y k es un número par, entonces el valor
de m + n es
a) 3k + 1 c) k + 1
b) 3k − 1 d) k − 1
11. Al trazar las diagonales del siguiente pentágono, éste queda subdividido en 10 triángu-
los y un pentágono interior (sombreado).

En los triangulitos se van a colocar números enteros positivos de modo que la suma de
todos ellos sea igual a 66; la suma de los números colocados en los triangulitos que
86

comparten un lado con el pentágono sombreado es 42. Para cada vértice del pentágono
mayor hay tres triangulitos que comparten ese vértice; la suma S de los tres números
colocados en esos triangulitos es la misma para todos los vértices. Entonces, el valor
de S es igual a
a) 8 c) 18
b) 20 d) 22

12. Sea ABC D un trapecio con AB paralelo C D y M el punto medio de AD. Si AB = 8,


C D = 6, C M = 7 y m]DC M = 30◦ , entonces el área de ABC D es
a) 48 c) 50
b) 49 d) 51

13. Un conejo es perseguido por un perro. El conejo le lleva una ventaja inicial al perro
de 50 de sus saltos. El conejo da 5 de sus saltos mientras el perro da 2 de sus saltos;
además se sabe que el perro, en 3 de sus saltos avanza lo mismo que el conejo en 8
de los suyos. El número de sus saltos que debe dar el perro para alcanzar al conejo
es
a) 240 c) 300
b) 270 d) 360

14. Un punto P está en el interior de un triángulo equilátero ABC de lado 2 3. Los
puntos L, M, N se encuentran en los lados AC , BC y AB respectivamente y son tales
que PL es perpendicular a AC , PM es perpendicular a BC y PN es perpendicular a
AB. Entonces PL + PM + PN es igual a

a) 3 c) 23 √3
b) 2 d) 32 3

15. Si
1 1 1
M = √ + √ + ... √
1 2 10000
entonces se cumple que
a) 197 < M < 198 c) 199 < M < 200
b) 198 < M < 199 d) 200 < M < 201

DESARROLLO

1. Determine todos los pares de números reales (x, y) tales que


 2

 x y

 x + y = −2

 xy2 4

 =
x−y 15
87

2. En la figura, 4ABC es isósceles, con AC = BC , su base AB mide 6 cm y su altura


trazada desde el vértice C mide 12 cm. AQ, BP y C R son las medianas del triángulo
ABC . Los puntos D, E y F , se encuentran, uno sobre cada mediana, de manera que
AD = BE = C F = 2 cm. Determine el área del triángulo DEF .

F
P Q

D E

A B
R

3. Hallar todos los números a, b enteros con 1 < a ≤ b tales que (a − 1)(b − 1) sea
divisor de ab − 1.

Tercera eliminatoria (final)


III ciclo (7◦ ,8◦ ,9◦ )
Primer dı́a

1. Una mañana la sra. Martı́nez, la Sra. Pérez, la Sra. Torres y la Sra. Gómez fueron
de compras. Cada una tenı́a que ir a dos lugares diferentes. Una tenı́a que ir a la
zapaterı́a, dos tenı́an que ir al Banco, dos tenı́an que ir a la carnicerı́a y tres tenı́an
que ir a la panaderı́a.
Sabemos que:

Dora no fue a la panaderı́a


Tanto Ester como la señora Gómez fueron a la carnicerı́a
Margarita fue al Banco
La sra. Pérez no fue a ninguno de los lugares donde estuvieron Lucı́a y la Sra.
Torres.

¿Cuál es el apellido de Margarita?

2. En la figura, ABC D es un paralelogramo y BC = C E, m]BC E = x ◦ , m]ADC =


3x ◦ + 15◦ . ¿Cuál es la medida de ]DAB?
88

D C

A E
B

3. Se tiene un número entero de tres cifras. Si de él suprimimos la cifra de las centenas
obtenemos un número de dos cifras, si suprimimos la cifra de las decenes tenemos otro
número de dos cifras y si suprimimos la cifra de las unidades obtenemos un tercer
número de dos cifras. La suma de estos tres números de dos cifras es igual a la mitad
del número de tres cifras inicial. Determine todos los números de tres cifras con dicha
propiedad.

Segundo dı́a
|BM|
4. Sea el 4ABC y sea M sobre BC tal que = ρ, exprese el área del 4ABM en
|MC |
función del área del 4ABC .

5. Sea ABC D un paralelogramo. Se traza por D una recta que corta al lado BC en P
y a la prolongación del lado AB en Q. Si el área del cuadrilátero ABPD es 29 y el
área de 4DPC es 8. Determinar el área de 4C PQ.

C
B
P

A
D

6. Considere el número de siete dı́gitos P = 33xy49z (x, y, z son dı́gitos). Determine x,


y, z, de modo que el número P sea múltiplo de 693.

IV ciclo (10◦ ,11◦ ,12◦ )


Primer dı́a
1. Sea Γ un cı́rculo unitario (es decir, de radio 1), de centro O y sea AB un diámetro
de Γ. Sea ` una tangente a Γ por A. Trace un cı́rculo unitario con centro en A y
tome C uno de los puntos donde este cı́rculo corta a Γ. Se dibuja un tercer cı́rculo
también unitario y centro en C , sea D el punto distinto de O donde este cı́rculo corta
89

al segundo cı́rculo construido. Trace la recta OD que corta a ` en E y sea H sobre `,




en la dirección EA tal que EH = 3.
Demuestre que BH es solución de la ecuación 9x 4 − 240x 2 + 1492 = 0.

2. Distribuya los números 1, 2, 3, 4, 5, 6, 7, 8, 9, 10, 11, 12, 13, 14, 15, 16 en dos grupos de
8 números cada uno de manera que la suma de todos los números de un grupo sea
igual a la suma de todos los números del otro grupo y la suma de todos los cuadrados
de los números de un grupo sea igual a la suma de todos los cuadrados del otro
grupo.

3. Sean a1 , a2 . . . ak números primos, fórmense con ellos P1 , P2 , . . . , Pk+1 números com-


puestos cuyos factores son únicamente uno o varios entre a1 , a2 , . . . , ak . Demuestre
que o alguno entre P1 , P2 , . . . Pk+1 es cuadrado perfecto o el producto de varios de
ellos es cuadrado perfecto.

Segundo dı́a
←→
4. Sea AB el diámetro de un cı́rculo de centro O. BM es tangente al cı́rculo en el punto
←→ ←→
B. C F es tangente al cı́rculo en el punto E, e interseca a BM en C . La cuerda AE,
← →
cuando se prolonga, interseca BM en D. Pruebe que BC = C D.
2f (x + w) − 3
5. Sea f : R −→ R − {2}; y w > 0 un número real fijo tal que f (x) = .
f (x + w) − 1
Pruebe que f es una función periódica de perı́odo 3w.

6. Demuestre que a2 − a + 1 divide a

a2n+1 + (a − 1)n+2 .
90
Solución de los ejercicios propuestos

NIVEL A
SELECCION
1. (d) El cociente es
10020 03004 00500 60070 08009
que tiene 25 dı́gitos.

2. (a) El área de cada cuadrado es 100 cm2 . La parte superpuesta es 14 de cada cuadrado,
es decir 25 cm2 . De modo que el área de la figura es 200 − 25 = 175 cm2 .

3. (a) Juan puede ser el padre de Antonio y éste, a su vez, el padre de Carlos. En este
caso se satisfacen las condiciones del problema y Carlos es el nieto de Juan. Se
pueden analizar las otras situaciones y convencerse de que no son ciertas.

4. (c) Tenemos py = bxy , qx = bxy , entonces b4 = (bxy bxy )z = b2xyz ; luego xyz = 2.

5. (a) Sea x la medida del lado de cada cuadrado. Entonces 7x 2 = 112 ⇒ x 2 = 112÷7 =
16 ⇒ x = 4. Ası́, el perı́metro es 4 · 16 = 64 cm.

1 −1
6. (b) 3x 4x − 1 = 11 ⇒ 12x − 1 = 11 ⇒ 12x = 12 ⇒ x = 1.

7. (a) El menor número entero para el cual el 65 % es un número entero es 20.

8. (d) Para que en un piso paren exactamente cuatro ascensores, deben parar A, que para
en todos, y tres de los restantes: B, C y D paran en los múltiplos de 5 · 7 · 17 = 595;
B, C y E paran en los múltiplos de 5 · 7 · 23 = 805; B, D y E paran en los múltiplos
de 5 · 17 · 23 = 1955; C, D y E paran en los múltiplos de 7 · 17 · 23 = 2737. Entre 1
y 1000 hay un solo múltiplo de 595 que es 595; también hay un solo múltiplo de 805,
que es 805 y no hay múltiplos de 1955 ni de 2737. Por lo tanto, los pisos en los que
paran exactamente cuatro ascensores son el 595 y el 805.

9. (c) 80 = 24 · 5. Luego p = 2α1 · 5β1 , q = 2α2 · 5β2 , máx{α1 , α2 } = 4, máx{β1 , β2 } = 1.


Si α1 = 4 entonces α2 ∈ {0, 1, 2, 3, 4} salen 5 posibilidades:
(α1 , α2 ) ∈ {(4, 0), (4, 1), (4, 2), (4, 3), (4, 4)}. Si α2 = 4 entonces α1 ∈ {0, 1, 2, 3, 4} y

91
92

(α1 , α2 ) ∈ {(0, 4), (1, 4), (2, 4), (3, 4), (4, 4)}. En total hay nueve parejas. Si β1 = 1,
β2 ∈ {0, 1}. (β1 , β2 ) ∈ {(1, 0), (1, 1)}. Para el caso en que β2 = 1, β1 ∈ {0, 1};
(β1 , β2 ) ∈ {(0, 1), (1, 1)}. Se obtienen tres parejas. Total 9 · 3 = 27.

10. (a) Como el taxista nunca ha ido a un juego de beisbol y Becerra y Espinoza van
juntos al juego de beisbol, se sigue que Villanueva es el taxista.
Espinoza nunca ha oı́do hablar de sellos, por lo tanto Espinoza no es panadero y como
Villanueva es el taxista, Espinoza es el bombero. Luego, Becerra es el panadero.

11. (c) (ANC M) = (ABC D)−(ADM)−(ABN) = 20·16− 10·16 20·8


2 − 2 = 320−80−80 = 160.

a2 −b2 (a+b)(a−b)
12. (c) Se tiene que a
b − ab = ab = ab = a+b
a b y la suma de estos factores
· a−b
es ab + ab .

13. (a) En 4ABC , el lado mayor es BC pues se opone al ángulo mayor. Pero este lado
pertenece también a 4BC D en el cual el lado mayor es BD que se opone al ángulo
mayor (de 65◦ ). Ası́, BD es el mayor.

14. (b) Como 3 y 5 son impares, también lo son 313 y 1513 , de modo que m es par y por
lo tanto el menor primo que lo divide es 2.

15. (a) Para facilitar el análisis sea:


(1) Ningún apellido coincide con el lugar de nacimiento.
(2) El nacido en Trujillo no tiene el mismo apellido que el nombre del lugar de
nacimiento del sr. Bolı́var.
(3) El nacido en Lara no es el señor Sucre.
(4) El nacido en Lara no tiene como apellido el nombre del lugar de nacimiento del
sr. Lara.
Si en Sucre nació Lara, en Lara nació Bolı́var, en Trujillo nació Sucre y en Bolı́var
nació Trujillo se cumplen las condiciones pedidas. Verifiquemos que las otras no son
posibles.
 En Sucre no nació Sucre (por (1)).
 Si en Sucre hubiera nacido Trujillo, entonces, por (1) y (3) en Lara debió haber
nacido Bolı́var esto significa que Lara debió nacer en Bolı́var, lo que no es posible
por (4) o en Trujillo, lo que no es posible por (2). Ası́, en Sucre no pudo nacer Trujillo.
 Si en Sucre hubiera nacido Bolı́var, entonces, por (1) y (3), en Lara debió haber
nacido Trujillo esto llevarı́a a que Sucre nació en Trujillo, lo que no es posible por (2)
o que Sucre nació en Bolı́var, lo que no es posible por (4). De este modo, en Sucre no
nació Bolı́var.
Necesariamente en Sucre nació Lara.
93

16. (a) Sean n, n + 1, n + 2, n + 3 los números, entonces


(n + 3)3 − (n + 2)3 + (n + 1)3 − n3 = 2(3n2 + 9n + 10).

Es decir, el resultado es divisible por 2, pero 3n2 + 9n + 10 = 3(n2 + 3n + 3) + 1 no


es divisible por 3.
17. (c) Sea la medida del menor de los ángulos 2x, entonces los otros miden 3x y 4x; ası́,
2x +3x +4x = 180◦ , es decir x = 20◦ y la suma de los dos menores es 2x +3x = 100◦ .
18. (d) Entre el 20 y el 53 (sin contarlos a ellos) hay 32 números, entonces hay 32
bailarines en esas posiciones, cada uno tiene uno al frente, entonces hay otros 32, si
a ellos agregamos el número 20 y el número 53 tenemos 66 bailarines.
19. (b) Los cubos pueden tener 3, 2, 1, 0 caras azules:
◦ Con tres caras azules son los cubitos que resultan de los vértices del cubo grande,
en total 8. ◦ Con dos caras azules son los cubitos que resultan de las aristas del cubo
que no están en los vértices, hay 2 por arista y son 12 aristas, en total 24 cubitos. ◦
Con una cara azul son los cubitos que están en las caras del cubo grande pero no en
las aristas; hay 4 por cara y son 6 caras, en total 24. ◦ Finalmente, con 0 caras azules
son todos los restantes: 64 − 8 − 24 − 24 = 8.
20. (b) Tenemos:
m = (2a + 1)2 − (2b + 1)2
= 4(a − b)(a + b + 1)
= 4(a2 + a − b2 − b)
= 4[a(a + 1) − b(b + 1)].
Puesto que el producto de dos enteros consecutivos es siempre divisible por dos, en la
última expresión, el entero que aparece dentro de los paréntesis cuadrados es divisible
por 2 y por lo tanto m es divisible por 8.
21. (a) Si
a2 − m2 + 2ab + b2 a+b+m
= ,
a2 − m2 + ab + mb a+m
entonces
(a + b + m)(a + b − m) a+b+m
= .
(a + m)(a + b − m) a+m
Esta última igualdad es válida si y solo si a + b − m 6= 0.
22. (d) El triángulo C DE es isósceles, su altura es 4. Si P es el punto en BA tal que
EP ⊥ BA, entonces el triángulo EPA es rectángulo y, ası́,
EA2 = 102 + 32 = 109.

Concluimos que EA = 109.
94

" b c−a # c1 h
2a−b 2b−c i1
23. (a) Tenemos = 2bc+ac−2c2 c = 2a+b−2c = 20 = 1.
c−b
2c+b
24. (b) Sabemos que tres de los candidatos son astutos. Si Carlos y Antonio son igualmente
astutos, entonces ambos son astutos. Como Antonio es astuto entonces Rubén no lo
es; se deduce que son astutos: Oscar, Carlos y Antonio (1). Oscar y Carlos tienen
igual grado de inteligencia; si ambos fuesen altamente inteligentes entonces Antonio
y Rubén no lo serı́an. Esto significarı́a que el único candidato con firmeza es Rubén
(cada candidato cumple al menos uno de los requisitos). Pero, en este caso, ningún
candidato tendrı́a todos los requisitos. Por consiguiente, podemos concluir que los dos
candidatos altamente inteligentes son Antonio y Rubén (2). De (1) y (2) se deduce que
el único candidato que reúne todas las condiciones es Antonio; en consecuencia el
será el elegido.

25. (d) Si el número es abcd, debemos tener a2 + d2 = 13, solo hay dos posibilidades:
a = 2 y b = 3 o a = 3 y b = 2. Por otra parte b2 + c2 = 85 = 22 + 92 = 62 + 72 y
de aquı́ salen cuatro posibilidades para los valores de b y c. Hay entonces 2 · 4 = 8
números con la propiedad indicada.

26. (b) En general, k es el padre de los hijos: 3k − 1, 3k, 3k + 1. Como 2000 no es de la


forma 3k ni 3k + 1, entonces 2000 = 3k − 1 y por lo tanto k = 667 es su padre.

27. (c) Cuatro personas pueden satisfacer las condiciones planteadas. Por ejemplo A y B
son hermanos, C es hijo de A y D es hija de B. Entonces A es tı́o de D, B es tı́a de
C ; C y D son primos.

28. (b) Puesto que BD es mediana y altura del triángulo isósceles ABC , entonces en el
triángulo rectángulo ADB:

100 = 64 + BD 2 ⇒ BD = 6.

Por condición dada en el enunciado del problema: BF = 23 · 6 = 4. Luego F D =


BD −BF = 6−4 = 2. Finalmente, el área del triángulo rectángulo AF D es 21 ·8·2 = 8
cm2 .

29. (b) Como 4ABE es isósceles entonces E es el punto medio de DC ; como F C = F B


entonces F es el punto medio de BC ; como ABC D es un cuadrado, por lo anterior, se
tiene que EC = F C . Ası́, 4EC F es rectángulo e isósceles y por lo tanto

m∠EF C = 45◦ ⇒
m∠EF B = 180◦ − 45◦ = 135◦ .

30. (a) El segundo número se obtiene al dividir el primero entre 2, el tercero al dividir
el segundo entre 3, el cuarto al dividir el tercero entre 4; ası́, el quinto se obtiene al
dividir el cuarto entre 5, es decir, 60 ÷ 5 = 12.
95

31. (b) Si es isósceles, dos de los lados tienen que ser iguales. Consideramos los diferentes
casos: b + 1 = 7 − b ⇒ b = 3, ası́, los lados deberı́an ser 3 + 1 = 4, 7 − 3 = 4
y 4 · 3 − 2 = 10, pero estos no pueden ser los lados de un triángulo puesto que
4 + 4 < 10; este caso no se da. Ahora, si b + 1 = 4b − 2, entonces b = 1; los lados
serı́an 2, 5 y 2, que tampoco corresponden a los lados de un triángulo. Finalmente, si
7 − b = 4b − 2, entonces b = 59 , los lados serı́an 145 , 265 y 265 que si corresponden a
los lados de un triángulo. La situación se da solo para un valor de b.

32. (a) Tenemos que 1998 = 2 · 33 · 37, entonces n = 3 y 10n + c = 37 ⇒ 30 + c = 37


⇒ c = 7.

33. (d) m∠C OD = 21 m∠BOA y

m∠C OD + m∠C OB + m∠BOA = 180◦


3
m∠BOA + 120◦ = 180◦
2
m∠BOA = 40◦ .

34. (c) El número N tiene que ser divisible por 2 y por 3. Ası́, b tiene que ser par y la
suma de los dı́gitos tiene que ser divisible por 3, es decir, 9 + a + b divisible por 3.
En conclusión, para b = 0, tenemos a = 0, 3, 6, 9; para b = 2, a = 1, 4, 7; para b = 4,
a = 2, 5, 8; para b = 6, a = 0, 3, 6, 9 y para b = 8, a = 1, 4, 7. Son 17 maneras de
escoger la pareja de números a, b.

35. (d) Comparemos los triángulos C F D y C ED. Si pensamos que tienen sus bases sobre
la diagonal BD, vemos que tienen la misma altura, pero la base del primero es la mitad
de la base del segundo. Por lo tanto, (C F D) = 12 (C DE). Como las diagonales parten
el cuadrado en cuatro triángulos congruentes a C DE, el área de dicho triángulo es
un cuarto del área del cuadrado. Por lo tanto,

1 1
(C F D) = (ABC D) = .
8 8

B
A

E
F
D
C

36. (b)
96

C
x B

37. (a) Vemos que DE = C E y C E = AF − BC , DE = 1, entonces


AB + BC + C D + DE + EF + FA = 9, 41 Luego:

1 + 2 + C D + 1 + 1 + 3 = 9, 41; C D = 1, 41

38. (b) Tenemos que

1099 − 99 = 1 00 . . . }0 − 99 = 99
| {z . . . }9 01,
| {z
99 veces 97 veces

entonces la suma es 9 · 97 + 1 = 874.

39. (d) Podemos dividir la figura en 4 rectángulos iguales:

Cada rectángulo tiene área igual a 50. Si llamamos a los lados pequeños con x,
entonces x · 2x = 50⇒x = 5. Por lo tanto el perı́metro de la figura es

8x + 4(2x) = 8 · 5 + 4 · 10 = 80.

40. (d) Podemos seguir los caminos en el siguiente árbol, partiendo de 1, hasta llegar a
7 en cada caso.

1
2 3
3 4 4 5
4 5 5 6 5 6 6 7
5 6 6 7 6 7 7 6 7 7 7
6 7 7 7 7 7
7
97

41. (d) (MPQD) = 12 (BNDM) = 1


2 · 12 (ABC D) = 1
4 · 5 · 3 = 3, 75.

42. (a) Se puede escribir 16555 = 43 · 11 · 7 · 5. Luego, la madre tiene 43 años y las edades
de sus hijas son 5,7 y 11. Ası́, 11 − 5 = 6 es el dato pedido.

43. (a) Si ponemos otro triángulo igual sobre el lado AB como se muestra en la figura,
tenemos un paralelogramo AC BC 0 .

C C0

A B

Por lo tanto, las rectas paralelas miden todas 10 y la suma de los 7 segmentos dentro
de 4ABC es igual a 7·10 2 = 35.

44. (d) Para formar un cuadrilátero no debemos tomar 3 ó 4 vértices en una fila. Luego solo
se puede tomar dos de cada fila y para cada fila esto se puede hacer de 6 maneras
diferentes, de modo que hay 6 · 6 = 36 maneras de tomar los vértices.

45. (b) En el primer movimiento quedamos en la silla 2, en el segundo movimiento estare-


mos en la 1 o en la 3, es decir, una silla con número impar. Entonces, en los movimientos
impares estaremos sentados en una silla con número par y en los movimientos pares
estaremos sentados en sillas con número par. Ası́, después del movimiento 19 estare-
mos ubicados en una silla par. Luego quitamos las sillas 1 y 5, entonces, al realizar
el primer movimiento luego de esto, necesariamente estaremos en la silla 3 (antes
estábamos en 2 o en 4), en el segundo movimiento estaremos sentados en una silla
con número par y ası́ sucesivamente, de modo que en el movimiento 99 que es impar,
estaremos en una silla de número impar, ésta solo puede ser la número 3.

46. (b) Hay tres tamaños de triángulos: los que constan de solamente un triangulito que
son 12; los que están formados por cuatro triangulitos que son 6 y los que están
formados por nueve triangulitos, que son 2. En total hay 20 triángulos.

47. (d) El arreglo posible es


3 4 1 2 5
2 5 3 4 1
4 2 5 1 3
1 3 4 5 2
5 1 2 3 4

48. (a) Hay 25 alumnos en la clase. Como 6 alumnos sacaron 60 en matemáticas, entonces
hay 19 alumnos que no sacaron 60 en matemáticas y, por lo tanto, hay a lo mas 19
deportistas. Si le damos un punto a cada uno de los deportes practicado por algún
98

alumno tenemos 17+13+8 = 38 puntos para los deportes y hay a lo más 19 personas
que practican deportes y ninguna de ellas practica tres deportes, entonces concluimos
que todas practican dos deportes. De los 19 deportistas, 17 son ciclistas; luego, hay
únicamente dos alumnos que son a la vez nadadores y futbolistas.

49. (c) Observe la figura:

R
A B

P
N Q

D C
M

Los siguientes son paralelogramos pues tienen sus lados opuestos paralelos: ARPN,
RBQP, PQC M, NPMD. Por lo tanto (RPN) = 21 (ARPN); (RQP) = 12 (RBQP);
(PQM) = 12 (PQC M); (NPM) = 12 (NPMD). Tenemos ası́ que

(RQMN) = 12 (ARPN) + 21 (RBQP) + 12 (PQC M) + 21 (NPMD)


= 12 (ABC D) = 1
2 · 40 = 20 cm2 .

50. (d) Para pisar la sexta grada, podemos escoger entre:


• 6 pasos de 1
• 2 pasos de 1 y dos de 2
• 4 pasos de 1 y uno de 2
• tres pasos de 2
Sin embargo, el orden de los pasos dados también es relevante. Las posibilidades
además son:
• Con 6 pasos de 1 solo existe un orden posible.
• Con 4 pasos de 1 y uno de dos, hay 5: 2 + 1 + 1 + 1 + 1; 1 + 2 + 1 + 1 + 1;
1 + 1 + 2 + 1 + 1; 1 + 1 + 1 + 2 + 1; 1 + 1 + 1 + 1 + 2.
• Con 2 pasos de 1 y dos de 2 hay 6: 2 + 2 + 1 + 1; 2 + 1 + 2 + 1; 2 + 1 + 1 + 2;
1 + 2 + 2 + 1; 1 + 2 + 1 + 2; 1 + 1 + 2 + 2.
• Con tres pasos de dos solo hay un modo.
En total hay 13 posibilidades. Para contar cuántas maneras de ir de la sexta a la
décima, procedemos de modo análogo; que nos da
• Con 4 pasos de 1 solo hay un modo
• Con 2 pasos de 1 y uno de 2 hay tres modos: 1 + 1 + 2; 1 + 2 + 1; 2 + 1 + 1
99

• Con dos pasos de 2 solo hay un modo.


Ası́, existen 5 posibilidades de la sexta a la décima grada. En total hay 13 · 5 = 65
maneras puesto que para cada una de las 13 posibilidades hay 5 maneras.

51. (c) y = a + bx , a − b = 1, a − 51 b = 5. Por lo tanto b = 5y a = 6, es decir a + b = 11.

52. (c) Tenemos que 4EF G está dentro del paralelogramo ABF G y 4F GH está contenido
en el paralelogramo DC F G, por lo tanto

(F EGH) = (EF G) + (HF G) = 21 (ABF G) + 12 (F C DG) = 12 (ABC D).

Luego, el área que buscamos es 5 cm2 .

DESARROLLO
1. Hay 4 que están en los tres clubes; como al menos 7 están en jardinerı́a y teatro,
entonces 3 están solamente en jardinerı́a y teatro, como al menos 4 están en jardinerı́a
y pintura, entonces no hay ninguno que esté solamente en esas dos; del mismo modo
hay 15 que están solamente en teatro y pintura. Ahora, 7 estudiantes están al menos
en jardinerı́a, pero de ellos hay 4 en los tres clubes y 3 también en teatro, entonces
ninguno está solo en jardinerı́a. También, 27 están al menos en teatro, pero de ellos
15 están también en pintura, 3 están también en jardinerı́a y 4 en los tres clubes, de
modo que hay 5 que están solo en teatro. De los 40 que están al menos en pintura hay
15 también en teatro y 4 en los tres clubes, de modo que hay 21 que están solamente
en pintura.
Finalmente, de los 93 estudiantes, 26 están en solo un club, 18 están en dos clubes y
4 en los tres clubes. Concluimos que los que no están en ningún club son

93 − 26 − 18 − 4 = 45.

2. Si x es la cantidad original, al primer hijo le dio 1000 + 101 (x − 1000), al segundo



2000 + 101 x − 3000 − 101 (x − 1000) , al igualar estas cantidades y resolver tenemos
x = 81000 y entonces reparte a cada uno 9000 colones. Tiene 9 hijos.

3. Observamos que todos los números enteros positivos pueden escribirse como 3k ó 3k +
1 ó 3k + 2 (por la división entre 3).
Consideremos primero el caso n = 3k. Tenemos N = 9k 2 + 1; como 9k 2 es divisible
por 3 y 1 no lo es, entonces N = 9k 2 + 1 no es divisible por 3.
Caso n = 3k + 1. Aquı́, N = 9k 2 + 6k + 2, el mismo argumento de antes prueba que
N no es divisible por 3.
Caso n = 3k + 2 ⇒ N = 9k 2 + 12k + 5, que, por las mismas razones no es divisible
por 3.
100

4. Observemos que para cualquier operación de adición o supresión de segmentos la


cantidad de letras U y letras A en ese segmento es la misma. Esto significa que la
diferencia entre la cantidad de letras A y la cantidad de letras U en una palabra
no varı́a, es decir, es constante. La palabra UAA tiene una U y dos A, la diferencia
del número de U’s menos el de A’s es −1; la palabra AUU tiene dos U y una A, la
diferencia del número de U’s menos el de A’s es 1, de manera que de la palabra UAA no
se puede obtener AUU con las operaciones permitidas y por eso no se puede afirmar
que estas palabras tienen necesariamente el mismo significado.

5. Como n es par, podemos escribir n = 2k con k entero. Ası́, N = 8k 3 + 40k =


8k k 2 + 5 , de modo que N es divisible por 8. Ahora:
   
k k 2 + 5 = k k 2 − 1 + 6 = (k − 1) k (k + 1) + 6k (1)

Como (k − 1) k (k + 1) es el producto de tres números enteros consecutivos, entonces


al menos uno es divisible por 2 y otro por 3, entonces (k − 1) k (k + 1) es divisible
 (k − 1) k (k + 1) + 6k es divisible por 6 y, entonces,
por 6, de manera que  por (1), se
tiene que k k 2 + 5 es divisible por 6 y en conclusión N = 8k k 2 + 5 es divisible
por 48.

6. Dado que
x y z
= 0 = 0,
x0 y z
entonces
xy0 = yx 0 , xz 0 = zx 0 , zy0 = yz 0 . (1)

Ahora
√ p √ 2
xx 0 + yy0 + zz 0
p √ p
= xx 0 + yy0 + zz 0 + 2 xx 0 yy0 + 2 xx 0 zz 0 + 2 yy0 zz 0 (2)

Por (1): xx 0 yy0 = (xy0 )2 , xx 0 zz 0 = (xz 0 )2 y yy0 zz 0 = (yz 0 )2 . Entonces la expresión (2)
se convierte en
xx 0 + yy0 + zz 0 + 2xy0 + 2xz 0 + 2yz 0 . (3)
Usando otra vez (1) tenemos que 2xy0 = xy0 +x 0 y, 2xz 0 = xz 0 +zx 0 y 2yz 0 = zy0 +yz 0 .
Por lo tanto (3) se convierte en

xx 0 + yy0 + zz 0 + xy0 + yx 0 + xz 0 + zx 0 + yz 0 + zy0 . (4)

La factorización de esta última expresión es



(x + y + z) x 0 + y0 + z 0
101

Es decir
√ p √ 2 
xx 0 + yy0 + zz 0 = (x + y + z) x 0 + y0 + z 0

y por lo tanto, obteniendo raı́z cuadrada:


√ p √ p
xx 0 + yy0 + zz 0 = (x + y + z) (x 0 + y0 + z 0 )

7. De las igualdades dadas se deduce que T = 49 A, A ≤ 9 y T es un número entero


positivo, por lo que A = 9, ası́, M = 6, T = 4, I = 3. Ahora la suma es

9 6 L P 4 +
9 O 6 9
L D
6 E 4
C O 6 3 3 4

En la primera columna de la derecha, al sumar se tiene que 17 + D debe terminar en


4 y como D es un dı́gito, se sigue que D = 7. Además 9 + . . . = C O; de aquı́ y del
hecho de que de la penúltima columna necesariamente debemos llevar 1, se tiene que
C = 1 y O = 0. Quedan los dı́gitos 8, 2, 5 para las letras P, L, E. Se tiene entonces
que P + L + E = 15, pero en la segunda columna, partiendo de la derecha, se debe
dar que P + 6 + L + E + 2 = 15 + 6 + 2 = 23 y, continuando con la suma, L + 6 + 2
debe terminar en 3, de donde L = 5. Como E < P, entonces E = 2, P = 8. Ası́, los
números dados representan las palabras: OLIMPIADA y MATEMATCA.

8. Sea ab + dc = n ∈ N. Entonces ad + bc = nbd ⇒ bc = d(nb − a); como dc es


irreducible entonces c y d son primos relativos y entonces b es divisible por d. De
ad + bc = nbd se tiene da = b(nd − c) y como a y b son primos relativos, entonces
d es divisible por b. Como b y d son positivos se tiene que son iguales (pues b divide
a d y d divide a b).

9. Digamos que un salto hacia adelante es positivo y un salto hacia atrás es negativo.
Lo que el problema indica es ver si es posible asignar el signo + o − a cada uno de
los números 1, 2, . . ., 1985 de modo que la suma de todos esos números (con signo)
sea igual a 0 (el grillo estarı́a otra vez en su posición original). Tenemos que

1985 · 1986
1 + 2 + · · · + 1985 = = 1971105,
2
que es un número impar. Si a alguno de sus términos le cambiamos el signo entonces
estarı́amos restando un número par y, por lo tanto, el nuevo resultado también es
impar, nunca podrı́a ser 0 porque 0 es par. El grillo no puede regresar a su posición
en 1985 saltos realizados de la manera indicada.
102

10. Si el número ası́ formado tiene 90 cifras entonces doce de ellas son iguales a y 78
son iguales a 2. Como un número es divisible por 9 si y solo si la suma de sus cifras
es divisible por nueve. Debemos tener entonces que

2 · 78 + 12a = 9k

o, de modo equivalente, 52 + 4a = 3k. Esto es, 3 tiene que dividir a 4a + 52. De aquı́,
a no puede ser múltiplo de 3, probamos con los otros dı́gitos y vemos que los que
sirven son a = 2, a = 5 y a = 8.

11. Supongamos que se dividen los 10 niños en tres equipos. Uno de los equipos debe
tener al menos 4 integrantes. Digamos que Diego es uno de los niños en el equipo de
por lo menos 4 integrantes. Diego tiene 7 amigos, pero entre los otros equipos hay a
lo más 6 niños; por lo tanto, Diego tiene un amigo en su equipo.

12. Suponga que coloreamos el tablero con casillas blancas y negras como se ve en la
siguiente figura:

Observamos que con la jugada del camello, la ficha se desplaza siempre de una casilla
a otra del mismo color en la que estaba. Como dos casillas vecinas tienen diferente
color, entonces no es posible desplazar la ficha de una casilla a alguna vecina usando
solamente la jugada del camello.

13. Notemos que DEAF es un rectángulo, por lo que EF = AD. El punto D para el cual
AD es mı́nimo es el pie de la altura trazada desde A.

C
D
E

A B
F

14. Designemos las personas por p1 , p2 , ..., p10 y construyamos un tablero (10x10) de la
siguiente forma:
103

? Escribimos un 1 en la posición (i, j) si pi conoce a pj


? En las posiciones (i, i) colocamos un *.
El enunciado asegura que en el tablero habrá exactamente 2 · 19 = 38 unos. Si todas
las personas conocieran al menos a 4 personas, entonces habrı́a al menos 4 · 10 = 40
unos, lo cual no puede ser. Entonces al menos una persona conoce como máximo a
tres personas.

15. Como todos los números de cada fila son diferentes entre sı́, y todos los números de
cada columna son diferentes entre sı́, entonces cada uno de los números está escrito
25 veces. Ası́, hay 25 unos, como están escritos en forma simétrica con respecto a la
diagonal principal y 25 es impar, entonces, por lo menos un 1 está en la diagonal
principal; del mismo modo, habrá al menos un dos, un tres, ..., un veinticinco. Pero la
diagonal principal tiene 25 casillas, entonces hay exactamente uno de cada uno de
los 25 números escritos en la diagonal principal y, entonces, todos son diferentes.

16. Como son consecutivos podemos escribir estos números como: c−2, c−1, c, c+1, c+2
y entonces

a + b + c + d + e = m3 ⇒ 5c = m3 (1)
b + c + d = n2 ⇒ 3c = n2 (2)

De (1) y (2) se deduce que c tiene que ser múltiplo de 3 y de 5. Siendo c mı́nimo, de
los únicos números primos que debe ser múltiplo son 3 y 5, entonces c es de la forma

c = 3r · 5s .

Por (1), r = 3p, s = 3q − 1 y por (2), r = 2u − 1, s = 2v. Los menores enteros


positivos que cumplen estas dos condiciones son r = 3 y s = 2. Por lo que

c = 33 · 52 = 675.

17. Numeremos los escalones del 1 al 10. Luego, la suma S de los números asignados a
los escalones es
S = 1 + 2 + · · · + 10 = 55.

Si una rana sube k peldaños, otra baja k peldaños. Esto significa que la suma de
los números asignados a los peldaños en donde se encuentran las ranas permanece
constante en 55 pues 55 + k − k = 55. Por ejemplo, si inicialmente la rana que estaba
en el peldaño 7 baja al 4 y la del 6 sube al 9, la suma S se calcuları́a ası́:

S = 1 + 2 + 3 + 4 + 4 + 5 + 8 + 9 + 9 + 10 = 55.

Lo anterior significa que las 10 ranas no pueden estar en el mismo peldaño, pues de
estar todas en el peldaño p tendrı́amos que S = 55 = 10p, que no tiene solución en
los enteros.
104

18. Hay 14 cifras 0. Entonces, la cantidad de cifras distintas de 0 es 1994 − 14 = 1980.


Sea k la cantidad de cifras 1 en N; entonces la cantidad de cifras 2 es 2k, la de
cifras 3 es 3k, ..., la cantidad de cifras 9 es 9k. Como N tiene 1980 cifras distintas de
0 entonces 1980 = k + 2k + · · · + 9k = 45k y, por lo tanto, k = 44. Si S es la suma
de las cifras de N, entonces

S = 14 · 0 + 1 · 44 · 1 + 2 · 44 · 2 + 3 · 44 · 3 + · · · + 9 · 44 · 9
 
= 44 12 + 22 + · · · + 92 = 12 540.

Vemos que la suma de las cifras de N es 12 540 que es múltiplo de 3, pero que no
es múltiplo de 9. Ası́, si N fuese igual a k 2 , siendo N múltiplo de 3, k tendrı́a que
ser múltiplo de 3 y en ese caso k 2 = N serı́a múltiplo de 9, cosa que no sucede.
Concluimos que N no es cuadrado perfecto.

19. En cada centena hay que escribir los números que terminan en 8 que son 10, más
los que empiezan en 8 que son otros 10. Ası́, son 20 en cada centena, excepto en la
centena de los ochocientos de modo que tenemos 9 · 20 = 180 de estas centenas,
más 120 ochos por la centena de los ochocientos que dan un total de 300 ochos en
el primer millar. Como hay dos millares, se tienen 600 de estos. Además 20 en la
siguiente centena y uno más en la última página. Ası́, se reescribieron 621 ochos en
las 2108 páginas.

20. El menor número terminado en 88 y cuyas cifras suman 88 es 9999999988. Pero este
número no es divisible por 8 (por lo tanto no lo es por 88). En el lugar de las centenas
debe haber un número par, pero en tal caso, para conservar la suma de los dı́gitos,
el número tiene que tener por lo menos 11 dı́gitos. El más pequeño de los números
que satisfacen las dos propiedades (suma de los dı́gitos igual a 88 y múltiplo de 8) es
19999999888, pues el dı́gito de la izquierda es 1. Este número es múltiplo de 11 pues
1 + 9 + 9 + 9 + 8 + 8 = 44 y 9 + 9 + 9 + 9 + 8 = 44 y 44 − 44 = 0. Entonces el
número es múltiplo de 8 · 11 = 88. Ası́ que, el número buscado es 19999999888.

21. El área del rompecabezas es

1 + 16 + 49 + 64 + 81 + 100 + 196 + 225 + 324 = 1056 = 25 · 3 · 11

que también es el área del rectángulo. Para ubicar el cuadrado de lado 18, los lados
del rectángulo deben medir al menos 18.
Las dimensiones posibles son 48 × 22, 32 × 33 o 24 × 44. Si tiene un lado de 24, al
poner el cuadrado de 18 quedan 6 y no se puede seguir. Si se tiene un lado de 22,
después de poner el cuadrado de 18 quedan 4, poniendo el cuadrado de 4, abajo de
éste queda un hueco que no se puede llenar. De 32 × 33, sı́ se puede armar, una
solución la da la siguiente figura.
105

10 × 10 9×9
14 × 14

4×4 7×7
8×8

18 × 18
15 × 15

22. Como 36 = 729 y q ≥ 11, si p ≥ 3 se tiene que p6 q ≥ 729 · 11 = 8019 > 2300.
Las relaciones anteriores nos dicen que p debe ser igual a 2. Ahora, 26 = 64 y
26 q = 64q < 2300, esto es, q ≤ 35. El mayor primo menor que 35 es 31, se llega a la
solución n = 26 · 31 = 1984.

23. Nombremos los vértices de la manera como lo muestra la figura:

B1 B2 B3 B4 B5 B6 B7 B8 B9 B10

A = A1 A2 A3 A4 A5 A6 A7 A8 A9 A10 = B

Cada Ai representa el número de caminos que hay para ir desde A = A1 hasta el


vértice denotado con Ai , del mismo modo, Bi representa el número de caminos que hay
para ir desde A = A1 hasta el vértice denotado con Bi . Por ejemplo, A1 = 1, B1 = 1,
B2 = 2, A2 = 3. Observe que para llegar a Bi+1 solo se puede hacer desde Ai o desde
Bi y para llegara a Ai+1 antes debe estar en Ai o en Bi+1 . Luego,

Bi+1 = Ai + Bi
Ai+1 = Ai + Bi+1 = 2Ai + Bi

Con la ayuda de estas relaciones y, puesto que A1 = 1 y B1 = 1, podemos generar la


siguiente tabla:
i 1 2 3 4 5
Bi 1 2 5 13 34
Ai 1 3 8 21 55
i 6 7 8 9 10
Bi 89 233 610 1 597 4 181
Ai 144 377 987 2 584 6 765

Como A10 = B, concluimos que hay 6 765 caminos para ir desde A hasta B.
106

24. Tenemos que a2 + b2 = 25 ab ⇒ a2 + 2ab + b2 = 25 ab + 2ab ⇒ (a + b)2 = 92 ab.


Procediendo análogamente, (a − b)2 = 12 ab. Entonces

9
(a + b)2 2 ab
a + b
= 1 = 9 ⇒ =3
(a − b)2 2 ab
a − b

y por lo tanto a+b


a−b puede tomar los valores 3 y −3.

25. Separemos los segmentos en dos clases:


Clase A: los segmentos que unen extremos de igual color.
Clase B: los segmentos que unen extremos de diferente color.
Si P es un punto especial, consideremos dos conjuntos de segmentos:
Ap : segmentos de A que tienen un extremo en P.
Bp : segmentos de B que tienen un extremo en P.
Supongamos que Ap tiene a elementos y que Bp tiene b elementos, tenemos, por las
condiciones del problema, que b > a.
Al cambiar el color de P, los segmentos de Ap tendrán extremos de distinto color y
los de Bp los tendrán de igual color. Observemos cuidadosamente la nueva situación:
la cantidad de segmentos de clase B aumentó en a y disminuyó en b, o sea que es
menor que antes de cambiar el color de P. En conclusión, en un número finito de
pasos no quedarán segmentos de clase B y por lo tanto no habrá puntos especiales.

26. Sean y a, b, c los lados. Entonces a2 = 505, b2 = 233, c2 = 52. De acuerdo con la
fórmula de Herón el área del triángulo es
1
p
4 (a + b + c)(b + c − a)(a + b − c)(a − b + c) =
q
1
4 (b2 +c2 −a2 +2bc)(a2 −b2 −c2 +2bc) =
p p √
1
4 (2bc + 220)(2bc − 220) = 14 4b2 c2 − 2202 = 41 64 = 2.

27. Sea x el número de hija y y el número de hijos, cada hija tiene el mismo número de
hermanas que de hermanos, es decir,

x − 1 = y;

cada hijo tiene el doble de hermanas que de hermanos, es decir

2(y − 1) = x.

De aquı́: 2(y − 1) = y + 1 ⇒ y = 3 y por lo tanto x = 4. En la familia hay tres hijos


y cuatro hijas.
107

28. Llamemos con a1 , a2 , b1 , b2 , c1 , c2 , a las afirmaciones de los diarios A, B, C, respecti-


vamente, en el orden en que fueron enunciadas. Sabemos que una de las afirmaciones
de A es falsa (F) y la otra verdadera (V).
i) Si a1 es F entonces X ganó y a2 es V. Dado que X ganó, b2 es V y por lo tanto b1
debe ser F. Entonces el orden de llegada fue X , Y , Z . Con este orden, c1 es V y c2 es
F.
ii) Si a2 es F entonces Y ganó y a1 es V. Dado que Y ganó, b1 es F y por lo tanto
b2 debe ser V. Entonces el orden de llegada fue Y , X , Z . Con este orden, c1 es V y
c2 es F.
El orden de llegada pudo haber sido X , Y , Z o bien, Y , X , Z ; ambos son consistentes
con las afirmaciones de los tres diarios. Solo puede afirmarse que Z llegó último.

29. Consideremos la figura:

D
C

h
P
Q
A B

Observando los triángulos ADC y PDC concluimos que DP = 12 AD y ambos triángulos


tienen la misma altura desde el vértice C ; por lo tanto (PDC ) = 12 (ADC ) . Del mismo
modo (PAB) = 12 (DAB). Por otra parte, tomando DC = 6 como base del 4ADC y
como altura h = 4, se tiene (ADC ) = 21 DC · h = 12. Luego, (PDC ) = 6.
Utilizando base AB = 10, altura h, tenemos (DAB) = 20, de donde (PAB) = 10.
Entonces tenemos
4 (10 + 6)
(C PB) = (ABC D) − (PAB) − (PC D) = − 10 − 6 = 16.
2

Los triángulos C PQ y C QB tienen igual área pues PQ = QB. En consecuencia se


cumple que (C PQ) = (C QB) y

1
(C PQ) = (C PB) = 8.
2

30. Observe que si n es par, con n = 2k, entonces

Sn = 1 − 2 + 3 − 4 + 5 + · · · − 2k
= −1 + −1 + · · · + −1 (k veces − 1) = −k
108

Si n es impar, n = 2k + 1, entonces

Sn = 1 − 2 + 3 − 4 + 5 + · · · − 2k + 2k + 1
= −1 + −1 + · · · + −1 + 2k + 1
= −k + 2k + 1 = k + 1.

Ası́, S57 = S2·28+1 = 28 + 1 = 29, S69 = S2·34+1 = 34 + 1 = 35, S60 = S2·30 = −30.
Entonces
S57 + S69 − S60 = 29 + 35 − (−30) = 94.
←→
31. Sean E y F los puntos de intersección de la recta QN con MP y BP.

M
A B

E
Q N
F

D C
P

Por la condición del enunciado EF 2·BN = 1. Entonces EF · BN = 2. Como M, N, P, Q


son los puntos medios de los lados se sigue que EP = NC = BN. Además BC ||MP.
Entonces BNPE es un paralelogramo (tiene los lados opuestos paralelos y congruen-
tes). En consecuencia, F es el punto medio de EN, ya que las diagonales de un
paralelogramo se bisecan. Por lo tanto AB = 2MB = 2EN = 2(2EF ) = 4EF . Por
otro lado BC = 2BN. Entonces (ABC D) = (AB)(BC ), es decir

(ABC D) = (4EF )(2BN) = 8(EF )(BN) = 8 · 2 = 16.

32. Por el teorema de Pitágoras se tiene que (x + 1)2 − b2 = (x − 1)2 − a2 . Luego,


b2 − a2 = (x + 1)2 − (x − 1)2 ⇒ (b − a)(b + a) = 4x. Como b + a = x, entonces, de
la última igualdad se sigue que (b − a)x = 4x y, puesto que x no es nula, se tiene
que b − a = 4.

33. El primer jugador tiene estrategia ganadora: sólo necesita en cada turno dejar el
mismo número de fichas con el lado rojo hacia arriba que de fichas con el lado negro
hacia arriba, quitando las que haya demás en uno u otro color. Como 1999 es impar
no puede haber al principio el mismo número de fichas con un lado hacia arriba que
con el otro, por lo que el primer jugador puede seguir su estrategia en el primer turno.
Ahora, si al principio de un turno del segundo jugador hay la misma cantidad de
fichas con cada color hacia arriba, este no podrá evitar dejar más fichas mostrando un
color que el otro. Ası́, el primer jugador siempre podrá seguir su estrategia, siempre
109

que la haya seguido hasta el turno anterior. Como el primer jugador siempre quita
al menos una ficha, el juego termina. Como después de cada uno de sus turnos el
segundo jugador deja diferentes cantidades de fichas con cada lado hacia arriba, el
primer jugador gana.

34. Tenemos
a + b−1 a + b1 a
= 1
= .
a−1 + b a +b
b

Debemos encontrar los a y b tales que

a
= 13 y a + b ≤ 80.
b

Como a
b = 13, entonces a = 13b y por lo tanto

a + b = 14b ≤ 80.

De manera que b es a lo sumo 5. Las parejas (a, b) que cumplen son (13, 1), (26, 2),
(39, 3), (52, 4) y (65, 5).



35. Trace una lı́nea HI paralela a EF por D. m∠GDI = 60◦ y m∠C DI = 45◦ , luego
m∠GDC = m∠GDI − m∠C DI = 60◦ − 45◦ = 15◦ . Por lo tanto m∠DGC = 75◦ .

E F
H D I

A C
G

36. Sea p el número de fósforos que necesitan ser retirados. Por cada fósforo que se retire
desaparecen, a lo más, dos cuadrados de 1 × 1. Luego, para desaparecer todos los
cuadrados de 1×1 hay que retirar al menos 8 fósforos; ası́, p ≥ 8. Pero si conseguimos
desaparecer todos los cuadrados de 1 × 1 retirando 8 fósforos, entonces ninguno de
esos fósforos pertenecı́a a la frontera del cuadrado grande, de modo que ası́ aún no
ha desaparecido el cuadrado de 4 × 4; necesariamente, entonces, p ≥ 9. El siguiente
dibujo es una solución con p = 9 (este es el mı́nimo):
110

37. Como m∠AF D = m∠BDF + m∠F BD, tenemos que b + 60◦ = a + m∠B. También, de
m∠BDE = m∠DEC + m∠DC E obtenemos que a + 60◦ = c + m∠C . Restando las
dos ecuaciones y recordando que m∠B = m∠C resulta b − a = a − c. Al despejar
a se obtiene a = b+c
2 .

b
F 60◦ E
c

60◦
a
B C
D

38. Si los dı́gitos de las unidades de los números de la tercera fila son todos diferentes,
entonces al sumar todos estos números, se obtiene un número cuya cifra de unidades
es 5, puesto que 0 + 1 + 2 + 3 + 4 + 5 + 6 + 7 + 8 + 9 = 45. Pero esa suma tiene que ser
igual a las suma de todos los números ubicados en las casillas de las filas primera
y segunda, como esos son los números del 1 al 10, dos veces, entonces esa suma es
90, cuya cifra de unidades es 0. Esa cifra no puede ser 5 y 0 a la vez, entonces no se
pueden escribir los números de la manera indicada en el enunciado.

39. Cuando sumamos un número de dos cifras y el número escrito con las mismas cifras
en orden inverso, la suma es un número de tres o de dos dı́gitos, en este último caso
los dı́gitos son iguales. Como ningún cuadrado perfecto menor que 100 tiene dos cifras
iguales, debemos buscar sumas que sean mayores que 100. En este tipo de sumas, si
el dı́gito de las unidades es t, entonces el dı́gito de las decenas es t + 1, por lo que
111

el único cuadrado al que podemos llegar es a 121; y las soluciones son 29, 38, 47, 56,
65, 74, 83 y 92.

40. Supongamos que no hay puntos marcados a distancia 21 de la orilla de la cuadrı́cula.


Entonces todos los puntos marcados están en la cuadrı́cula central de 6 × 6. Dividimos
esta cuadrı́cula en nueve cuadrados de 2×2. Al haber 10 puntos marcados, debe haber
2 en √
un mismo cuadrado de 2 × 2. La distancia entre esos dos puntos es menor o igual
que 2.

a 2
41. En la figura, sea a el lado del cuadrado; entonces F B = 2 (media diagonal). √La
AE·a 2
base del triángulo AEB es AE y su altura es F B, entonces

su área es AE·F B
2 = 4 ,
AE·a 2
como es igual al área del cuadrado, se tiene 4 = a2 , es decir
4a
√ √ √
2

5 2
AE = √ 2
= 2a 2 ⇒ AF = 2a 2 + a 2 = 2 a.
Por el teorema de Pitágoras en 4AF B, se tiene
 √ 2  √ 2
130 = 5 2 2 a + 22 a ⇒ 130 = 252 a2 + 21 a2 ⇒ a2 = 10.
El área del cuadrado es 10.

E
A F C

42. Sean n, n + 1, n + 2, n + 3 los números; su suma es 4n + 6. Supongamos que esta


suma es un cuadrado perfecto: 4n + 6 = p2 . Entonces 2(2n + 3) = p2 , por lo tanto p
tiene que ser par, digamos p = 2q. Tenemos 2(2n + 3) = 4q2 ⇒ 2n + 3 = 2q2 ; pero
2n + 3 es impar y 2q2 es par, entonces no es posible la igualdad. La suma no puede
ser un cuadrado perfecto.

43. abcd + dcba = 6435 ⇒

6435 = (1000a + 100b + 10c + d) + (1000d + 100c + 10b + a)


6435 = 1001(a + d) + 110(b + c)
585 = 91(a + d) + 10(b + c)
91(a + d) = 585 − 10(b + c)
91(a + d) = 5[117 − 2(b + c)]

⇒ 5 divide a a + d. Como a y d son dı́gitos, con a 6= 0, de lo anterior tenemos que


a + d = 5 ó 10 ó 15; pero no puede ser 10 ni 15 porque entonces 91(a + d) > 585 lo
112

cual no puede ser, entonces a + d = 5. En esta situación, 91(a + d) + 10(b + c) = 585


⇒ 10(b+c) = 585−455 = 130 ⇒ b+c = 13. Hay cinco parejas de dı́gitos (a, d) (con
a 6= 0) tales que a + b = 5 y hay seis parejas de dı́gitos (b, c) tales que b + c = 13,
entonces hay 5 · 6 = 30 números con la propiedad pedida.

44. Los números pueden ser escritos como x − 2, x, x + 2. La suma de los cuadrados será

(x − 2)2 + x 2 + (x + 2)2 = aaaa,

donde a es un dı́gito diferente de 0. Desarrollando se tiene

3x 2 + 8 = 1000a + 100a + 10a + a ⇒


3x 2 = 1000a + 100a + 10a + a − 8

Luego 1000a + 100a + 10a + a − 8 es divisible por 3. Esto dice que a no puede ser
múltiplo de 3, pues 8 no lo es.
Para a = 8 tendrı́amos 8888 − 8 = 8880 ⇒ 3x 2 = 8880 ⇒ x 2 = 8880 ÷ 3 = 2960 y
en este caso x no serı́a entero.
Para a = 7 tendrı́amos 7777 − 8 = 7769. Pero 7769 no es divisible por 3.
Para√a = 5 tenemos 5555 − 8 = 5547 ⇒ 3x 2 = 5547 ⇒ x 2 = 5547 ÷ 3 = 1849 ⇒
x = 1849 = 43.
Ası́, los números 41, 43, 45 satisfacen las condiciones pedidas (412 + 432 + 452 = 5555).

45. P = x 3 = 2y2 , entonces 2|x ⇒ x = 2k ⇒ 8k 3 = 2y2 ⇒ 4|y2 ⇒ 2|y ⇒ y = 2l


⇒ 4k 3 = 4l2 ⇒ k 3 = l2 , entonces, k es un cuadrado y l es un cubo, i.e. k = r 2 ⇒
x = 2r 2 ⇒ P = 8r 6 . Para r = 1, 2, 3, 4, 5, se obtienen P = 8, 64, 729, 4096, 15625 y
estos son los números buscados.

46. Sea n = 1a1b1c1d1. Para que n sea múltiplo de 33, debe ser múltiplo de 3 y de 11.
Para que sea múltiplo de 3, la suma de sus dı́gitos debe ser divisible por 3, es decir
1 + a + 1 + b + 1 + c + 1 + d + 1 = 5 + (a + b + c + d) tiene que ser múltiplo de 3.
Para que n sea divisible por 11 entonces la suma alternada de sus dı́gitos tiene que
ser divisible por 11, es decir, 1 − a + 1 − b + 1 − c + 1 − d + 1 = 5 − (a + b + c + d)
tiene que ser divisible por 11. Ası́, a + b + c + d tiene que ser de la forma 3k + 1
para que al sumarle 5 sea múltiplo de 3; también tiene que ser de la forma 11k + 5.
Consideremos M = (a + b + c + d) − 16. Como a + b + c + d es de la forma 3k + 1,
entonces M es divisible entre 3 y como a + b + c + d es de la forma 11k + 5, entonces
M es divisible entre 11. Es decir (a + b + c + d) − 16 es divisible entre 33 y por lo
tanto a + b + c + d es de la forma 33k + 16. Como a, b, c, d, son dı́gitos, su suma
es a lo más 36 y si la forma de a + b + c + d es 33k + 16, entonces necesariamente
a + b + c + d = 16. Asi, la cantidad de números de la forma 1a1b1c1d1 divisibles
entre 33 es igual al número de forma de escoger cuatro dı́gitos a < b < c < d cuya
113

suma sea 16. Hay 16 formas de hacerlo:

a b c d a b c d
0 1 6 9 1 2 4 9
0 1 7 8 1 2 5 8
0 2 5 9 1 2 6 7
0 2 6 8 1 3 4 8
0 3 4 9 1 3 5 7
0 3 5 8 1 4 5 6
0 3 6 7 2 3 4 7
0 4 5 7 2 3 5 6

47. Tenemos
22 + 25 + 2j = 22 + 22 · 23 + 2j = 9 · 22 + 2j = r 2 .
Luego: 2j = r 2 − 9 · 22 = (r + 6) (r − 6). Ası́, r + 6 = 2n y r − 6 = 2m , donde n y m
son enteros positivos con m + n = j y, desde luego, n > m.
Como 2n − 2m = (r + 6) − (r − 6) = 12 = 22 · 3, se tiene que 2m (2n−m − 1) = 22 · 3;
por lo que m = 2 y n − m = 2. Estos es, n = 4, y por lo tanto el único valor posible
para j es j = 6.

48. De acuerdo con las condiciones del problema se tiene la siguiente figura.

A
F

B C
D

En 4EDB, isósceles, se tiene m]BED = m]EDB, m]BED + m]EDB = m]ABD


(prop. ángulo exterior).
Como m]ABD = 2 · m]AC B, resulta que
m]EDB = m]AC B.
También m]F DC = m]EBD (opuestos por el vértice).
Entonces 4DC F es isósceles y tenemos DF = F C . m]DAF = 90◦ − m]AC B,
m]ADF = 90◦ − m]AC B. De donde el 4AF D es isósceles y tenemos DF = AF .
1
Resumiendo AF = DF = F C y como AF + F C = AC , resulta que DF
AC = 2 .

49. Primero veamos que 111, 222, 333, . . ., 999 son equilibrados. Notemos también que si el
número de tres cifras abc es equilibrado entonces también lo son acb, bac, bca, cab
y cba, además si dos cifras de un número equilibrado son iguales, entonces la otra
114

debe ser igual a estas dos, por lo que los números equilibrados de cifras diferentes
aparecen de 6 en 6. Finalmente, debemos tener en cuenta que si abc es equilibrado,
con c = a+b
2 , entonces a y b deben tener la misma paridad (ambos son pares o ambos
impares a la vez). Construimos ahora los números equilibrados siguiendo un orden,
primero los que tienen primera cifra 1, luego con 2, etc.:

132, 153, 174, 195,


243, 264, 285,
354, 375, 396,
465, 486,
576, 597,
687,
798.

Por las observaciones anteriores, estos 16 número equilibrados generan a 16 · 6 = 96


números y con los 9 primeros tenemos en total 105 números equilibrados.

50. Los números n y n + 1 son enteros consecutivos, por lo que uno de ellos es par y,
entonces, n(n + 1) es divisible por 2. Ası́, n(n + 1)(2n + 1) y
n(n + 1)(4n + 5) son divisibles por 2.
Ahora, podemos escribir n en la forma n = 3p + k, donde p y k son enteros y k = 0,
o k = 1, o k = 2. Si k = 0, entonces n = 3p y entonces es divisible por 3 de modo
que los dos números:
n(n + 1)(2n + 1) y n(n + 1)(4n + 5)
son divisibles por 3 y por lo tanto por 6 (pues ya vimos que son divisibles por 2). Si
k = 2, entonces n + 1 = 3p + 3 y entonces es divisible por 3, resultando que los
números indicados son divisibles por 3 (y, por lo tanto, por 6). Finalmente, si k = 1,
entonces 2n + 1 = 6p + 3 es divisible por 3 y 4n + 1 = 12n + 9 también es divisible
por 3. Esto muestra que siempre los números indicados son divisibles por 6.

51. No es posible realizar tal distribución. La mayor diferencia entre los números escritos
en triangulitos vecinos es 2. Ası́, la mayor diferencia entre los números ubicados en
dos triangulitos, uno de los cuales es vecino del vecino del otro es igual a 4. Por
ejemplo, en el gráfico, la mayor diferencia entre a y b es 4.

b
a
115

Siguiendo ese razonamiento vemos que la mayor diferencia posible entre los números
escritos en dos triangulitos cualesquiera es 12 (considerando los triangulitos más
alejados uno del otro). Pero la diferencia entre 1 y 16 es 15 > 12. De modo que no es
posible ubicar todos los números con la condición pedida.

52. Consideremos la figura:

C1

B1

B C
A1

El triángulo A1 B1 C1 se divide en los triángulos AA1 B1 , AA1 C1 , AB1 C1 .


←→ ← →
Como BB1 kAA1 , (AA1 B1 ) = (AA1 B) (si pensamos que la base de ambos triángulos
es AA1 , tienen también la misma altura: la distancia de AA1 a BB1 ). Análogamente,
(AA1 C1 ) = (AA1 C ). Por lo tanto (AA1 B1 ) + (AA1 C1 ) = (AA1 B) + (AA1 C ). Veamos ahora
←→ ←→
el triángulo AB1 C1 : Como BB1 kCC1 , entonces (BB1 C1 ) = (BB1 C ). Restando (BB1 A) a
cada lado de esa igualdad se obtiene que (AB1 C1 ) = (ABC ). Por lo tanto,

(A1 B1 C1 ) = (AB1 C1 ) + [(AA1 B1 )] + (AA1 C1 ) = 2 (ABC ) = 3998.

53. Vemos que para todo entero a ≤ 4, se cumple que a ≤ (a−2)·2. Por tanto, ninguno de
los números elegidos será mayor o igual que 4. Ası́, solamente elegiremos los números
1, 2 ó 3. Pero como 2 · 2 · 2 < 3 · 3, 1 · 1 < 2, 1 · 3 < 2 · 2, 1 · 2 < 3, entonces, para
formar el 1999, el número 1 no puede aparecer y el número 2 no puede aparecer más
de dos veces, entonces la descomposición que cumple lo pedido es

1999 = 3 + 3 + · · · + 3 + 2 + 2.
z }| {
665 veces el 3

54. i) Supongamos que sı́ es posible tal acomodo y sean a, b y c los números colocados en
los vértices. Entonces las sumas en los lados, sin contar los extremos, son 27 − a − b;
27 − a − c; 27 − b − c, de donde

(a + b + c) + (27 − a − b) + (27 − a − c) + (27 − b − c) = 1 + 2 + · · · + 12 = 78.

Luego, 3 · 27 − (a + b + c) = 78 ⇒ a + b + c = 3, y esto no es posible, ya que a, b,


c, varı́an de 1 a 12 sin repetirse.
116

ii) En el caso de 28: 3·28−(a+b+c) = 78 ⇒ a+b+c = 6. Pongamos a = 1, b = 2,


c = 3 (que no es la solución única, pues pueden considerarse las permutaciones de
esos números). Entonces las sumas de los lados, sin considerar los extremos, deben
ser 25, 24, 23 y se pueden acomodar tal como se muestra en la siguiente figura:

1
12 7
4 6
9 11
2 10 5 8 3

NIVEL B
SELECCION
1. (c)Formando un triángulo rectángulo con uno de sus ángulos agudos en A, con catetos
5 y 12, entonces la hipotenusa es 13. De manera que sin A = 5/13, cos A = 12/13.
Concluimos que
sin A 5/13 1
= = .
1 + cos A 1 + 12/13 5

2. (a) De acuerdo con las condiciones dadas, existen enteros k, l, m tales que

N − 8 = 8k ⇒ N = 8(k + 1)
N + 9 = 9l ⇒ N = 9(l − 1)
N − 7 = 7m ⇒ N = 7(m + 1)

Es decir, N es divisible por 7, 8 y 9, por lo tanto N = 7 · 8 · 9 · M = 504M. Como N


es de tres cifras, entonces M = 1 y por lo tanto N = 504. El resto de dividir este
número entre 5 es igual a 4.

3. (b) Al sumar las unidades se obtiene 9, al sumar las decenas se obtiene una cantidad
entre 10 y 19, por lo que se lleva 1; ası́, al sumar las centenas se tiene también una
cantidad entre 10 y 18, se lleva otra vez 1. La suma es entonces de la forma

9876 + A32 + B1 = 10X Y 9,

donde X y Y son cifras, por lo tanto la suma tiene 5 cifras.

4. (c) Los términos de la progresión son de la forma 3n + 1. El mayor número de cuatro


dı́gitos que es de esa forma es 9997, que se escribe como 9997 = 3 · 3332 + 1.
117

5. (c) Puesto que los triángulos son equiláteros, m∠ADC = 120◦ ; entonces, por la ley de
los cosenos 2 2 2
√en el triángulo ADC se tiene AC = 2 +1 −2·1·2·cos 120 = 4+1+2 = 7

⇒ AC = 7.

6. (d) Sea x el número de personas del grupo. Si todos pagasen, entonces cada uno
pagarı́a 7200
x , como hay tres que no pagan, entonces cada uno de los restantes paga
7200
x−3 , pero esto es 400 más que si hubieran pagado todos, es decir

7200 7200
= + 400
x−3 x

Esto se convierte en x 2 − 3x − 54 = 0, cuyas soluciones son x = −6 o x = 9. Como


el número de personas es positivo concluimos que x = 9.

7. (c) Los números enteros a y b sin incluir a a ni a b son a+1, a+2, . . ., a+(b−a−1), es
decir, son b−a−1 números. Tomando a = 9992 y b = 10002 , tenemos 10002 −9992 −1 =
(1000 − 999) (1000 + 999) − 1 = 1999 − 1 = 1998 números.

8. (a) Como lkm entonces m∠C AB = 40◦ . Además, m∠C BA = 80◦ , ya que es suple-
mentario del ángulo que mide 100◦ . Entonces

m∠AC B = 180◦ − (m∠C AB + m∠C BA)


= 180◦ − (40◦ + 80◦ ) = 60◦ .

Como x es suplementario de ∠AC B, entonces x = 120◦ .

9. (a) Si ambos p y q son primos diferentes de 2, entonces ellos son impares y por
lo tanto su suma es par. Como 403 es impar, concluimos que p tiene que ser 2.
Ası́, 403 = 2 + 401 (ambos son primos) y esta es la única manera de satisfacer las
condiciones del problema.

10. (b) Puesto que AH ⊥ BC y MNQR es un cuadrado, entonces

MN k BC y 4AMN ∼ 4ABC .

De BC = 6, MN = a, AH = 4, AT = 4 − a (T punto de intersección de MN y AH),


entonces a6 = 4−a
4
⇒ a = 2, 4 m.

11. (b) Observemos que 74 = (72 )2 = (49)2 y 47 = (22 )7 = (27 )2 = (128)2 . Puesto que,
entre 49 y 128 hay 128 − 49 − 1 enteros positivos (es decir, 78) habrá 78 cuadrados
perfectos entre 74 y 47 .
19
12. (a) m]C + m]D =√ 180◦ ⇒ m]D = 180◦ − 120◦ = 60◦ . Luego x = 19 cos 60◦ = 2,

h = 19 sin 60◦ = 192 3 , y = tanh30◦ = 3h = 572 ⇒ AD = 192 + 572 + 10 = 48.
118

B 10
C
150◦ 120◦
h h

30◦ 60◦
A y 10 x D
M N

13. (a) El m.c.d. de las longitudes del trapezoide es 7. Por consiguiente, cada par de postes
consecutivos deben estar separados por una distancia de 7 metros. Como el perı́metro
es 665 m, al dividir 665 ÷ 7 = 95 se concluye que son 95 los postes necesarios.

14. (d) Tenemos que (2 · 23)2 = 2116 > 1999, mientras que (2 · 22)2 = 1936 < 1999. Para
todo los números de la forma (2 · n)2 con n = 1, ..., 22, podemos encontrar un número
impar positivo m tal que
(2 · n)2 + m = 1999.

15. (c) Consideremos el camino de A a B. Sea a el número de kilómetros cuesta arriba,


b el número de kilómetros cuesta abajo y c los restantes. En la subida se tardan 60
a

horas, en la bajada 90 horas y en el resto 72 horas. De este modo


b c

a b c
+ + =5 (*)
60 90 72

Al regreso, lo que antes era subida, ahora es bajada y viceversa, de modo que ahora
el tiempo total es
a b c
+ + =4 (**)
90 60 72
De (*) y (**) tenemos que

6a + 4b + 5c = 1800
4a + 6b + 5c = 1440

Sumando tenemos 10(a + b + c) = 3240 y por lo tanto la distancia de A a B es


a + b + c = 324 km.

16. (d) De acuerdo con la manera en que se forman los términos de la sucesión, los
primeros 6 términos son 1, 3, 2, −1, −3, −2. Luego los términos se van repitiendo de
esa misma forma de seis en seis. Ası́, al sumar los primeros 1998 términos se obtiene
0. El término 1999 es 1, el término 2000 es 3 y el término 2001 es 2, de modo que la
suma total es 6.

17. (a) Tenemos que 902 < 8xy9 < 972 . Ası́, el único cuadrado con esta condición, que
termina en 9 es 932 = 8649, por lo tanto x = 6, y = 4 y x + y = 10.
119

18. (b) Tenemos


r s
12n (4n + 1) n 22n 3n (4n + 1)
=
n

8n · 3n + 6n 23n · 3n + 2n 3n
s
22n 3n (4n + 1)
= n

2n · 3n (22n + 1)

= 2 = 2.
n n

19. (c) Al ser γ, x ángulos externos: x = γ + θ; γ = α + β. Ası́ x = α + β + θ.


4EBC ∼ = 4ABF por LAL, de donde ]AF B = θ y ]BEC = α. Luego en 4EBC :
α + 60 + β + θ = 180 y α + β + θ = 120◦ = x.

B
60 60
β
E F

O γ
60 α x θ 60
A C

20. (c) De acuerdo con la figura se tienen las igualdades: (1) qa = 1, (2) mq = 2, (3)
mn = 3, (4) np = 4, (5) pb = 16 y x = ab. Multiplicando (2) y (4): mnpq = 8 y, por
(3), se deduce que pq = 8/3. Multiplicando (1) y (5): (ab)(pq) = 16. Luego 38 x = 16 y,
entonces, x = 6.

1 q 2 q
p
a m
n 3 n 4
A a B m p

x b b 16

D
C

21. (b) Si escribimos p = x + y, q = x − y, el sistema se convierte en

p − 1 = aq + a
bp + b = −q − 1
120

Multiplicando la segunda ecuación por a y sumando a la primera se obtiene

p(1 + ab) + ab − 1 = 0

y por lo tanto
1 − ab
x+y=p= .
1 + ab

22. (b) Trácense las perpendiculares desde G y E a los lados BC y C D, como se muestra
en la figura.

E
A B
x H
x

90

G x J

D C
I F

Por postulado de semejanza A–A:

4EIF ∼ 4GJH

y como EI = GJ, por postulado A–L–A:

4EIF ∼
= 4GJH

y HG = EF = 10 cm. De donde HG = 10 cm.

23. (a) Digamos que cada persona comió una galleta de cada caja que comió. Si n es el
número de cajas y p es el número de personas, de la condición (i) se tiene que el
número de galletas comidas fue 3n; por la condición (ii) se tiene que el número de
galletas comidas fue 2p. Es decir, 2p = 3n y como 2 y 3 son primos relativos entonces
3 divide a p. Si fueran 3 personas, para que se cumplan (i) y (ii) a la vez, deberı́a
haber solo dos cajas, pero en este caso no podrı́a satisfacerse (iii). Si p = 6, si se
pueden cumplir las condiciones habiendo 4 cajas de galletas. Por ejemplo, si las cajas
son A, B, C y D y las personas son 1, 2, 3, 4, 5, 6. Las condiciones dadas se cumplen
si 1,2,3 comen de A; 1,4,5 comen de B; 2,4,6 comen de C y 3,5,6 comen de D. Ası́, el
mı́nimo es 6.
121

24. (c) Tenemos la siguiente secuencia de igualdades:


99 . . . 9 × 55 . . . 5 = (1095 − 1)(55 . . . 5)
= 1095 · 55 . . . 5 − 55 . . . 5
= 55 . . . 500 . . . 0 − 55 . . . 5
(95 cincos 95 ceros)−(95 cincos)
= 55 . . . 544 . . . 45
(94 cincos 95 cuatros 1 cinco)

La suma de las cifras es


94 · 5 + 95 · 4 + 5 = 855.
25. (d) Sea x el número de postales y y el precio de cada una, entonces xy = 1000.
Después de vender 8 le quedaban x − 8, perdió la cuarta parte de esto, es decir,
4 postales.
perdió, x−8 Ası́ que el número de postales que vendió es 3 · x−8
4 + 8, de modo
 3 3
que 3 · 4 + 8 y = 850 ⇒ 4 xy + 2y = 850 ⇒ 4 · 1000 + 2y = 850 ⇒ y = 50 ⇒
x−8

x = 20.
26. (d) Digamos que el número de dos cifras es ab, entonces 10a + b = 2a · b ⇒
b = 2a · b − 10a (*) ⇒ b es par. Entonces b es 0 ó 2 ó 4 ó 6 u 8. Sustituyendo estos
números sucesivamente en (*) tenemos: para b = 0, a = 0 (no sirve); para b = 2,
2 = −6a (no sirve); para b = 4, a = −2 (no sirve); para b = 6, a = 3 (si sirve); para
b = 8, 4 = 3a (no sirve). El número buscado es 36 y la suma de sus cifras es 9.
27. (a) Tenemos
 2
2 2 2
101999n +2 + 1 = 103998n +4 + 2 · 101999n +2 + 1
= 1 00 . . . }0
| {z + 2 00 . . . }0 + 1
| {z
3998n2 +4 ceros 1999n2 +2 ceros
= 100 . . . 0200 . . . 01
Ası́, el número tiene dos dı́gitos iguales a 1, un dı́gito igual a 2 y los demás iguales
a 0. La suma de ellos es 4.
28. (c) Puesto que las diagonales del rectángulo se cortan en su punto medio, entonces
(AOB) = (AOC ) = (C OD) = (BOD), entonces el área de cada uno de estos triángulos
es el 25 % del área del rectángulo.

P
A B

C D
Q
122

Además
4AOP ∼
= 4DOQ y 4BOP ∼
= 4C OQ,
por lo tanto, el área de la región sombreada es igual al área de 4AOB. Es decir, esta
área es el 25 % del área del rectángulo ABDC .
29. (b) El último dı́gito del resultado de aplicar la operación a un número solo depende
del último dı́gito del número. Por lo tanto, podemos hacer las cuentas considerando
únicamente la última cifra. Empezamos con 1, le sigue el 8, luego el 29. Nos quedamos
únicamente con el 9. Sigue 32, es decir 2, luego 11, es decir, 1. Entonces, a partir de
aquı́ todo se repite. Ası́, la sucesión de los últimos dı́gitos es 1, 8, 9, 2, 1, 8, 9, 2, 1, 8,
9, 2, . . . . Al repetir la operación un número de veces que sea múltiplo de 4, el último
dı́gito es 1. Como 1999 es uno menos que un múltiplo de 4, se obtiene 2 como último
dı́gito.
30. (c) La dos desigualdades son equivalentes a: 17 · 2 < 5n y 13n < 17 · 11, es decir, n
debe cumplir 6 < n y n < 15. Ası́ las n posibles son las que cumple 7 ≤ n ≤ 14.
31. (c) Llamemos a los números con a, b, c, d, e, f , g, como se muestra en la figura.

e f g

a b c

Sea x la suma de los tres números en cada lı́nea. La suma de los números que debemos
acomodar es 95, por lo tanto x = 95−d
2 . Observemos que x tiene que ser impar, ya que
es la suma de tres números impares y además x debe ser primo. Veamos los distintos
valores que puede tomar d. Si d es 5, 7, 11, 17, 19 ó 23, vemos que x es par o en los
casos en que es impar, no es primo. Por lo tanto, la única solución posible es cuando
d = 13, entonces x = 41 que es primo. Luego a + g = 28 = c + e y una posible
solución es a = 17, g = 11, c = 5 y e = 23, f = 7, b = 19.
32. (b) Los triángulos BC X y BT Y son semejantes pues ambos son rectángulos y tienen
en común el ángulo B. Luego
m∠BT Y = m∠BC X = m∠C = 180◦ − m∠A − m∠B
= 180◦ − 60◦ − 50◦ = 70◦ .

33. (b) Tenemos que


     
1 1 n+1 m−1
1+ 1− = = 1,
n m n m
entonces m = n + 1.
123

34. (c) Sumando las dos últimas ecuaciones se tiene que a2 = 13 , es decir, a = 6. Sustitu-
yendo este valor en las dos primeras ecuaciones tenemos que b1 + c1 = 56 y c1 − b1 = 61 .
Sumando estas dos tenemos c2 = 1, es decir, c = 2. Luego, de lo anterior encontramos
que b = 3 y por lo tanto
a + 2b + 3c = 18.

35. (b) La ecuación dada es equivalente a 5A + 7B = 31. De aquı́, como A y B son


naturales, se deduce que A < 7, pues si A ≥ 7, tendrı́amos que 5A + 7B > 31. Por
otra parte, 5A + 7B = 31 ⇒ 31 − 5A es múltiplo de 7, como 1 ≤ A ≤ 6, los únicos
valores posibles son A = 2, B = 3.
36. (d) Dividimos el triángulo X Y Z en la siguiente forma:

A F

B E

Y Z
C D

Cada triángulo pequeño tiene área 2. Llamemos S al área de la región sombreada:


S = 18 − [(DEZ ) + (BX F ) + (Y BD)] .
Pero (DEZ ) = 2 y (BX F ) = (Y BD) = 4. Por lo tanto S = 18 − 2 − 2 · 4 = 8.
37. (d) Puesto que 146 = 29 · 5 + 1, entonces m tiene que valer al menos 30. De lo
contrario, los números sumarı́an menos de 146.
38. (d) Sea DB = a, entonces AD = 2a; si AE = 2b, entonces EC = 3b. De modo que
(ADE) = 8 y por lo tanto
2a · 2b
(ADE) = sin A = 8,
2
4
luego sin A = . Concluimos que
ab
4
5b · 3a 15ab ·
(ABC ) = sin A = ab
= 30.
2 2

39. (b) Observe que b = (2k +√1)2 , con lo cual n3 + 2n2 = (2k + 1)√2 y ası́ expresamos
n2 (n + 2) = (2k + 1)2 ⇒ n n + 2 = 2k + 1. Necesitamos que n n + 2 ∈ N, con lo
que vemos que n = 2 no sirve ya que 4 = 2k + 1 no tiene solución en los enteros.
Observe que n = 7 sı́ sirve ya que 21 = 2 · 10 + 1.
124

40. (c) Observamos que


(5a + 1)(3a + 2) = 15a2 + 13a + 2.

El problema se reduce a buscar los números a tales que 13a + 2 es divisible por 15,
es decir, por 3 y por 5. Tenemos

13a + 2 = 10a + (3a + 2) y


13a + 2 = 12a + (a + 2)

Necesitamos que 3a + 2 sea divisible por 5 y a + 2 sea divisible por 3. La segunda


relación dice que a debe ser de la forma 3k + 1 (con k = 0, 1, ...). Regresando a la
primera relación
3(3k + 1) + 2 = 9k + 5.

Luego k debe ser múltiplo de 5. Entonces son los números de la forma 15s + 1 con
s = 0, 1, 2, ..., 99, pues el mayor número, menor que 1500, que tiene esa forma es
1486 = 15 · 99 + 1; de modo que hay 100 números con la propiedad pedida.

41. (a) Tenemos

n = 65743 · 5438 + 34257 · 4562 + 65743 · 4562 + 34257 · 5438


= 65743(5438 + 4562) + 34257(5438 + 4562)
= 65743 · 104 + 34257 · 104
= (65743 + 34257) · 104 = 109 .

42. (d) Tenemos que

ar + ar 2 + ar 3 + · · · + ar 10 = 18 (1)
1 1 1
+ 2 + · · · + 10 = 6 ⇒ (2)
ar ar ar
1 + r + · · · + r9
=6 (3)
ar 10
Dividiendo (1) entre (3):

(ar + ar 2 + · · · + ar 10 )ar 10
= 3 ⇒ a2 r 11 = 3.
1 + r + · · · + r9

Entonces (ar)(ar 2 ) · · · (ar 10 ) = a10 r 55 = (a2 r 11 )5 = 35 = 243.

43. (d) Sean a, b, c, d, e, f las longitudes de los segmentos en que se ha dividido los lados
del rectángulo (ver la figura).
125

6 f

12 4 6 e

8 d
a b c

A partir de los perı́metros conocidos tenemos a + e = 6, b + e = 2, c + e = 3,


d + b = 4, f + b = 3. Sumando tenemos

a + b + c + d + e + f + 2b + 2e = 18
a + b + c + d + e + f = 18 − 2(b + e) = 14

El perı́metro del rectángulo grande es 28.

44. (d) Tenemos n2 + 2n = n(n + 2). Hay que determinar cuándo n2 + n − 1 y n + 2 tienen
divisores comunes: n2 + n − 1 = n(n + 2) − (n + 1). Pero n + 1 y n + 2 no pueden
tener divisores comunes por lo que la fracción siempre es irreducible.

45. (a) El área del triángulo ABC se puede calcular tomando como base 100 y altura 120,
125h
entonces el área es 6000. También se puede calcular la misma área como b·h
2 = 2 .
Ası́, 125h
2 = 6000 y, por lo tanto, h = 96.

46. (b) Debemos tener b < a, de lo contrario la resta x − y serı́a negativa (no podrı́a ser
un cuadrado perfecto). Además b = a − 1, pues si b < a − 1 entonces x − y ≥ 100
(no podrı́a ser de dos cifras). Ahora,

x = 100a + 10b + c
y = 100b + 10c + a,

entonces

x − y = 100(a − b) + 10(b − c) + c − a
= 100(a − a + 1) + 10(a − 1 − c) + c − a
= 9(10 − c + a).

Para que este número sea cuadrado perfecto debemos tener que 10 − c + a es
un cuadrado perfecto. Como x − y es de dos cifras, entonces 10 − c + a = 9, o
10 − c + a = 4 o 10 − c + a = 1, para el primer caso se tienen siete números que
cumplen las condiciones: 213, 324, 435, 546, 657, 768, 879. Para el segundo caso hay
dos: 218 y 329. Para el tercer caso no hay ninguno. En total son 9.
126

47. (d) Invirtiendo las expresiones dadas se tienen las ecuaciones


1 1 1
+ = ,
y x a
1 1 1
+ = ,
z x b
1 1 1
+ =
y z c
1 1 1
Luego x − z = a − c1 , 1
x + 1
z = 1
b ⇒ 2
x = 1
a + 1
b − c1 , esto es, x = 2abc
ac+bc−ab .

48. (c) Observe que al escribir por última vez el número n hemos escrito en total
1 + 2 + 3 + 4 + ··· + n
números (una vez 1, dos veces 2, tres veces 3, ..., n veces n). Pero
n(n + 1)
1 + 2 + 3 + 4 + ··· + n = .
2
n(n + 1)
Si hacemos = 1993 obtenemos n2 + n − 3986 = 0 y, por lo tanto n = −63, 7
2
ó n = 62, 7, pero n debe ser entero positivo. Lo que ocurre es que el término número
62 · 63
1993 no es el último de la secuencia iguales a n. Para n = 62 tenemos = 1953,
2
es decir, el término número 1953 es el último 62. Luego, en la secuencia, siguen los
números 63; 40 de ellos para llegar al término número 1993. Esto es, el término número
1993 es un 63 y al dividir 63 por 5 se obtiene 3 como resto.
√ 2 2
49. (a) Como
√ 2 BD = 2√y m]DBC =√30 , entonces√
◦ DC = 1 y BC = √ √3. Luego AD √ =√1 +
(2+ 3) = 8+4 3√= 6+2·2 √ 3+2 = 6+2 12+2 = 6+2 6 2+2 = ( 6+ 2)2 .
Por lo tanto AD = 6 + 2.
50. (b) b < a < c ⇒ a − b < c − b, y√como b > 0: c − b√< c ⇒ 0 < a −
√b < c − √
b<c

⇒ 0 < a− b √ < c < a +b ⇒ 0 < a − b < c < a + b ⇒ 0 < a − b + a <
√ √ √
c+ a< a+b+ a ⇒
1 1 1
√ √ <√ √ <√ √ .
a+b+ a c+ a a−b+ a
Racionalizando el denominador de cada fracción obtenemos que B < A < C .
51. (a) Consideremos el triángulo ABC que se muestra en la figura:

5 a
b √
40

B a A
127


a 2 2

b 2
Por el Teorema de Pitágoras se satisfacen las relaciones 2 +b = 25, a2 + 2 =
40. Es decir
a2 + 4b2 = 100
4a2 + b2 = 160

De aquı́ se sigue que a2 = 36 y b2 = 16. De donde c2 = 52 y, entonces, c = 2 13.
52. (a) La suma es
1 1 2
 1
 
S= 1 + 2 + 2 + 3 + 32 + 3
3 + ··· + 1
1000 + ··· + 1000
1000
= 1 + 12 (1 + 2) + 13 (1 + 2 + 3) + · · · + 1
1000 (1 + 2 + 3 + · · · + 1000)
= 1+ 21 · 2·3 1 3·4 1 1000·1001
2 + 3 · 2 + · · · + 1000 · 2
1
= 1 + 2 (3 + 4 + 5 + · · · + 1001)
= 250 750.

DESARROLLO
   
1. Observamos que si x < 18, entonces x2 < 9 y x3 < 6 ⇒ x2 + x3 < 9 + 6 = 15,
de modo que ningún entero  menor
  que 18 resuelve la ecuación. Si x = 18 tenemos
que x2 = 9 y x3 = 6 ⇒ x2 + x3 = [9] + [6] = 9 + 6 = 15. Si x = 19 tenemos que
19 19
x  x 
2 = 2 = 9, 5 y 3 = 3 = 6, 33 ⇒ 2 + 3  = [9,  5]+[6,
 33] = 9+6 = 15. Finalmente,
x x

si x ≥ 20, entonces x2 ≥ 10 y x3 ≥ 6, 66, y x2 + x3 ≥ [10] + [6, 66] = 10 + 6 = 16.


Por lo tanto las únicas soluciones de la ecuación son x = 18 y x = 19.
2. Multiplicando la primera ecuación por 5 y la segunda por −2 tenemos

5mx + 10y = 5a
−14x − 10y = −2b
Sumando estas ecuaciones se tiene que
(5m − 14) x = 5a − 2b.
La única forma de que x sea entero para cualesquiera sean los valores enteros de a
y b es que 5m − 14 = 1, es decir m = 3.
3. En primer lugar vemos que
k 3 + m3 + n3 = (k + m + n)3 − 3 (k + m) (m + n) (n + k)

Como k + m + n es divisible por 6, entonces (k + m + n)3 es divisible por 6. Además,


como todo número entero positivo es par o impar, dados tres de ellos siempre tendre-
mos al menos dos que tienen la misma paridad (ambos pares o ambos impares) de
modo que al menos una de las suma k + m, m + n ó n + k es par y por lo tanto
3 (k + m) (m + n) (n + k) es múltiplo de 6. Como ambos términos del lado derecho de
la igualdad anterior son múltiplos de 6, entonces k 3 + m3 + n3 es múltiplo de 6.
128

4. Consideremos el polinomio
Q(x) = P(x) − 1.
Tenemos que
Q(a) = Q(a + 1) = Q(a + 2) = 0.
Es decir, a, a + 1, a + 2 son raı́ces de Q(x). De manera que podemos escribir

Q(x) = (x − a) (x − a − 1) (x − a − 2) R(x) (1)

Donde los polinomios Q(x) y R(x) también tienen solamente coeficientes enteros. Si
existiera k tal que P(k) = 8, entonces tendrı́amos Q(k) = 7. Reemplazando en (1):

7 = Q(k) = (k − a) (k − a − 1) (k − a − 2) R(k).

Pero 7 no tiene tres factores consecutivos diferentes de 0. Por lo tanto, no existe


ningún entero k tal que P(k) = 8.

5. Sean n − 1, n, n + 1 las longitudes de los lados AB, BC y C A de T. El área de T es


1p 1 p
A(T ) = 3n(n − 2)n(n + 2) = n 3n2 − 12
4 4
(fórmula de Herón). Para que A(T ) sea entero es necesario que n sea par, ponemos
n = 2m, entonces p
A(T ) = m 3m2 − 3,

de donde 3m2 − 3 es la longitud de la altura AD y es un entero. Ahora, por Pitágoras,

DC 2 = (2m + 1)2 − 3(m2 − 1) = (m + 2)2 ⇒


DC = m + 2yBD = 2m − (m + 2) = m − 2.

Finalmente, DC − BC = 4 y como los triángulos rectángulos tienen los lados enteros


entonces son pitagóricos.

6. Sea x el precio del cuaderno, y el precio del lapicero y z el precio del lápiz. De las
primeras condiciones se tiene que
x y 2z
+ + = 80
5 2 5

De las otras condiciones se tiene que


x y z
+ + = 120.
2 4 3

Multiplicando por 10 la primera ecuación y por 12 la segunda, se tiene el sistema



2x + 5y + 4z = 800
(1)
6x + 3y + 4z = 1440
129

Sumando estas dos ecuaciones tenemos


8x + 8y + 8z = 2240,
y por lo tanto la compra total costó x + y + z = 280 colones.
Ahora, si en el sistema (1) restamos las ecuaciones se tiene 2x − y = 320, es decir
x + (x − y) = 320 (2). Como la compra total fue de 280, entonces x ≤ 280 y entonces,
por (2), x − y > 0, es decir x > y y por lo tanto es más caro el cuaderno que el
lapicero.
7. Sean va la velocidad del automóvil, vb la velocidad de la bicicleta y vm la velocidad de
la motocicleta. Sea X la distancia de A a B. Sea t1 el tiempo que tarda el automóvil
en ir a B, regresar y encontrarse a la motocicleta, en este caso la distancia que ha
X +x
recorrido es X +x y entonces t1 = , pero este es el mismo tiempo que ha tardado
va
la motocicleta hasta el punto de encuentro con el automóvil; la motocicleta ha recorrido
X −x X +x X −x
una distancia igual a X − x, de modo que t1 = , es decir, = .
vm va vm
X +y X −y
Del mismo modo se pueden establecer las igualdades t2 = = y
va vb
X +z X −z
t3 = = . Estas tres ecuaciones se pueden poner en la forma
vm vb
vm (X + x) = va (X − x) (1)
vb (X + y) = va (X − y) (2)
vb (X + z) = vm (X − z) (3)

Dividiendo (1) entre (2) se tiene


vm X −x X +y
= · ;
vb X −y X +x
X +z
de (3) se tiene vm
vb = X −z , por lo tanto
X −x X +y X +z
· = .
X −y X +x X −z
Despejando X de esta última ecuación se tiene que la distancia de A a B es
r
xyz
X= .
x−y+z

8. Primero vemos que (0, 0) es una solución y es la única en la que al menos una de
las incógnitas es 0. Supongamos ahora que x 6= 0, y 6= 0; en este caso podemos “dar
vuelta” a las ecuaciones: 

 1 + x2 1
 2x 2 = y


 1 + y2 1

 =
2y 2 x
130

Es decir, 
 1 1 1
 2x 2 + 2 = y


 1 1 1
 + =
2y 2 2 x
Si escribimos u = x1 , v = 1
y (*), el sistema serı́a
 2

 u 1
 + =v (1)
2 2
 2
 v +1 =u

(2)
2 2

Restando (2) de (1) se tiene u2 − v 2 = 2(v − u) ⇒ (u − v) (u + v + 2) = 0 ⇒ u = v


ó u + v = −2.
Caso u = v. Sustituyendo en (1) obtenemos u2 + 1 = 2u ⇒ u2 − 2u + 1 = 0 ⇒
(u − 1)2 = 0 ⇒ u = 1 y, por lo tanto, v = 1. Por (*) tenemos x = 1, y = 1. Se obtiene
ası́ otra solución del sistema (1, 1).
Caso v = −2 − u. Al sustituir esto en (1) se tiene u2 + 2u + 5 = 0, que no tiene
soluciones reales. El sistema tiene solo dos soluciones reales (0, 0) y (1, 1).

9. Consideremos la figura.

D C
O
h 60◦ 5

A B
10

Al trazar las diagonales en un paralelogramo, éste queda dividido en cuatro triángulos


de igual área, de manera que el área del paralelogramo es 4 veces el área de uno de
estos triángulo. Ası́, con la notación de la figura se tiene que

(ABC D) = 4 · (BOC )
= 2 · OB · OC · sin 60◦ (*)

= 3 · OB · OC

Aplicando el teorema de los cosenos al triángulo BOC , se tiene

25 = OB 2 + OC 2 − 2 · OB · OC · cos 60◦
25 = OB 2 + OC 2 − OB · OC . (1)
131

Dado que las diagonales se cortan en sus puntos medios, entonces OD = OB y


OA = OC . Ası́, aplicando el teorema de los cosenos al triángulo DOC , se tiene
100 = OB 2 + OC 2 − 2 · OB · OC · cos 120◦
100 = OB 2 + OC 2 + OB · OC . (2)

De (1) y (2) se deduce que OB 2 + OC 2 = 125 . Sustituyendo esto en (1) se tiene que
75 75
√ 2
OB · OC = 2 y, por (*), (ABC D) = 2 3. Por otra parte, si h es la altura pedida,

sabemos que el área del paralelogramo es h · AB y, por lo tanto h · AB = 752 3 ⇒
√ √
10h = 752 3 ⇒ h = 75 20 3.

10. Sea p = (x + 2y), la expresión original se transforma en


(p − y)2 + (p + y)2 − 10(p + y) − 4(p − y) + 29 = 0
Desarrollando y simplificando se convierte en
29
p2 − 7p + y2 − 3y + = 0.
2
Completando cuadrados se obtiene
7 3
(p − )2 + (y − )2 = 0.
2 2
De aquı́ se tiene que p = 72 y y = 32 , de donde 2y2 + xy = y(x + 2y) = yp = 21
4.
 √  p 
11. x + x 2 + 1 y + y2 + 1 = 1 ⇒
p 1
x+ x2 + 1 = p (1)
y + y2 + 1

Racionalizando el denominador en el lado derecho se obtiene


p q
x + x 2 + 1 = y2 + 1 − y
y, entonces q p
x+y= y2 + 1 − x 2 + 1 (1)

Si en (1) racionalizamos el numerador en el lado izquierdo, entonces


1 1
√ = p ⇒
2
x +1−x y + y2 + 1
q p
y + y2 + 1 = x 2 + 1 − x

y, entonces q
p
x+y= x 2 + 1 − y2 + 1 (3)

Sumando las igualdades (2) y (3) se tiene que 2(x + y) = 0 y, por lo tanto, x + y = 0.
132

12. Sea M(A) la suma de los volúmenes de aceites s (soya), m (maı́z), g (girasol) en el
recipiente A, M(B) la suma de los volúmenes en en el recipiente B, M(C ) la suma de
los volúmenes en el recipiente C y V (A) la cantidad de litros en el recipiente A, V (B)
la cantidad de litros en el recipiente B y V (C ) la cantidad de litros en el recipiente
C. Al inicio se tiene
M0 (A) = 2s V0 (A) = 2
M0 (B) = 3m V0 (B) = 3
M0 (C ) = 4g V0 (C ) = 4

Al transferir un litro de A a B se tiene


M1 (A) = 1s V1 (A) = 1
M1 (B) = 3m + 1s V1 (B) = 4
M1 (C ) = 4g V1 (C ) = 4

Al transferir 1 litro de B a C se tiene

M2 (A) = 1s V2 (A) = 1
3 3 9
M2 (B) = (3m + 1s) = s + m, V2 (B) = 3
4 4 4
1 1 3
M2 (C ) = 4g + (3m + 1s) = s + m + 4g, V2 (C ) = 5
4 4 4
Al transferir 1 litro de C a B se tiene

M3 (A) = 1s V3 (A) = 1
3 9 1 1 3 4 12 4
M3 (B) = s + m + ( s + m + 4g) = s + m + g, V3 (B) = 4
4 4 5 4 4 5 5 5
Finalmente, al transferir 1 litro de B a A se tiene en A:
1 4 12 4 6 3 1
M4 (A) = 1s + ( s + m + g) = s + m + g, V4 (A) = 2
4 5 5 5 5 5 5
6/5 6
Ası́, en el recipiente A, el porcentaje de soya es 2 = 10 = 60 %, el de maı́z es
3/5 3 1/5 1
2 = 10 = 30 % y el de girasol es 2 = 10 = 10 %.

13. Supongamos que esto es falso. Enumeremos a los que están alrededor de la mesa en
orden, empezando por cualquier lugar. Si en la k−ésima silla hay un hombre, entonces
es claro que en la casilla k − 2 y en la casilla k + 2 hay mujeres. Pero, puesto que
hay el mismo número de hombres que de mujeres, entonces para cualquier mujer que
esté sentada en la silla n, es correcto que en la n − 2 y en la n + 2 hay hombres.
Si ahora consideramos solo las 31 personas que están sentadas en las sillas pares,
entonces obtenemos que entre estas personas los hombres y las mujeres se alternan,
si recorremos la mesa en alguna dirección, pero esto no es posible puesto que 31 es
impar.
133

14. Si k(k+1)
2 = N 2 entonces k 2 + k − 2N 2 = 0. Los valores posibles para N 2 son 12 , 22 ,
2
. . ., 99 . Para que k sea entero, el discriminante D de la ecuación de segundo grado
debe ser un cuadrado perfecto, esto es
D = 12 − 4(−2 · N 2 ) = 1 + 8N 2
tiene que ser cuadrado perfecto. Esto reduce los posibles valores de N 2 a 12 , 62 y 352 .
Ası́, los valores enteros positivos de k son 1, 8 y 49.
15. En el triángulo BC D, BE y C E son bisectrices interiores del triángulo, estas se
intersecan en E. Como las bisectrices de un triángulo son concurrentes, se debe tener
que DE es bisectriz de ∠BDC , por lo que m∠BDE = m∠EDC .
16. Para que la ecuación tenga soluciones para cualquier valor de m su discriminante
tiene que ser no negativo para cualquiera que sea el valor de m, es decir:
 
(5m + a)2 − 4 6m2 + 5m − 4 ≥ 0
m2 + (10a − 20) m + 16 + a2 ≥ 0

para todo m. Para esto, el discriminante de m2 + (10a − 20) m + 16 + a2 ≥ 0 tiene


que ser negativo o 0; es decir
 
(10a − 20)2 − 4 16 + a2 ≤ 0
6a2 − 25a + 21 ≤ 0
 
Al resolver esta última inecuación tenemos 67 ≤ a ≤ 3. Es decir, si a ∈ 76 , 3 ,
entonces la ecuación dada tendrá soluciones para cualquiera que sea el valor de m.
17. Observamos que si k es un número entero positivo entonces
4k 2 + 4k + 1 > 4k 2 + 4k
y, por lo tanto, (2k + 1)2 > 2k · 2 (k + 1). Esto implica que
1 1
2
< ⇒
(2k + 1) 2k · 2 (k + 1)
2 1 1 1
2
< = − .
(2k + 1) 2k (k + 1) 2k 2k +2

Utilizando esto tenemos que


1 1 1 1 1 1
+ + ··· + 2
= 2 + 2 + ··· +
9 25 (2n + 1) 3 5 (2n + 1)2
     
1 1 1 1 1 1 1
< − + − + ··· + −
2 2 4 4 6 2n 2n + 2
 
1 1 1 1 1 1
= − = − < .
2 2 2n + 2 4 4n + 4 4
134

18. Si a cada vértice designado con A le asignamos el valor 2 y a cada vértice designado
con B le asignamos el valor √12 , entonces vemos que el valor que corresponde a cada
lado es igual al producto de los números asignados a sus vértices. De este modo
el producto de todos los números escritos sobre los lados del polı́gono es igual al
cuadrado del producto de todos los números asignados a los vértices, puesto que cada
vértice pertenece a dos lados. Por lo tanto, el producto buscado es
 2
√ a  b
2 √1 = 2a−b .
2

19. Asignemos a la cuadrı́cula una red de caminos que conectan los cuadritos adyacentes
como señala el problema. A los nodos de la lı́nea inferior y a los de la lateral izquierda
se puede llegar a partir de solamente un nodo “anterior”. A los demás se puede llegar
desde tres nodos “anteriores”.

El número de maneras de llegar es igual a la suma del número de maneras de llegar


a los nodos “anteriores”.
Ahora podemos ir señalando en cada nodo el número de maneras de llegar a él, como
lo muestra la figura.

7 25 63
1

5 13 25
1

3 5 7
1

1 1 1

Vemos que para ir del cuadrito inferior izquierdo al superior derecho hay 63 maneras.

20. Si 2n3 − 1 es múltiplo de 1999 entonces, existe m entero tal que 2n3 − 1 = 1999m. Es
decir, 2n3 = 1999m + 1; de aquı́ se deduce que m tiene que ser impar. De lo anterior
135

se tiene que n3 − 1000 = 1999 · m−1


2 y como m es impar, entonces 2 =
m−1
 k entero.
3 2
Ası́, n − 1000 = 1999k. Factorizando se tiene (n − 10) n + 10n + 100 = 1999k.
Como 1999 es primo, se deduce que
 
1999|(n − 10) ó 1999| n2 + 10n + 100 .

Si 1999|(n − 10), entonces n = 10 + 1999s con s entero. Si 1999| n2 + 10n + 100 ,
entonces
n2 + 10n + 1 = 1999t,
con t entero, es decir,

(n + 5)2 + 75 = 1999t ⇒
(n + 5)2 = 1999t − 75,

el primer número de la forma 1999t −75 que es cuadrado perfecto es 1999·4−75 = 892 .
Ası́, escribiendo t = r + 4, tenemos (n + 5)2 = 1999(r + 4) − 75 = 1999r + 892 ⇒
(n + 5)2 − 892 = 1999r ⇒

(n + 5 − 89) (n + 5 + 89) = 1999r ⇒


(n − 84) (n + 94) = 1999r ⇒

1999|(n−84) ó 1999|(n+94) y, por lo tanto, n = 84+1999x (x entero) ó n = −94+1999y


(y entero). En conclusión los valores de n que satisfacen lo pedido son los n de la
forma 10 + 1999s ó 84 + 1999x ó −94 + 1999y (dicho de otra manera son los n
congruentes a 10 o a 84 o a −94 módulo 1999).
21. Observamos que los triángulos ALN y AC L son semejantes, por lo tanto AL2 = AN ·AC .
También 4ALM ∼ 4ABL y por lo tanto AL2 = AM · AB. Entonces

AL4 = AB · AC · AN · AM (1)

N
D
M

B C
L

Por otro lado, m∠MAN + m∠MLN = 180◦ , por lo tanto el cuadrilátero AMLN es
concı́clico, luego los ángulos AMN y ALN son iguales y, por lo tanto, 4ADM ∼ 4ANL,
por lo que
AD · AL = AN · AM (2)
136

De (1) y (2) se deduce que


AL3 = AB · AC · AD.

22. Trazamos la figura del enunciado y llamamos con J y K a las intersecciones de la




recta HI con AB y AC respectivamente; tenemos que JK k BC .

J
F H
D

G
I
A C
E K

Por Thales:
JH HG BD JH
= ⇒ = (1)
BD DC DC HG
GI IK BD GI
= ⇒ = (2)
BD DC DC IK

Por (1) y (2):

JH GI
= (3)
HG IK
JG GK BD JG
= ⇒ = (4)
BD DC DC GK

Por (2) y (4):


GI JG GK JG
= ⇒ = (5)
IK GK IK GI

Pero GI+IK
IK = GI
IK + IK .
IK
Luego, usando (3) y (5), resulta que

JG GI IK JH HG JG
= + = + = .
GI IK IK HG HG HG
De donde, HG = GI como querı́amos probar.

23. Llamemos con a1 , a2 , ..., a10 los diez números, y, sin perder generalidad digamos que
están colocados en orden decreciente (a1 es el mayor y ası́ sucesivamente). Ası́ tenemos
que a2 +a3 +· · ·+a10 = 86, ..., a1 +a2 +· · ·+a9 = 96 (uno de los resultados se repite
pues son 9 resultados para 10 sumas). Si sumamos todas estas igualdades tenemos
9(a1 + a2 + · · · + a10 ) = 812 + r (*), aquı́ 812 es la suma de los nueve resultados
137

dados y r es uno de ellos (el que se repite). Pero de (*) se tiene que 9|(812 + r) y esto
se logra solo con r = 88, entonces 88 es el resultado que se repite. En consecuencia

9(a1 + a2 + · · · + a10 ) = 900 ⇒


a1 + a2 + · · · + a10 = 100

y como a2 + · · · + a10 = 86 entonces a1 = 14, razonando de modo análogo obtenemos


que a2 = 13, a3 = 12, a4 = 12, a5 = 11, a6 = 10, a7 = 9, a8 = 8, a9 = 7 y a10 = 4.

24. Recordemos que x 2 + y2 ≥ −2xy. Luego,

2xy ≥ −x 2 − y2 ,

y ası́ se tiene que

−x 2 − y2 −y2 − z 2 −z 2 − t 2 −t 2 − x 2
xy + yz + zt + tx ≥ + + +
2 2 2 2
1 2
= − (2x + 2y2 + 2z 2 + 2t 2 )
2
= −(x 2 + y2 + z 2 + t 2 ) = −1

Por otra parte, podemos expresar

xy + yz + zt + tx = (x + z)(y + t).

De la primera condición del enunciado se tiene


x + z = −(y + t). Luego,

xy + yz + zt + tx = −(y + t)2 ≤ 0.

Concluimos que
−1 ≤ xy + yz + zt + tx ≤ 0.

25. Sea H el pie de la altura del 4MC B trazada desde el vértice M. En el triángulo
rectángulo MHC , el ángulo HC M mide 30◦ , entonces MH = 21 MC = b2 .

D C
Q

A B
P
138

Trazamos por M la paralela a BC que corta a la recta C D en Q y a la recta AB en


P. El paralelogramo PBC Q tiene área BC · MH = ab 2 . Este paralelogramo tiene la
misma área que el trapecio ABC D, pues

(ABC D) = (APM) + (PBC DM),


(PBC Q) = (DQM) + (PBC DM)

y los triángulos APM, DQM son congruentes (tienen DM = MA, QM = MP y


ab
m]DMQ = m]AMP). Por lo tanto (ABC D) = .
2
26. Sea a = 2n + 1, entonces 3a − 2 = 6n + 1 es un cuadrado perfecto. Considerando
b = 4n y c = n2 − 4n tenemos a + b = 6n + 1, a + c = n2 − 2n + 1 = (n − 1)2 ,
b + c = n2 , a + b + c = n2 + 2n + 1 = (n + 1)2 que son cuatro cuadrados perfectos.
Además, si a > 17, entonces n > 8 y 4n es distinto de n2 − 4n, es decir, b 6= c y
además c = n(n − 4) > 0.

27. Tenemos que (a − b)2 ≥ 0, (2a − c)2 ≥ 0, (4a − d)2 ≥ 0, entonces

0 ≤ 16(a − b)2 + 4(2a − c)2 + (4a − d)2


0 ≤ 16(a2 − 2ab + b2 ) + 4(4a2 − 4ac + c2 ) + 16a2 − 8ad + d2
0 ≤ 48a2 + 16b2 + 4c2 + d2 − 32ab − 16ac − 8ad

De aquı́ se obtiene que

48a2 + 16b2 + 4c2 + d2 ≥ 32ab + 16ac + 8ad.

28. Considere la figura

α α

B C
1 M L H 1
2

1
(a) BH = 1 + 2 = 32 , C H = 1
2 (*)
Por Pitágoras en 4ABH :
AB 2 = BH 2 + AH 2
y en 4AHC :
AC 2 = AH 2 + HC 2
139

y, entonces,

AB 2 − AC 2 = BH 2 − C H 2
2 2
= 23 − 12 = 2.
(∗)

Razón de los lados AB y AC : La bisectriz divide el lado opuesto en dos segmentos


que son respectivamente proporcionales a los lados adyacentes del triángulo:
√ √
3− 3
AC = LC = 3−1 = 3 (∗∗)
AB BL √


(b) AB 2 − AC 2 = 2 y AB = 3AC , (por (**)). Ası́,

3AC 2 − AC 2 = 2 ⇒ AC = 1 y AB = 3

Con los datos en la figura, 4AMC es isósceles puesto que MH = HC = 21 . Es decir,


AH es altura y es mediana, entonces AC = AM = 1.
1
(c) Si β = ]AC B, tenemos, considerando el triángulo AHC , que cos β = 2 y, entonces
β = 60◦ .

3
Si δ = ]ABC , tenemos, considerando el triángulo ABH, que cos δ = 2 y, entonces
δ = 30◦ .
De lo anterior tenemos que ]C AB = 90◦ .
29. Tenemos que  
299998 − 1 249999 − 1 249999 + 1
= .
3 3
Si tenemos tres números enteros consecutivos, entonces uno de ellos debe ser divisible
por tres; los tres números 249999 − 1, 249999 , 249999 + 1, son consecutivos, por lo tanto
uno de estos es divisible por tres. Observe que 249999 no es divisible por 3, porque
si lo fuera, al ser 3 primo, entonces dividirı́a a 2 y eso es falso. Se concluye que
249999 − 1 ó249999 + 1 es divisible por tres y, en cualquiera de los dos casos, entonces
249999 − 1 249999 + 1 299998 − 1
es entero, es decir, es entero.
3 3
30. Sea p un número primo que se escribe como suma de dos primos y, también, como
diferencia de dos primos. Primero vemos que p 6= 2, pues 2 no se puede escribir como
suma de dos números primos. Ası́, p es impar; como p = q + r con q y r primos y p
es impar, entonces uno de estos dos q ó r tiene que ser par y el otro impar; como el
único primo par es 2, entonces, digamos q es 2. Ası́, p = 2 + r ⇒ r = p − 2.
Del mismo modo p = s − t con s y t primos, por la misma razón de antes, debemos
tener t = 2, es decir p = s − 2 ⇒ s = p + 2. Tenemos entonces que p − 2, p y
p + 2 son primos y además son tres impares consecutivos, de modo que uno de ellos
es múltiplo de 3, entonces, necesariamente p − 2 = 3, p = 5, p + 2 = 7, de manera
que 5 es el único número que cumple las condiciones.
140

31. Los elementos A y B componen la primera mezcla en razón 3 : 5, entonces cada gramo
de la primera mezcla contiene 83 de gramo de A y 58 de gramo de B. Análogamente,
cada gramo de la segunda mezcla contiene 13 de gramo de B y 32 de gramo de C ; un
gramo de la tercera mezcla contiene 25 de gramo de A y 53 de gramo de C . Si tomamos
x gramos de la primera mezcla, y gramos de la segunda y z gramos de la tercera ,
obtendremos x + y + z gramos de la nueva mezcla. Esta nueva mezcla contendrá:
3
8x + 25 z gramos de A,
5
8x + 13 y gramos de B y
2
3y + 53 z gramos de C .
Debemos tomar la primera, segunda y tercera mezcla en cantidades tales que la mezcla
nueva contenga los elementos A, B y C en razón 3 : 5 : 2, esto significa que 1 gramos
de la nueva mezcla debe contener 103 de gramo de A, 105 de gramo del elemento B y
2
10 de gramo del elemento C . Ası́, en x + y + z gramos de la mezcla nueva habrá:
3
10 (x + y + z) gramos de A,
5
10 (x + y + z) gramos de B y
2
10 (x + y + z) gramos de C .
De todo lo anterior se forma el sistema
 3

 x + 52 z = 3
10 (x + y + z)
 8
5 1 5
 8x + 3y = 10 (x + y + z)

 2 3 2
3y + 5z = 10 (x + y + z)

Si multiplicamos la primera ecuación por −5 y la segunda por 3 y sumamos ambas


ecuaciones resultantes obtenemos −2z + y = 0, es decir, y = 2z. Sustituyendo esto
en la segunda ecuación y despejando x, obtenemos x = 203 y. Si sustituimos estos dos
resultados en la tercera ecuación vemos que ella se satisface; entonces, la proporción
en que hay que tomar las mezclas es x : y : z = 20 : 6 : 3.

32. De acuerdo con la hipótesis, 1


a + 1
b + 1
c = abc, entonces bc + ac + ab = (abc)2 .
Entonces se tiene que

a2 b2 c2 + c2 ab + bc + ac + c2 (b + c) (a + c)
a2 b2 + 1 = 2
= 2
= ,
c c c2
a2 b2 c2 + a2 ab + bc + ac + a2 (a + b) (a + c)
b2 c2 + 1 = = = ,
a2 a2 a2
a2 b2 c2 + b2 ab + bc + ac + b2 (a + b) (b + c)
c2 a2 + 1 = = = .
b2 b2 b2
141

De donde se tiene
     (a + b)(b + c)(a + c) 2
2 2 2 2 2 2
a b +1 b c +1 c a +1 =
abc

33. Con la notación de la figura, tenemos


m∠ABD = m∠AC D = m∠F DM = m∠EDM = 60◦ .

E M F

B C
D

a 3
Tenemos

BD = DC = a2 ; DE = DF = a
2 sin 60◦ = 4 . Además sin 60◦ = FM
DF ⇒
3 3a 6a
2 = F√M
a 3
⇒ FM = 8 ⇒ F E = 2F M = 8 . Por Pitágoras en 4F MD se tiene que
2√ √ √
a 3 1 6a a 3 3a2 3
MD = 8 . De manera que el área de 4DEF es igual a 2 · 8 · 8 = 64 .

34. Como (y − z)2 + (z − x)2 + (x − y)2 = (y + z − 2x)2 + (z + x − 2y)2 + (x + y − 2z)2 ,


entonces
(x − y)(x − z) + (y − z)(y − x) + (z − x)(z − y) = 0.

Hagamos y−z = a, z −x = b, x −y = c. Entonces bc +ca+ab = 0 y a+b+c = 0.


Luego (a + b + c)2 − 2(bc + ca + ab) = 0; es decir, a2 + b2 + c2 = 0, de donde
a = b = c = 0, es decir, x = y = z.
35. Se bajan desde D y E las perpendiculares sobre el lado AC como se muestra en la
figura.

a
D
T
a
m R E
S
a n a
C A
b F b G b
142

Se tiene que 4AEG ∼ 4DRE, puesto que los ángulos A y E son congruentes,
los mismo que los ángulos D y E y los segmentos DE y EA son congruentes. Ası́,
RE = b = F G y DR = a = T S. De modo análogo 4BT D ∼ 4DRE. En 4C DF y
4C EG se verifica que

m2 = (2a)2 + b2 ; n2 = a2 + (2b)2 .

Por lo tanto, m2 + n2 = 5(a2 + b2 ) y, como m2 + n2 = 1 (por hipótesis), se sigue que


a2 + b2 = 51 . Finalmente, en 4ABC :
  9
AB 2 = (3a)2 + (3b)2 = 9 a2 + b2 = ,
5

de donde AB = √3 = 35 5.
5

36. Primero veamos que n es par. Si n fuera impar entonces cada ai deberá ser impar,
pero la suma de un número impar de impares no puede ser igual 0. Luego n es par.
Ahora veamos que n es múltiplo de 4. Si n fuera de la forma

n = 4m + 2 = 2(2m + 1),

entonces alguna ai es 2 y las restantes n − 1 son impares, luego, la suma de los ai


impares es impar (pues n − 1 es impar) y si agregamos el 2 tenemos que

a1 + a2 + · · · + an es impar,

luego, no puede ser 0. Ası́, m debe ser de la forma 4m.

37. Acomodamos el rectángulo con el lado de 15 horizontal y el de n vertical. Veamos


como podemos llenar el renglón hasta abajo. Podemos colocar una U con el lado de 3
hacia abajo, ocupando tres cuadritos de la base; las otras orientaciones posible para
U son con el lado de 2 hacia abajo.

En cualquiera de los casos, el espacio vacı́o solo puede ser llenado (de modo que
el resto del rectángulo pueda ser construido) con una cruz y después de la cruz
forzosamente debe colocarse otra U ası́:
143

o o a o o

o a a a o

o o a o o

(La cruz formada por a’s y las U’s por o’s) y ası́ ocupamos 5 cuadritos de la base.
Es claro que si ponemos una cruz en la base, debemos poner necesariamente dos U’s
a los lados y formar otro de estos rectángulos de 5 × 3. Ası́ que en cualquier caso
ocupamos 3 ó 5 cuadritos de la base. Entonces el número de cuadritos de base, 15,
debe ser expresado como suma de 3’s y 5’s, esto solo puede hacerse de dos formas,
tres cincos o cinco treces. Con los 3 cincos estarı́amos poniendo 3 de esos rectángulos
de 5 × 3 y por lo tanto ocuparı́amos un rectángulo de 15 de base por 3 de alto. Si
usamos los 5 treces, son 5 U’s horizontales, es fácil ver que en las cavidades de las
U’s solo podemos poner cruces, ası́

# a = $ b = c = ...

a a a b b b c c c ...

o a o o b o o c o ...

o o o o o o o o o o

Las cruces están hechas de a’s, b’s, c’s ... y las U’s de o’s. Como en el cuadro marcado
con # solo puede ir una U de cabeza, después de ponerla repetimos el argumento
en el cuadro $ y ası́ sucesivamente, vemos que debemos completar un rectángulo de
15 × 3 o bien uno de 15 × 5, si ahora aplicamos el argumento a lo que quede del
rectángulo de 15 × n, vemos que se puede llenar si y solo si n = 3a + 5b, con a, b
enteros no negativos. Los número menores que 10 que son de esa forma son 3, 5, 6, 8
y 9. Además todos los n > 9 son de esa forma; en efecto, sea n = 3q + r (con r = 0, 1
ó 2). Como n > 9, entonces q > 3. Si r = 0, n = 3q + 5 · 0.
Si r = 1, n = 3(q − 3) + 5 · 2.
Si r = 2, n = 3(q − 1) + 5 · 1.
En conclusión, el rectángulo puede formarse si n = 3, 5, 6 o n ≥ 8.
1 1 1 1 1 1 1 1 1 1
38. Observe que 5 = 1·5 , 45 = 5·9 , 117 = 9·13 , 221 = 13·17 , 357 = 17·21 , de modo que la
1
n−ésima fracción es .
(4n − 3)(4n + 1)
144

Para calcular la suma de los trece primeros términos note que


 
1 1 1 1
= − ,
(4n − 3)(4n + 1) 4 4n − 3 4n + 1

de manera que
1 1 1 1 1
1·5 + 5·9 + 9·13 + · · · + 45·49 + 49·53
    
= 41 11 − 15 + 14 15 − 91 + 14 91 − 131 + ··· + 1
4
1
45 − 1
49 + 1
4
1
49 − 1
53
 
1 1 1 52 13
= 1− = · = .
4 53 4 53 53

39. Considere la figura:

X
m D r
F C

n n

A m E
r B
Y

Por L–A–L se sigue que 4DXC ∼


= 4BYA. Entonces

m∠DXC = m∠AY B, XC = YA.

Luego, AXC Y es un paralelogramo, lo mismo que AF C E. Si llamamos los perı́metros


de los triángulos BC E y C DX por PBC E , PC DX , respectivamente, entonces se tiene
que el perı́metro de 4C F Y es

m + DC + C B + BY + r + n = (m + n + C B) + (DC + DX + r)
= PBC E + PC DX

De donde, p = (a + 2b) + (b − 2a) = 3b − a.

40. Si el cuadrado de un número es igual al cubo de otro, entonces este número es una
sexta potencia y, por tanto, el número es un cubo perfecto. Los números menores que
1000 (de más de una cifra) cubos perfectos son: 27, 64, 125, 216, 343, 512, 729. Como el
cuadrado de un número de tres cifras es mayor que 10000 y 3 · 93 = 2187 < 10000,
entonces el número debe ser de dos cifras. El único que cumple las condiciones es 27.
145

41. Considere la figura:

D a C
F

P
Q

A a
B E

Se tiene m∠C PD = m∠EPB (opuestos por el vértice), m∠PC D = m∠PBE (ambos


son rectos) y a = DC = BE, entonces 4DC P ∼
= 4EBP, por lo tanto BP = C P, es
1
2 a·a a2
decir, BP = a2 . Por otra parte, (PBE) = (PBQ) + (BQE); pero (PBE) = 2 = 4 ;

1 a
(PBQ) = · · BQ · sin 30◦
2 2
1
y (BQE) = 2 · BQ · a · sin 60◦ . Entonces

a2 a a
= · BQ · sin 30◦ + · BQ · sin 60◦
4 4 2

2a(2 3−1)
y, despejando BQ, se tiene BQ = .
Por lo tanto, el área buscada es
11
 √ 
a 2 2 3−1
1
(PBQ) = PB · BQ · sin 30◦ = .
2 44

42. La serie de números primos es infinita, de modo que para cualquier entero positivo a,
existen tres números primos p1 , p2 , p3 con a < p1 < p2 < p3 que tomaremos como
los tres primeros mayores que a. Tenemos M = a3 + b3 = (a + b)(a2 − ab + b2 ). Si
Cecilia elige b = p1 p2 p3 − a, consigue
h i  
M = [a + (p1 p2 p3 − a)] a2 − ab + b2 = p1 p2 p3 a2 − ab + b2

De modo que M contiene al menos tres factores primos diferentes.

43. Si (w, x, y, z) es solución del sistema, entonces es solución de la ecuación:

yz − w − x + wx − y − z = 0,

que es equivalente a

(w − 1) (x − 1) + (y − 1) (z − 1) = 2.
146

Encontremos las soluciones enteras positivas de esta última ecuación y comprobemos


si son soluciones del sistema original. Como las soluciones son enteras tenemos tres
casos:
(a) (w − 1)(x − 1) = 2, y (y − 1)(z − 1) = 0, que admite las soluciones:
(2, 3, 1, 5), (3, 2, 1, 5), (2, 3, 5, 1) y (3, 2, 5, 1).
(b) (w − 1)(x − 1) = 1, y (y − 1)(z − 1) = 1, que admite solo la solución (2, 2, 2, 2).
(c) (w − 1)(x − 1) = 0, y (y − 1)(z − 1) = 2, que admite las soluciones:
(1, 5, 2, 3), (1, 5, 3, 2), (5, 1, 2, 3) y (5, 1, 3, 2).
Es fácil comprobar que estas nueve cuartetas son soluciones del sistema original.

44. Sea an la cantidad que recibe el individuo n, n ≥ 2. Tenemos a2 = 5, a3 = 5 · 2 + d,


a4 = 2(5 · 2 + d) + d = 22 · 5 + d(1 + 2), . . .,

an = 5 · 2n−2 + d(1 + 2 + 4 + · · · + 2n−3 ) = 5 · 2n−2 + d(2n−2 − 1).

Como a10 = 1917, 5, entonces 1917, 5 = 5 · 28 + d(28 − 1) y, entonces d = 25 . Ası́

5  13 
a15 = 5 · 213 + 2 − 1 = 61437, 5.
2

45. Considere la figura:

a
b
D
a
b−a

B C

Con los datos del enunciado tenemos que m∠BAC = 36◦ , m∠ABC = m∠AC B = 72◦ ,
entonces en 4BC D se tiene que m∠BC D = 36◦ , m∠C DB = m∠DBC = 72◦ .
Por otra parte, en 4AC D se tiene que m∠DAC = m∠AC D = 36◦ , m∠ADC = 108◦ .
De lo anterior se deduce que 4BC D y 4ADC son isósceles y 4BC D es semejante
a 4BAC . Para los lados se tiene: DC = AD = a, BD = b − a. Expresando la
147

proporcionalidad derivada de la semejanza anterior:

b−a a
= ⇔ a2 = b2 − ab
a b
⇔ a2 + ab − b2 = 0
 a 2 a
⇔ + −1=0
b b √ 
√ 5 − 1 b
a 5−1
⇒ = ⇒a= .
b 2 2

46. Tenemos que

p2 + q 2 p2 + q2 + 2pq − 2pq (p + q)2 − 2pq 2pq


= = =p+q− .
p+q p+q p+q p+q

2+q 2 2pq
Luego, pp+q es entero si y solo si p+q es entero. Pero, en este caso, p + q divide a
2pq y, como los divisores de 2pq son 1, 2, p, q, 2p, 2q, pq, 2pq tendrı́amos:
X p + q = 1 ó p + q = 2, imposible;
X p + q = p ó p + q = q ⇒ p = 0 ó q = 0, imposible;
X p + q = 2p ó p + q = 2q ⇒ p = q ;

X p + q = pq ⇒ p = q(p − 1) y como p y q son primos, entonces p = q = 2 ;


X p + q = 2pq ⇒ p = q(2p − 1) y como q es primo, obtenemos p = 1, imposible.

47. Los divisores de 26 son 1, 2, 13 y 26, por lo que a2 + b2 + c2 tiene que dar como
resultado alguno de estos cuatro números. Se harán cuatro casos:
Caso I: a2 + b2 + c2 = 1. El único cuadrado perfecto menor o igual que 1 es 12 = 1.
Como solo hay un número posible para a, b, c, entonces las otras posibilidades son
dos ceros, pero como a 6= 0, entonces a = 1, b = c = 0. Ası́ el único número de tres
dı́gitos abc en que a2 + b2 + c2 = 1 es 100.
Caso II: a2 + b2 + c2 = 2. Aquı́ también el único menor que 2 que es cuadrado
perfecto es 1 y la forma en que se puede utilizar para sumar 2 es
12 + 12 + 02 = 2; como a 6= 0, entonces a = 1 y si b = 1, entonces c = 0 y si b = 0
entonces c = 1. Ası́ los números abc que cumplen la propiedad a2 + b2 + c2 = 2 son
101 y 110.
Caso III: a2 + b2 + c2 = 13. Los cuadrados perfectos menores que 13 son 12 = 1,
02 = 0, 22 = 4, 32 = 9. La única forma de combinar tres de estos cuadrados para que
sumen 13 es 02 + 22 + 32 = 0 + 4 + 9 = 13; como a 6= 0 entonces puede ser 2 ó 3,
de esta forma obtenemos que los números abc con a2 + b2 + c2 = 13 son 203, 230,
302, 320.
148

Caso IV: a2 + b2 + c2 = 26. Tenemos que 02 , 12 , 22 , 32 , 42 y 52 son los cuadrados


menores que 26; los únicos cuadrados que suman 26 son 02 +12 +52 = 2 y 12 +32 +42 =
26. Las maneras de combinarlos en números de tres dı́gitos abc (con a 6= 0) son 105,
150, 501, 510, 134, 143, 314, 341, 413 y 431. Se obtienen ası́ 17 números con las
condiciones pedidas, ellos son: 100, 101, 105, 110, 134, 143, 150, 203, 230, 302, 314, 320,
341, 413, 431, 501, 510.

48. Observamos primero que

a2 a(a + b) − ab ab
= =a− .
a+b a+b a+b

a+b 2
Como ab ≤ 2 , tenemos ab
a+b ≤ 4 ,
a+b
luego − a+b
ab
4 . Por lo tanto,
≥ − a+b

a2 b2 c2   
+ + = a− + b − b+c
bc
ab
+ c − c+a
ca
a+b b+c c+a 
a+b

a+b b+c c+a
≥a+b+c− + +
4 4 4
a+b+c
= = 1.
2

49. Sea ai el total acumulado de libros vendidos hasta el final del i−ésimo dı́a (por
ejemplo a10 representa el total de libros vendidos desde el primer dı́a hasta el décimo
inclusive). Como al menos se vendió un libro por dı́a, entonces

a1 < a2 < . . . < a364 < a365 = 600 ([)

(a365 = 600 pues es el total vendido en el año). El total de libros vendidos en el


perı́odo que va del dı́a (i + 1) al dı́a j, ambos inclusive, es aj − ai . Queremos ver si
existen i, j con j > i, tal que aj − ai = 129. A cada término en ([) súmese 129:

a1 + 129 < a2 + 129 < . . . < a365 + 129 = 729.

Sea bi = ai + 129:
b1 < b2 < . . . < b364 < b365 = 729.

Tenemos 365 término ai y 365 términos bi , entre 1 y 729, esto es, hay 730 términos que
pueden tomar 729 valores diferentes, por lo tanto dos de ellos tienen que ser iguales;
pero si i 6= j entonces bi 6= bj y ai 6= aj , por lo que existen m y n tales que bn = am
y entonces an + 129 = am ⇒ am − an = 129; entonces en el perı́odo de m − n dı́as
se vendieron 129 libros exactamente.

50. Como son consecutivos se tiene que estas secuencias siempre tienen la forma

a + 1, . . . , a + k
149

y se quiere que su suma sea 2000. Ası́ se obtiene

k(k + 1)
ka + = 2000,
2

es decir,
4000 − k 2 4000
2a + 1 = = −k (1)
k k

De (1), por ser a y k enteros, se tiene que k divide a 4000 y, además k < 63. Además
2
2a + 1 = 4000−k
k debe ser entero impar, luego los valores de k que sirven son

k = 1 que lleva a {2000}


k = 5 que lleva a {398, 399, 400, 401, 402}
k = 25 que lleva a {68, 69, ..., 91, 92}
k = 32 que lleva a {47, 48, ..., 77, 78}

51. Trace la altura correspondiente a AN que corta AN en R, sean x = |AR|, y = |RN|,


|AB| = a y |PR| = h. Dado que 4ARP ∼ 4ABM se tiene que ax = a/2 h
además
y
como 4NRP ∼ 4NAD se tiene que a/2 = a . De estas últimas relaciones se obtiene
h

que yh = hx por lo que 4ARP ∼ 4PRN, de donde se obtiene la conclusión.

52. Sea Sn la suma de todos los xm tales que m tiene n cifras. Vamos a separar los
números de n cifras en 9 grupos, G1 , G2 , ..., G9 según su primer dı́gito sea 1, 2, ..., 9:
  

 1000..,0 
 2000..,0 
 9000..,0

 
 


 1000..,1 
 2000..,1 
 9000..,1

 .. 
 .. 
 ..

 
 

 .  .  .
G1 = 1099..,9 G2 = 2099..,9 ...G9 = 9099..,9

 
 


 1100..,0 
 2100..,0 
 9100..,0

 .. 
 .. 
 ..

 
 


 . 
 . 
 .
  
1999..,9 2999..,9 9999..,9

Vemos que para cada grupo, los números que lo integran son idénticos si exceptuamos
el primer dı́gito. Si sumamos todos los números del grupo Gi tenemos
X X
xm = i · xm
m∈Gi m∈G1

P
Ahora, si m tiene al menos un dı́gito igual a 0, entonces xm = 0; de ahı́ que xm =
m∈G1
150

Sn−1 . Entonces
X X X
Sn = xm + xm + · · · + xm
m∈G1 m∈G2 m∈G9
X X X
= xm + 2 xm + · · · + 9 xm
m∈G1 m∈G1 m∈G1
X
= 45 xm = 45Sn−1
m∈G1

Ahora, Sn = 45Sn−1 = 45 (45Sn−2 ) = ... = 45n−1 S1 y como S1 = 1 + 2 + 3 + · · · + 9 =


45 se tiene que Sn = 45n .

53. Sean a, b tales que a2 +b2 es cuadrado perfecto, digamos k 2 , y suponga que a2 +b2 +c2
es cuadrado perfecto, digamos p2 . En ese caso se tiene:

a2 + b2 + c2 = p2
k 2 + c2 = p2
k 2 = p2 − c2
k 2 = (p − c)(p + c).

Tómese r = p + c, s = p − c, es decir p = (r + s)/2, c = (r − s)/2 de aquı́ r, s deben


tener la misma paridad.
Si r, s son pares r = 2m, s = 2n ası́ que k 2 = 4mn = rs. Tómese r = 2mn y s = 2,
en ese caso p = mn + 1 y c = mn − 1 es decir: p = k 2 /4 + 1 y c = k 2 /4 − 1
Similarmente si ambos son impares.

54. Consideremos la siguiente figura:

M F

x
h

A y C
H I N

En esta figura se ha trazado, además de lo indicado en el enunciado, la altura MH


de 4AMN, la altura BI de 4ABC y el segmento MF , paralelo a la base AC . Sea
x = AM, y = AN y h = MH; en este caso tenemos que MB = 11 − x, NC = 10 − y.
Como los perı́metros del triángulo y el cuadrilátero indicados son iguales, entonces

x + y + MN = MN + 10 − y + 9 + 11 − x
151

⇒ x + y = 15.

Utilizando la fórmula de Herón
√ obtenemos
√ que el área de 4ABC es 30 2 y, de
acuerdo con esto, 10·IB
2 = 30 2 ⇒ IB = 6 2.
Además,
√ puesto que (AMN) =√(MNC B), entonces
√ cada una de esas áreas es igual a
yh
15 2. Ası́, (AMN) = 2 = 15 2 ⇒ yh = 30 2.
Observe que (MNC B) = (MNC F ) + (MF B).
Como MF kAC , entonces 4BMF ∼ 4BAC , luego 11−x 11 10
MF√ = 10 ⇒ MF = 11 (11 − x).
En 4BMF , la altura sobre

el lado MF es BI − h = 6 2 − h. De esto tenemos que
10 6 2−h 10
(BMF ) = 11 (11 − x) · 2 . Además, (MNC F ) = [10 − y + 11 (11 − x)] h2 . Por lo tanto

(MNC B) = (MNC F ) + (MF B)



10 h 10 6 2−h
= [10 − y + (11 − x)] + (11 − x) ·
√ 11 2 11 2
= 15 2
10 √ √
⇒10h + (11 − x) · 6 2 = 30 2 + hy
11
10 √ √
⇒10h + (11 − x) · 6 2 = 60 2
11

Multiplicando la última igualdad por y tenemos

10 √ √
10hy + (11 − x) · 6 2y = 60 2y ⇒
11
√ 10 √ √
300 2 + (11 − x) · 6 2y = 60 2y ⇒
11

Entonces, dividiendo por 60 2:

1
5+ (11 − x)y = y ⇒
11
55 + 11y − xy = 11y ⇒
xy = 55.

En resumen, tenemos x + y = 15, xy = 55 y, por lo tanto

x+y 15 3
= =
xy 55 11
1 1 1 1 3
⇒ + = + = .
x y AM AN 11

55. Consideremos la figura siguiente; en ella hemos trazado adicionalmente la altura ME


de 4MDC .
152

B
M

A C
D E

Puesto que M es el punto medio de BC , entonces E es el punto medio de AC , de


manera que EC = AD+DC
2 .
En 4DME, m∠DME = 45◦ y por lo tanto ME = DE (*). Por otra parte, 4BAC ∼
1
1 BC
4MEC ⇒ BC AC = EC , pero MC = 2 BC ⇒ AC = EC ⇒ 2EC = AC ⇒ 2(DC −
MC BC 2

ED) = AC ⇒ 2ED = 2DC − AC ⇒ 2ED = 2DC − (AD + DC ) = DC − AD ⇒


ED = 21 (DC − AD) (*) ME = 12 (DC − AD).

Usando el teorema de Pitágoras en 4MEC tenemos

BC 2

AD+DC 2

DC −AD 2
2 = 2 + 2
BC = AD + DC + DC + AD 2
2 2 2 2

⇒ AD 2 + DC 2 = 12 BC 2 = 12 h2 .

56. Como a, b, c son naturales se tiene que a3 ≥ 3abc es decir a2 ≥ 3bc de donde
2(b + c) ≥ 3bc. Finalmente se tiene que 2c ≥ b(3c − 2).
Si c = 0, a = b y a2 = 2b, de donde b = 0 ∨ b = 2 obteniéndose las tripletas
(2, 2, 0) y (0, 0, 0).
2c
Si c > 0, b ≤ 3c−2 ≤ 2, y hay dos casos:
? b = 0 que conduce a a = c y a2 = 2c y, por lo tanto, c = 0 ∨ c = 2 que conducen
a la nueva tripleta (2, 0, 2).
? Finalmente si b = 1 se sustituye en las ecuaciones originales, lo que conduce a
a3 − c3 = 3ac + 1 y a2 = 2(c + 1). Despejando y sustituyendo se obtiene
 3  2 
3 a2 a
a − − 1 = 3a −1 +1
2 2

Esta última ecuación conduce a

−a6 + 6a4 − 4a3 −12a2 +24a = 0

cuyas soluciones enteras son a = 0, a = 2 lo que lleva a la tripleta (1, 2, 1); el caso
a = 0 se elimina pues llevarı́a a c = −1 que no es un número natural.
m 3n − 1 1 m 1
57. m = 3n − 1 ⇒ = =3− ⇒ < 3 ( pues n > 0) (1)
n n n n
153

Como el menor de los n es 1 y el menor valor de m es 2 (puesto que m, n ∈ Z+ ),


1 1 5 5 5
entonces ≤ y, multiplicando miembro a miembro por se obtiene ≤
mn 2 4 4mn 8
(2)
Ahora, sumando miembro a miembro (1) y (2) se obtiene que:

m 5 5 29
+ <3+ = (3)
n 4mn 8 8
√ √
Como 298 < 14 puesto que 29 < 8 14 (4), entonces de (3) y (4) se concluye que:
m 5 √
+ < 14.
n 4mn
58. Trazamos los segmentos PM y QN y la altura AI del triángulo ABC .

M
P
D N
Q
T
B C
I

Tenemos que PM y QN son paralelos a BC y además PM = 31 BC , QN = 23 BC .


Sea h = AI. Por semejanza de triángulos, las alturas del triángulo APM, el trapecio
PMNQ y el trapecio QNC B son iguales entre sı́ y por lo tanto iguales a 13 h.
1
3 BC · 13 h 1 (BC + 23 BC ) · 31 h
De este modo, (AMP) = = 9 (ABC ); (QNBC ) = =
2 2
5
9 (ABC ).
Ahora, T N es paralela media al lado BC del triángulo BMC y por lo tanto T N = 12 BC .
h0 00
Además 4PMD ∼ 4NT D y, entonces PM = ThN (donde h0 es altura de 4PMD
h0 h00
y h00 altura de 4T DN, h0 + h00 = 31 h). Es decir 1 = 1 ⇒ h0 = 23 h00 ⇒
3 BC 2 BC
h00 + 23 h00 = 13 h ⇒ h00 = 15 h. Además QT = 32 BC − 12 BC = 16 BC . De todo esto
tenemos:
1
6 BC · 13 h 1
(QT P) = = (ABC ),
2 18
1 1
3 BC · 3h 1
(PMT ) = = (ABC ),
2 9
1 1
2 BC · 5h 1
(T DN) = = (ABC ).
2 10
154

De manera que

 
1 5 1 1 1
(DMN) = 1 − − − − − (ABC )
9 9 18 9 10
1
= (ABC ).
15

59. Sean AD, BE y C F las medianas, con BE⊥C F en el punto G. Dibujamos la altura
AK sobre el lado BC .

F E

B C
K D

Entonces

BK + C K BC BC 2GD 2
cot B + cot C = = ≥ = =
AK AK AD 3GD 3

60. En cada caso se borran dos números a y b y se sustituyen por un número ab + a + b.


Observamos que ab + a + b + 1 = (a + 1)(b + 1). Si consideramos el producto
P = (1 + 1)(2 + 1) · · · · · (20 + 1) (el producto de todos los números aumentados en 1),
nos damos cuenta que en cada caso, al quitar a y b y poner ab + a + b, el producto
de todos los números que quedan, aumentados en 1, no va a variar, por lo tanto, al
final, el número que queda en la pizarra es

(1 + 1)(2 + 1) · · · · · (20 + 1) − 1 = 2 · 3 · · · · · 20 · 21 − 1.
155

61. Tenemos que

11 . . 11} 22
| .{z . . 22} 5 = 102n+1 + 102n + · · · + 10n+2 + 2(10n+1 + · · · + 10) + 5
| .{z
n veces n+1 veces
= 10n+2 (10n−1 + · · · + 1) + 20(10n + · · · + 1) + 5
10n − 1 10n+1 − 1
= 10n+2 · + 20 · +5
9 9
102n+2 − 10n+2 + 20 · 10n+1 − 20 + 45
=
9
102n+2 + 10 · 10n+1 + 25
=
9
 n+1 2
10 +5
=
3

Puesto que la suma de los dı́gitos de 10n+1 + 5 es 6, el número 10 3 +5 es un entero


n+1

y ası́, el número dado es un cuadrado perfecto.


Q Q Q
62. Sean a = pai i , b = pbi i , c = pci i , donde pi denota un factor primo de a, b, c
(algunos exponentes pueden ser 0).
Q máx(ai ,bi ) Q mı́n(a ,b )
Como [a, b] = pi , (a, b) = pi i i , etc., tenemos que probar que

2 máx(ai , bi , ci ) − máx(ai , bi ) − máx(bi , ci ) − máx(ci , ai )


=2 mı́n(ai , bi , ci ) − mı́n(ai , bi ) − mı́n(bi , ci ) − mı́n(ci , ai )

Sin pérdida de generalidad podemos suponer, para el ı́ndice i, que ai ≥ bi ≥ ci , en


este caso tendremos 2ai − ai − bi − ai = 2ci − bi − ci − ci y se tiene lo indicado.

NIVEL C
SELECCION
1. (c) Tenemos f (2) + f (1) = 4 ⇒ f (2) = 3, f (3) + f (2) = 9 ⇒ f (3) = 6, f (4) + f (3) = 16
⇒ f (4) = 10; sucesivamente se obtiene que f (5) = 15, f (6) = 21, f (7) = 28, f (8) = 36,
f (9) = 45 y f (10) = 55.

2. (c) Observamos que


√ √ 2
x+2 x−1= x−1+1 y
√ √
x − 2 x − 1 = (1 − x − 1)2 ,
156

por lo tanto
1 1 1 1 2
p √ +p √ = √ + √ = .
x+2 x−1 x−2 x−1 1+ x−1 1− x−1 2−x

3. (d) Sea N el número, tenemos N + 1 = m2 (1) y N


2 + 1 = n2 ⇒ N + 2 = 2n2 (2).
Restando (1) de (2) tenemos que 1 + m2 = 2n2 .
Por otro lado, como N es de dos cifras, su mitad debe ser menor que 50, de manera
que n ≤ 7. Probando sucesivamente con n = 7, 6, 5, . . ., obtenemos n = 5 y, por lo
tanto N = 48 (por (2)). La suma de sus cifras es 12.

4. (a) Llamemos P al centro del hexágono. Como BH = AB = al radio del cı́rculo, se


tiene que (ABH) = (BC H) = (BAP) = 61 (ABC DEF ). Luego, la razón que se pide es
igual a 31 .

F A

B H
E
P

D C

5. (b) Para ser divisible por 30 tiene que ser divisible por 2, 3 y 5. Ası́, en particular el
número es divisible por 10 y por lo tanto necesariamente Z = 0. Hay que ver entonces
cuántos son divisibles por 3, para ello debemos tener X + 2 + Y + Z = 3k (múltiplo de
3), es decir X + Y = 3k − 2, los valores posibles para k son 1, 2, 3, 4, 5, 6 y tenemos

k 1 2 3 4 5 6
X + Y 1 4 7 10 13 16
posibilidades 1 4 7 9 6 3

En total hay 30 números con la condición pedida.

6. (c) Sean α, β, γ cada uno de los ángulos congruentes en que las bisectrices dividen
a los ángulos B, A y C , respectivamente del 4ABC . Entonces 2α + 2β + 2γ = 180,
de donde α + β + γ = 90 (*). Por la condición del enunciado 2α + 2γ = 5(2β) ⇒
α + γ = 5β (**). Ası́ en (*) se tiene que β + 5β = 90 ⇒ β = 15. En 4BOC :

m]BOC = 180 − (α + γ) = 180 − 5β.


(**)

Puesto que β = 15 se sigue que

m]BOC = 180 − 5(15) = 105◦ .


157

7. (d) Tenemos

F (1970) + F (1971) + · · · + F (2001)


=10 · 17 + 10 · 10 + 10 · 19 + 3 · (1 + 3 + 5 + 7 + 9) + 1 = 536.

8. (c) Sean x, y, z las medidas de los lados, entonces xz = 72, yz = 32, xy = 144. Es
decir
xy 144 y
= = 2 ⇒ = 2 ⇒ y = 2z.
xz 72 z

Como yz = 32, entonces 2z 2 = 32 ⇒ z = 4, de aquı́, y = 8 y x = 18.

9. (b) Por las propiedades de los logaritmos tenemos que



3
a 1 1 4 1
logab √ = logab a − logab b = − logab b.
b 3 2 3 2

Como 1 = logab ab = logab a + logab b = 4 + logab b, √ entonces logab b = −3 y,


3
a
sustutyendo en la igualdad anterior, se tiene que logab √ = 43 − 12 (−3) = 176
b
10. (c) De acuerdo con la fórmula de Herón, el área del triángulo de 10, 10, 12 es

A= 16 · 6 · 6 · 4 = 48

De modo que, usando Herón nuevamente, en el otro triángulo tenemos


r
x  x  x   x
48 = 10 + 10 −
2 2 2 2
r
x x 2
48 = 100 − ⇒
2r 4
x2
96 = x 100 −
4
 
2 x2
9216 = x 100 −
4
36864 = 400x 2 − x 4
0 = x 4 − 400x 2 + 36864

Haciendo x 2 = y, la última ecuación se convierte en y2 −400y+36864 = 0. Resolviendo


se tiene y = 144, y = 256 y por lo tanto x = 16, x = 12. El dato que andamos
buscando es x = 16.
158

11. (d) La entrada, en la casilla número k, de la fila superior es ak = 4(k − 1) + 1000; la


entrada, en la casilla número k, de la fila inferior es bk = 7(k − 1) + 20. Queremos
ver cuándo ak < bk , es decir

4(k − 1) + 1000 < 7(k − 1) + 20 ⇒ 980 < 3(k − 1) ⇒ 327, 66 < k

De manera que en la primera casilla en la cual el número inferior es mayor al superior


es en la número 328. Ası́, n = 328 y la suma de sus cifras es 13.

12. (a) Como ABC D es un cuadrado con AD = 10, entonces MD = 5 (pues M es punto
medio de C D). Si RA = x, entonces MR = 4x y, por lo tanto,
√ MA = 5x. Por Pitágoras
2
en el triángulo MDA, tenemos 25x = 125, entonces x = 5. La distancia que andamos
buscando es la longitud del segmento RN en la figura adjunta.

D A
R

M Q
P

C B
N

Si trazamos el segmento MQ, perpendicular a AB, tenemos que Q es el punto medio


de AB. Si P es el punto intersección entre MQ y RN entonces los triángulos MAQ y
MRP son semejantes y, por lo tanto, 105 = MPRP ⇒ MP = 2RP. Ası́, por Pitágoras en
2
el triángulo MPR, se tiene 5RP = 80 ⇒ RP = 4 y, por lo tanto, RN = 9.


   12m   2√ m3
 √n 9+m 2 9
13. (d) f n2 + 1 =k ⇒ f = f +1 = k 2.
m2 m2

14. (a) Si E es tal que AE = BE, entonces E se encuentra sobre la mediatriz del segmento
AB; la intersección entre esta mediatriz y la recta m es un único punto. Si E es tal
que BA = BE, entonces E se encuentra sobre un cı́rculo de centro B y radio BA, por
las condiciones del problema, la intersección entre la recta m y este cı́rculo son dos
puntos. De igual forma, si E es tal que BA = AE, entonces E se encuentra sobre un
cı́rculo de centro A y radio AB, la intersección entre este cı́rculo y la recta m son dos
puntos. Por las condiciones del problema, estos cinco puntos son distintos entre sı́.
159

A m
B

15. (d) Tracemos OC , que resulta ser la hipotenusa de los triángulos rectángulos C AO y
C BO.

O C

Estos
√ triángulos son congruentes y de ángulos 30◦ , 60◦ , 90◦ . Luego, como AO = OB =
3, √se tiene que BC = C A = 3 y entonces el área buscada es (AOBC ) = 2 (C AO) =

2 · 3 2 3 = 3 3.

16. (b) Si x ≥ 4, entonces 23+x es entero mientras que 23−x no es entero, por lo que la
suma no puede ser un número entero. Concluimos que x ≤ 3. Análogamente, x ≥ −3.
Para −3 ≤ x ≤ 3, ambas potencias de 2 son enteras. Como 65 es impar, entonces una
de las potencias de 2 tiene que ser impar, es decir, tiene que ser 20 = 1 y la otra
26 = 64. Luego, las únicas posibilidades son 3 − x = 0 y x + 3 = 6 ó x + 3 = 0 y
3 − x = 6, es decir x = 3 ó x = −3.

17. (b) Si unimos los puntos A, B y C con el punto O, donde se intersectan los tres
cı́rculos, tenemos 3 cuadriláteros concı́clicos, como se muestra en la siguiente figura.
160

A O
w

u
C

Si α = m∠AOC , β = m∠AOB y γ = m∠BOC , entonces u + α = 180◦ , v + β = 180◦


y w + γ = 180◦ , por lo tanto

u + α + v + β + w + γ = 3 · 180◦ .

Luego

u + v + w = 3 · 180◦ − (α + β + γ)
= 3 · 180◦ − 360◦ = 180◦ .

18. (d) Como el hexágono es regular podemos dividirlo en 12 triángulos rectángulos iguales
como se muestra en la figura, entonces el área estará dada por la suma de las áreas
de los 12 triángulos, es decir,
H = 12 · h,
donde h es el área de uno de los triángulos rectángulos.

B
A

Tenemos también que


H
H = 6 · h + 4 · (ABC ) = + 4 · (ABC ) .
2
Luego, 21 H = 4 · (ABC ). Es decir,
1
(ABC ) = H.
8
161

19. (b) De la primera ecuación, y = 2 − x. Sustituyendo en la segunda: x(2 − x) − z 2 = 1


⇒ z 2 = −(x − 1)2 . Como el lado izquierdo es mayor o igual que 0 y el lado derecho
es menor o igual que cero, la única posibilidad de que sean iguales es que ambos
sean 0, de ahı́ tenemos que z = 0 y x = 1 y, por lo tanto, y = 1. Solo una tripleta
satisface el sistema.
20. (a) La suma de los ángulos internos de un hexágono es 720◦ , si el hexágono es
equiángulo entonces todos sus ángulos son iguales y por lo tanto miden 7206 = 120◦

cada uno. Prolongando en ambas direcciones los lados de medida x, medida 10 y


medida 12, obtenemos tres triángulos: uno sobre el lado de medida y, otro sobre el
lado de medida 6 y otro sobre el lado de medida 14 y también un triángulo grande
ABC como se muestra en la figura.

6 B
10 6
6
y
12
A y

y 14

x 14

14
C

Como cada ángulo interno del hexágono es de 120◦ , entonces cada ángulo externo
mide 60◦ , de modo que tanto los tres triángulos pequeños como el triángulo grande
son equiláteros. Entonces tenemos 14+x +y = 14+12+6 = 32, con lo que x +y = 18
y, entonces el perı́metro del hexágono es x + y + 10 + 6 + 12 + 14 = 18 + 42 = 60.
21. (c) Tenemos que
N = 1 + 2 + 3 + · · · + 1011

1011 1011 + 1
=
2
1011  11 
= 10 + 1
2  
= 210 511 1011 + 1 .

Como 511 y 1011 + 1 son impares, entonces el factor 2 aparece 10 veces.


22. (a) Tenemos que x 2n+1 − 1 = 2r y
 
x 2n+1 − 1 =(x − 1) x 2n + x 2n−1 + · · · + x + 1 .
162

Luego  
2r = (x − 1) x 2n + x 2n−1 + · · · + x + 1 ,

lo cual es imposible ya que si x > 1 y n > 1, el número x 2n + x 2n−1 + · · · + x + 1 es


impar y mayor que 1.

23. (a) De las condiciones dadas en el problema se puede construir la siguiente tabla

Caso América Asia Europa Africa


A 4 2 5 4
B 2 4 3 6
C 3 3 4 5
D a b c 4

En el caso D, por ser a + b = 6, entonces a = 5, b = 1 ó a = 1, b = 5 ó a = 2, b = 4


ó a = b = 3. Las dos últimas posibilidades no se dan puesto que habrı́an continentes
representados por el mismo número de delegados. Si a = 5, b = 1, entonces c = 6,
en cuyo caso, el número total de delegados serı́a 16. Si a = 1, b = 5, entonces c = 2
y el número total de delegados serı́a 12. Ası́, el caso D no es posible. Ası́ mismo, los
casos A y C no podrán darse porque habrı́an continentes representados por el mismo
número de delegados. La única situación posible, es por tanto B.

24. (a) El cuadrilátero AQPR es un paralelogramo, por lo tanto AQ = RP y QP = AR,


m]QPB = m]AC B y m]RPC = m]ABC (correspondientes entre paralelas). Luego
los triángulos QBP y RPC son triángulos isósceles, ya que 4ABC es isósceles.
Entonces QB = QP y RP = RC . Si AQ = PR = b y QP = AR = a, entonces
QB = 17 − b, QP = a, puesto que QB = QP, se sigue que 17 − b = a, es decir,
a + b = 17, entonces el perı́metro del cuadrilátero AQPR es 2(a + b) = 2(17) = 34
cm .

25. (d) Queremos que la suma sea máxima, entonces colocamos 9 en el centro porque
ahı́ aporta a cuatro segmentos. Luego colocamos a su alrededor al 8, 7, 6 y 5:

5
8 9 6
7

El 4 aporta más a la suma si se coloca entre el 8 y el 7.Luego se acomodan los


restantes.
2 5 1
8 9 6
4 7 3
Si sumamos los valores de cada segmento obtenemos 134.
163

26. (a) Hagamos 2p + 1 = m3 ⇒ 2p = m3 − 1 ⇒ 2p = (m − 1)(m2 + m + 1) ⇒


m−1 2
p= (m + m + 1) (*)
2

Como m3 es impar, entonces m es impar y por lo tanto m−1


2 es un entero. Pero como
2
p es primo, 2 = 1 o m + m + 1 = 1. Pero lo último no puede ser, por lo tanto
m−1

m − 1 = 2, es decir, m = 3 y 2p + 1 = 27 ⇒ p = 13. Solo hay un número con la


propiedad indicada.

27. (c) Tenemos x+r


t
= 2 ⇒ 2t = x + r; r−xt
= 3 ⇒ 3t = r − x ⇒ 2t + 3t = 2r ⇒ 56 t = 2r
⇒ r = 125 t ⇒ r < t. Por otra parte, como r − x = 3t y todos son positivos entonces
r − x > 0 ⇒ r > x. Concluimos que

x < r < t.

28. (b) Coloquemos los números en forma triangular

1
2 4
5 7 9
10 12 14 16
17 19 21 23 25
..
.
1850 · · · · · · · · · 1936
1937 · · · 2023 2025
2026 · · ·

Vemos que el último número en cada fila es un cuadrado perfecto, ası́, el número par
más cercano a 2000 que aparece en la colección es 2026.

29. (c) Los triángulos ABC y C DP tienen la misma altura sobre los lados iguales AB y
C D, por lo tanto tienen la misma área. Las demás opciones solo ocurren en casos
particulares.
p √ √ √
30. (b) Escribamos 2 + 3 = x + y. Elevando al cuadrado e igualando términos
semejantes se tiene 
x + y = 2 (1)
xy = 43 (2)

Al despejar y de (1) y sustituir en (2) se obtiene 4x 2 − 8x + 3 = 0. Como x > 0, resulta


que x = 23 y por lo tanto y = 12 . Entonces
q r r √ √
√ 3 1 6 2
2+ 3= + = + .
2 2 2 2
164

31. (b) Llamemos con h el lado del triángulo equilátero: 2(AN)2 = h2 , (MD)2 +1 = h2 (por
Pitágoras). Además
√ AN = 1√− DM. De todo esto, (DM)2 − 4DM + 1 = 0 y, entonces √
DM = 2 ± 3. Pero 2 + 3 no vale (es mayor que 1), entonces DM = 2 − 3.
Tenemos
(C MN) = 1 − 2(C BN) − (AMN) (*)

(1−DM)2

Pero (C BN) = DM
2 y (AMN) = 2 . Sustituyendo en (*): (C MN) = 2 3 − 1.

32. (a) En el siguiente gráfico se observa que con 6 personas designadas por A, B, C , D,
E, F y con 4 cajas de galletas 1, 2, 3, 4, se puede resolver el problema.

A B C

1 2 3 4

D E F

Además, éste es el número mı́nimo de personas que pueden satisfacer las condiciones
del problema, ya que si de cada caja comen 3 personas y cada persona come de dos
cajas distintas, necesariamente dicho número es múltiplo de 3 · 2 = 6.

33. (d) (AC DE) = 12 (24 + 15) · 12 = 234 cm2 , (OBC D) = 234 − (DEA) − (AOB). Los
triángulos 4EOD, 4AOB son semejantes, por el postulado ángulo–ángulo (]EOD
y ]AOB son opuestos por el vértice, ]EDA y ]DAC son alternos internos entre
paralelas). Sea h la medida de la altura del triángulo EDO, trazada desde el vértice
O, la medida de la altura de 4AOB, trazada desde el vértice O es 12 − h. De la
semejanza de los triángulos se tiene

15 h 16
= ⇒ 12 − h =
12 12 − h 3

Ası́, (DEA) = 90 cm2 , (AOB) = 32 cm2 , (OBC D) = 112 cm2 .

34. (c) El radio del segundo cı́rculo es igual a la apotema del triángulo equilátero inscrito
en el primero, esto es, R2 , el radio del tercer cı́rculo es igual a la apotema del segundo
triángulo equilátero, es decir 2R2 y ası́ sucesivamente, el radio del n−ésimo cı́rculo es
R
2n−1
y por lo tanto su área es
 2  n−1
R 21
π = πR .
2n−1 4

35. (c) 22n+1 ·52n+3 −1 = (2·5)2n+1 ·25−1 = 102n+1 ·25−1 = 25 |0 ·{z


· · }0 −1 = 24 |9 ·{z
· · }9 .
2n+1 ceros 2n+1 nueves
Luego la suma de los dı́gitos es (2n + 1) · 9 + 4 + 2 = 18n + 15.
165

36. (b) A partir del enunciado tenemos que a + b = 3c y c + d = 3a y, por lo tanto

b + d = 2(a + c)
b − d = 4(c − a)

Además, a2 −3ac−8d = 0 y c2 −3ac−8b = 0. Restando estas dos últimas igualdades


y utilizando las relaciones de arriba se tiene que

a2 − c2 = 8(d − b) ⇒ (a − c)(a + c) = 32(a − c).

Como a 6= c, se concluye que a + c = 32. Por lo tanto, b + d = 2(a + c) = 64. De


modo que a + b + c + d = 96.
37. (b) AB 2 + BC 2 = AC 2 , entonces AB 2 + 64 = 100 ⇒ AB = 6. De aquı́, el perı́metro
del cı́rculo pequeño es 2 y entonces su perı́metro es 4π.
38. (b) Tenemos que
q  
1
2 = cos A + cos B = 2 cos A+B
2 cos A−B
2
q  
3
2 = sin A − sin B = 2 sin A−B
2 cos A+B
2

Entonces, elevando al cuadrado:


 
1
2 = 4 cos2 A+B
2 cos2 A−B
2
 
3
2 = 4 sin2 A−B
2 cos2 A+B
2

Sumando ambas relaciones:


h 2  i
2 = 4 cos2 A+B
2 sin A−B
2 + cos2 A−B
2

  q
Por lo tanto cos2A+B
2 = 21 ⇒ cos A+B 2 = ± 12 . El valor negativo se descarta
puesto que 0 ≤ A+B ≤ 90◦ y, entonces, su coseno es positivo. En este caso, tenemos
 q 2
cos A+B
2 = 12 ⇒ A+B 2 = 45 y, por lo tanto, A + B = 90 .
◦ ◦

39. (b) Considere la figura:

x
D C
a
E `
` x
`
x−a
a
A B
x−a F
166

√ √
Tenemos (C EF ) = 3 = 43 ` 2 , de donde ` = 2 (` es el lado del triángulo equilátero).
Además 4EDC ∼ = 4F BC (hipotenusa–cateto). Luego
ax √ ax (x − a)2 √ (x − a)2
(ABC D) = + 3+ + = ax + 3 + .
2 2 2 2
Por Pitágoras a2 + x 2 = 4 y 2(x − a)2 = 4 ⇒ 2 2
√ x −2 2ax + a = 2 y de esto concluimos
que ax = 1 y, por lo tanto (ABC D) = 2 + 3 cm .
40. (d) Sea A el área sombreada, entonces
π π π π
A = C A2 − C D 2 − DA2 = (C A2 − C D 2 − DA2 ),
8 8 8 8
pero C A = C D + DA, entonces
π π
A = [(C D + DA)2 − C D 2 − DA2 ] = C D · DA
8 4
Además sabemos que el ángulo C BA es recto (está inscrito en una semicircunferencia)
y por lo tanto BD es altura sobre la hipotenusa del triángulo C AB, entonces BD 2 =
C D · DA. Luego A = π4 BD 2 .
41. (a) 72 = 23 · 32 , entonces si N es divisible por 72, debe serlo por 8 y por 9 simultánea-
mente; es decir, el número formado por sus tres dı́gitos finales debe ser divisible por
8 y la suma de sus dı́gitos debe ser divisible por 9. Entonces b = 3 ó b = 7 (para
que el número sea divisible por 8). Como la suma de los dı́gitos debe ser múltiplo de
9 entonces a + b = 4 ó a + b = 13. Todo esto es posible solo si a = 6 y b = 7
ó a = 1 y b = 3. Solo hay dos números: 46872 y 41832.
42. (b) Observe lo siguiente:
sin 1◦ sin(k + 1 − k)◦
=
cos k ◦ cos(k + 1)◦ cos k ◦ cos(k + 1)◦
sin(k + 1)◦ cos k ◦ − cos(k + 1)◦ sin k ◦
=
cos k ◦ cos(k + 1)◦
= tan(k + 1)◦ − tan k ◦

De esto se sigue que


S = tan 1◦ − tan 0◦ + tan 2◦ − tan 1◦ + + · · · + tan 1999◦ − tan 1998◦
= tan 1999◦ − tan 0◦ = tan 1999◦ .

43. (b) El ]ABC es inscrito y mide la mitad del arco AC que intercepta. El ]RAE es
semiinscrito y su medida también es la mitad del arco AC . Luego ]ABC ∼
= ]RAE. Al

ser DE k T , por ángulos alternos internos entre paralelas, ]AED = ]RAE. Ası́, por
ángulo–ángulo, se sigue que 4ADE ∼ 4ABC . De la semejanza se sigue que
AD AE 6 5
= ⇒ = ⇒ BD = 4.
AC AB 12 6 + BD
167

44. (c) Sea r el diámetro. Por ser L tangente en D se tiene que AD ⊥ L. Sea h =
BD y m = C D. Como AD es diámetro entonces BD es altura de 4ADP (sobre la
hipotenusa). Igualmente C D es altura sobre la hipotenusa en 4ADQ. De todo esto
r 2 = h2 + (46, 08)2 y h2 = 46, 08 · 3, 92; luego r 2 = 2304. Además r 2 = (28, 8)2 + m2
y por lo tanto 2304 = (28, 8)2 + m2 ⇒ m2 = 1474, 56. Por otra parte, m2 = 28, 8 · C Q
y, entonces C Q = 51, 2.

P
B

h
A r D
m

C
Q

45. (c) Observe que m]AC B = 45◦ , por lo que 4ABC es rectángulo isósceles
√ y QC = QT
(donde T es el punto intersección de BC y PQ). La diagonal AC mide
√ 2 centı́metros
y AQ = 1 pues √ corresponde a la rotación de AD. Ası́, QC = 2 − 1. El área de
4T QC es (3 − 2 2)/2. De modo que

(ABT Q) = (ABC D) − (AC D) − (QT C ) = 2 − 1.

46. (a) El cı́rculo pequeño está inscrito en un triángulo equilátero. La mediana AD de ese
triángulo es bisectriz del ángulo y mide 2R (R es el radio del cı́rculo mayor). El punto
de intersección de las medianas en cualquier triángulo se encuentra a una tercera
1 1
parte de la base. Ası́, si O es el centro del cı́rculo menor, ODAD = 3 , es decir, 2R = 3 (r
r

es el radio del cı́rculo menor). Por lo tanto r = 2R 3 y el área del cı́rculo pequeño es
 2
2 2R 4
πr = π = πR 2 ,
3 9
pero πR 2 = 1 pues es el área del cı́rculo mayor. De modo que el área del cı́rculo
menor es 49 .
47. (d) Sea a = 932 − 65 · 930 + 3n , entonces a = (4 · 915 )2 + 3n . Sea k ∈ N tal que a = k 2 ,
luego (4 · 915 ) + 3n = k 2 .
Por lo tanto (k − 4 · 915 )(k + 4 · 915 ) = 3n . Luego, existe p ∈ {0, 1, 2, ..., n} tal que
k + 4 · 915 = 3p
k − 4 · 915 = 3n−p
168

con p > n − p. Luego, 8 · 915 = 3n−p (32p−n − 1), y de esto se sigue que n − p = 30
y 2p − n = 2. De aquı́, n = 62 y p = 32.
48. (d) Sea ABC D cualquier cuadrilátero. X , Y , Z , W , los puntos medios de AB, BC , C D
y DA respectivamente. Los segmentos X Y y W Z son paralelos a la diagonal AC y
también W X k Z Y , es decir X Y Z W es un paralelogramo. Sean H1 y H2 los pies de
las alturas trazadas desde B hasta AC y X Y respectivamente y sean H3 y H4 los
pies de las alturas trazadas desde D hasta AC y W Z respectivamente y sean I, J las
intersecciones de AC con X W y Y Z respectivamente.

A
X

B
I H1

W H2
H3
Y
H4
J
D C
Z

Tenemos:
AC · BH1
(ABC ) = ,
2
AC · DH3
(ADC ) =
2
AC · (BH1 + dH3 )
(ABC D) = .
2
Además H1 H2 = H2 B y H3 H4 = H4 D, por semejanza de triángulos, y W Z = X Y .
(AC − X Y ) · H1 H2
(AIX ) + (C F Y ) =
2
(AC − X Y ) · H3 H4
(AIW ) + (C F Z ) =
2
X Y · BH2
(X Y B) = y
2
W Z · DH4
(W Z D) = .
2
Entonces
 
AC AC − X Y XY
(X Y Z W ) = [BH1 + DH3 ] − (H1 H2 + H3 H4 ) + (BH2 + DH4 )
2 2 2
AC 1
= [BH1 + DH3 ] = (ABC D)
2 2
169

De esta forma, el área del cuadrilátero inscrito es la mitad del área del cuadrilátero
original y ası́ sucesivamente, de modo que el área de An Bn Cn Dn es (ABC D)/2n .

DESARROLLO
q q
1. Observamos que (x + 1) = |x + 1| y (x − 1)2 = |x − 1|, ası́, si x 6= 0:
2

1
f (x) = (|x + 1| − |x − 1|)
2x

De acuerdo con la definición de valor absoluto tenemos que




 1

 si x > 1

 x
f (x) = 1 si − 1 ≤ x ≤ 1



 1

 − si x < 1
x

De modo que el rango de f es el intervalo de números reales ]0, 1].

2. Sean di = ai+1 − ai las diferencias, i = 1, . . . , 108. Por un lado tenemos que

d1 + d2 + · · · + d108 = a109 − a1
≤ 1998 − 1 (1)
= 1997.

Ahora, sea d10 ≤ d20 ≤ . . . ≤ d108


0 las diferencias ordenadas. Si suponemos que ningún

valor se repite más de tres veces entonces, d10 , d20 , d30 valen al menos 1, d40 , d50 , d60
valen al menos 2, ..., d106
0 , d0 , d0
107 108 valen al menos 36, por lo tanto

d1 + d2 + · · · + d108 = d10 + d20 + · · · + d108


0

≥ 3 (1 + 2 + · · · + 36) = 1998.

Esto contradice (1), por lo que al menos una de las diferencias debe repetirse 4 veces
o más.
Para la segunda parte del problema considere a1 = 1 y ai = a1 + d1 + · · · + di−1 ,
donde d1 = d2 = d3 = 1, d4 = d5 = d6 = 2, . . ., d106 = d107 = d108 = 36.

3. Podemos escribir sin 3x = 3 sin x − 4 sin3 x, cos 2x = 1 − 2 sin2 x. Ası́, la desigualdad


dada se puede escribir como
   
4 3 sin x − 4 sin3 x + 5 ≥ 4 1 − 2 sin2 x + 5 sin x,

o bien
16 sin3 x − 8 sin2 x − 7 sin x − 1 ≤ 0.
170

Si escribimos y = sin x, entonces la desigualdad se convierte en

16y3 − 8y2 − 7y − 1 ≤ 0 ⇒ (y − 1) (4y + 1)2 ≤ 0.

Ahora, como y = sin x, entonces y < 1 pues sin x < 1 para todo x, por lo que la
última desigualdad es válida y por lo tanto lo es la desigualdad inicial para todo x.

4. Observamos que los términos de la secuencia (a) son de la forma

an = 3 + 4 (n − 1) ,

para n = 1, . . . , 287. Los términos de la secuencia (b) son de la forma

bk = 2 + 7 (k − 1) ,

para k = 1, . . . , 287. Ası́, debemos encontrar los números n y k para los cuales
an = bk . Tenemos

an = bk ⇒
3 + 4(n − 1) = 2 + 7 (k − 1) ⇒
4n + 4 = 7k.

De aquı́ tenemos que k tiene que ser múltiplo de 4, es decir, 4 = 4s; los valores de s
serán 1, 2, . . . , 71 pues 1 ≤ k ≤ 287. Ahora, si k = 4s, entonces

4 (n + 1) = 7 · 4s,

o sea n = 7s − 1. Dado que 1 ≤ n ≤ 287, entonces los valores admisibles para s son
únicamente 1, 2, . . . , 41. Ası́, hay 41 números que pertenecen a ambas sucesiones.

5. Consideremos la figura:

A D

B C

Al trazar la diagonal AC vemos que los ángulos ∠BK C y ∠BAC son inscritos y
subtienden el mismo arco, por lo tanto tienen la misma medida, es decir m∠BAC =
171

α. Si el radio de la circunferencia es R, tenemos que la hipotenusa del triángulo


rectángulo BAC es 2R y el cateto adyacente a ∠BAC es a, por lo tanto
a
cos α = ,
2R
es decir
a
R= .
2 cos α
6. En primer lugar vemos que x = 0, y = 0 es una solución. Por otra parte, x > 0, y > 0

y como x + xy > 0, resulta que x > y. Obteniendo raı́z cuadrada en la segunda

ecuación tenemos x + y = 2(x − y). Despejando y y racionalizando:

( 2 − 1)x √
y= √ = ( 2 − 1)2 x.
2+1
Sustituyendo en la primera ecuación tenemos
√ √ √ √
x − ( 2 − 1) x = x + ( 2 − 1)x ⇒

√ √ √ √ 2− 2
(2 − 2) x = 2x ⇒ x = √
2
√ √
De aquı́, x = 3 − 2 2 y y = 17 − 12 2.

7. Construyamos la figura correspondiente.

K
B C

a−y
P
x
N M a−x S
y
R
A L D

Sea a la medida del lado del cuadrado. Sea x la distancia desde M al lado AB, y sea
y la distancia de M al lado AD. Tenemos entonces que la distancia de M a BC es
a − y y la distancia de M a C D es a − x. Sea P el punto intersección de C D con
←→
la recta indicada y tracemos el segmento AS paralelo a NP, entonces la tangente
←→ ← →
del ángulo ∠SAD es 31 . Sea R el punto intersección de NP y AD y sean K y L
puntos en BC y AD respectivamente, tales que M ∈ K L y K LkC D. Como PD = 2SD,
concluimos que RD = 2AD = 2a (por semejanza de triángulos), por lo que

RL = 2a − (a − x) = a + x.
172

Además tenemos que tan (∠MRL) = ML LR , de donde y = 3 . Ası́, las medidas de los
a+x
2a−x
lados, en el orden indicado, son: x, a+x
3 , a− 3 = 3
a+x
y a − x.
Es fácil ver que esto cuatro números forman una progresión aritmética.

8. Es claro que las soluciones deberán satisfacer las condiciones x > 0, x 6= 1. Utilizando
las propiedades de los logaritmos tenemos que:

1 loga x + 1
logx (ax) = 1 + logx a = 1 + =
loga x loga x
loga (ax) = 1 + loga x
   
1 1
loga2 = loga2 a−1 = −
a 2

De modo que la ecuación se puede escribir como


2
loga x + 1 1
=− .
loga x 2

Es decir,
2 5
loga x + log x + 1 = 0.
2 a

Escribiendo y = loga x, tenemos la ecuación cuadrática y2 + 52 y + 1 = 0, cuyas


soluciones son y = − 12 , y = −2. Si y = − 12 , entonces loga x = − 12 ⇒ x = a−1/2 .
Si y = −2, entonces loga x = −2 ⇒ x = a−2 . Ası́, la ecuación tiene dos soluiones
x = √1a y x = a12 .

9. Supongamos que el cilindro de radio r de la base y altura h está inscrito en el cono.


De la semejanza de los triángulos AOS y BO1 S, en la siguiente figura, se deduce que
R (H − h)
r= .
H

O1 B

O A
173

El área lateral del cilindro es S = 2πrh, es decir,


2πR
S= h (H − h) ⇒
H
−2πR 2
S= h + 2πRh.
H
Vemos que S es una función cuadrática de h, en la que el coeficiente del término
cuadrático es negativo, por lo tanto tiene un valor máximo que se encuentra en el
−2πR
vértice de la parábola correspondiente. La abscisa de dicho vértice es h = =
2 · −2πR
H
H
. Ası́, el cilindro inscrito que tiene la mayor superfice lateral es el que tiene la altura
2
igual a la mitad de la altura del cono.

10. Evidentemente la pareja (0, 0) es una solución y es la única para la cual alguna de
las dos incógnitas es 0. Supongamos entonce ahora que x 6= 0 y y 6= 0. Multiplicando
la primera ecuación por x y la segunda por y el sistema se convierte en

yx = x 4 − 3x 2
xy = y4 − 3y2

Es decir,

x 4 − 3x 2 = y4 − 3y2
x 4 − y4 − 3(x 2 − y2 ) = 0
  
x 2 − y2 x 2 + y2 − 3 = 0
 
(x − y) (x + y) x 2 + y2 − 3 = 0

Si x − y = 0, entonces x = y. Sustituyendo en la primera de las ecuaciones del


sistema se tiene x = x 3 − 3x ⇒ x 3 − 4x = 0 ⇒ x(x − 2)(x + 2) = 0 ⇒ x = 0 ó x = 2
ó x = −2. Además de la solución (0, 0) se obtiene de aquı́ dos soluciones más: (2, 2)
y (−2, −2).
Si x + y = 0, entonces y = −x. Sustituyendo en la primera de las ecuaciones del
3 3
√  √ 
sistema tenemos que −x = x − 3x ⇒ x − 2x = 0 ⇒ x(x − 2) x + 2 = 0 ⇒
√ √
x = 0 ó x = 2√ó x = √ − 2. Además
√ √ de la solución (0, 0) se obtiene de aquı́ dos
soluciones más: ( 2, − 2) y (− 2, 2).
Si x 2 + y2 − 3 = 0 (*), entonces x 2 − 3 = −y2 (**). La primera ecuación del sistema se
puede escribir como y = x(x 2 − 3), entonces, por (**), se tiene y = −xy2 , como x 6= 0,
2 1
y 6= 0, se tiene 1 = −xy ⇒ y = −1 x . Sustituyendo en (*) se tiene x + x 2 − 3 = 0
4 2 1 1

⇒ x − 3x + 1 = 0. Las soluciones de esta última ecuación son: x = − 2 + 2 5,
√ √ √
x = − 21 − 12 5, x = 12 + 21 5, x = 21 − 12 5 y como y= −1 x , a partir de esto se
1 1
√ 1 1

obtienen otras cuatro soluciones para el sistema original: − 2 + 2 5, − 2 − 2 5 ,
174
 √ √   √ √   √ √ 
− 12 − 1
2 5, − 12 + 21 5 , 21 + 12 5, 12 − 12 5 y 21 − 12 5, 12 + 12 5 .

11. Si f (x1 ) = x2 , f (x2 ) = x3 , f (x3 ) = x1 , entonces


1
= x2 (1)
ax1 + b
1
= x3 (2)
ax2 + b
1
= x1 (3)
ax3 + b

Sustituyendo (2) en (3) tenemos


ax2 + b
x1 = (4)
abx2 + b2 + a

Sustituyendo (1) en (4) obtenemos

abx1 + b2 + a
x1 =  ⇒
a2 + ab2 x1 + b3 + 2ab
  
0 = a2 + ab2 x12 + b3 + ab x1 − b2 + a
Realizando las sustituciones de otra manera también obtenemos
  
0 = a2 + ab2 x22 + b3 + ab x2 − b2 + a
  
0 = a2 + ab2 x32 + b3 + ab x3 − b2 + a
Es decir, los números x1 , x2 y x3 deben ser soluciones de la ecuación
     
0 = a2 + ab2 x 2 + b3 + ab x − b2 + a

Pero, dado que esta ecuación es de segundo grado, la única manera en que pueden
existir tres números distintos que satisfagan las condiciones dadas es que los tres
coeficientes sean iguales a 0, es decir

0 = a2 + ab2 ,
0 = b3 + ab,
 
0 = − b2 + a .

Por lo tanto a = −b2 . Como el denominador en f (x) no puede ser 0, entonces con-
cluimos que a y b no pueden ser 0 al mismo tiempo. Ası́, las condiciones sobre a y b
que resuelven el problemas son ab 6= 0 y a = −b2 .
√ √
y b√son números positivos entonces  2 a2 + b2 ≥ a + b, esto es cierto puesto
12. Si a √
que 2 a2 + b2 ≥ a + b ⇔ 2 a2 + b2 ≥ (a + b)2 ⇔ (a − b)2 ≥ 0.
175

G
D C

A B
E


Ahora, por el teorema de Pitágoras, EF = BE 2 + BF 2 y por la desigualdad indicada
antes tenemos √
2EF ≥ BE + BF .

De modo análogo se obtienen las desigualdades



2F G ≥ C F + C G,

2GH ≥ DG + DH,

2HE ≥ AH + AE.

Si P es el perı́metro de EF GH y sumando estas cuatro desigualdades resulta que



2P ≥ AE + EB + BF + F C + C G + GD + DH + HA = 1 + 1 + 1 + 1 = 4,

y por lo tanto P ≥ 2 2.

13. Primero vemos que si uno de ellos es negativo la desigualdad es evidente: digamos
b < 0, entonces, como a + b = 2, entonces a = 2 − b > 2, luego, a4 + b4 > 24 + b4 =
16 + b4 > 2; lo mismo sucede si a < 0.
Veamos ahora el caso en que a ≥ 0 y b ≥ 0. Como a + b = 2, entonces (a + b)2 = 4.
La desigualdad media aritmética-media
√ geométrica aplicada a los números a2 y b2
2 2 2 2
nos dice que a +b ≥ a2 b2 , es decir a +b ≥ ab ⇒ 2ab ≤ a2 + b2 . Tenemos
2 2 
4 = (a + b)2 = a2 + b2 + 2ab ≤ a2 + b2 + a2 + b2 = 2 a2 + b2 , es decir,
2
2 ≤ a2 + b2 . De aquı́ se tiene que 4 ≤ a2 + b2 .
Aplicando nuevamente la desigualdad media aritmética-media geométrica, ahora a los
4 4 
2 + b2 2 =
números a4 y b4 obtenemos que a2 b2 ≤ a +b
2 . De esto tenemos: 4 ≤ a
a4 + b4 + 2a2 b2 ≤ a4 + b4 + a4 + b4 = 2 a4 + b4 , es decir, 2 ≤ a4 + b4 , como
querı́amos probar.

14. Restando las ecuaciones dadas tenemos que

3p − 4q = 1.
176

Vemos que una solución para esta última ecuación es p = −1 y q = −1, pues

3(−1) − 4(−1) = 1.

Tenemos entonces que

3p − 4q = 3(−1) − 4(−1)
3(p + 1) = 4(q + 1)

Luego 3 divide a q + 1 y 4 divide a p + 1, es decir, q + 1 = 3k, con k entero, entonces


q = 3k − 1. Como x = 4q − 2, entonces x = 12k − 2. De modo que los x que cumplen
lo pedido son todos los de la forma 12k − 2 con k entero.

15. Del sistema dado se deduce que


2
1 + xi−1
xi = ,
2
por lo tanto xi ≥ 0. Multiplicando todas las ecuaciones del sistema tenemos
    
1 + x12 1 + x22 · · · 1 + x1999
2
= 21999 · x1 x2 · · · x1999 (1)

De la desigualdad entre las medias aritmética y geométrica tenemos que


q
1 + x12
≥ 1 · x12 ⇒ 1 + x12 ≥ 2x1 .
2
En donde la igualdad se cumple solo si x1 = 1. De igual manera tenemos

1 + x22 ≥ 2x2
···
2
1 + x1999 ≥ 2x1999

De donde:
    
1 + x12 1 + x22 · · · 1 + x1999
2
≥ 21999 · x1 x2 · · · x1999 (2)

en donde la igualdad se cumple solo si x1 = x2 = · · · = x1999 = 1. Sin embargo,


debido a (1) la igualdad es obligatoria. Por lo tanto, la única solución del sistema es
x1 = x2 = · · · = x1999 = 1.

16. Sean las circunferencias de centro Oi y radio ri , con i = 1, 2, ..., 6 y sea A el punto
que ellas tienen en común. De los ángulos Oi AOj considere el menor, digamos que es
Oa AOb . Entonces este ángulo es menor o igual a 60◦ y Oa Ob es menor que el mayor
de AOa y AOb , si, por ejemplo, AOb es el mayor, entonces la circunferencia buscada
es la de centro Ob y radio rb .
177

17. De la segunda ecuación tenemos z = a − x − y. Sustituyendo en la primera

x 2 + y2 = a − x − y ⇒
x 2 + x + y2 + y = a ⇒
   
1 2 1 2 1
x+ + y+ =a+
2 2 2

1
De la última ecuación vemos que si a < −1
2 entonces a +< 0 y no hay solución,
2
1
2 2
si a > hay infinitas soluciones y si a =
2 − 12entonces x + 12 + y + 12 = 0 y
hay una única solución: x = − 21 , y = − 21 , z = a − x − y = − 12 + 12 + 12 = 21 .
18. Sean Sc , Sv y Sa las sumas de los números asignados a las caras, vértices y aristas
respectivamente. Como cada cara está formada por tres arsitas tenemos que

2Sa = 3Sc . (1)

Como cada vértice es común a tres aristas, tenemos que

2Sa = 3Sv . (2)

Además,
Sc + Sv + Sa = 1 + 2 + · · · + 14 = 105. (3)

Resolviendo el sistema formado por (1), (2) y (3) resulta que Sc = Sv = 30 y Sa = 45.
Para que el número asignado a una arista sea entero, debemos tener que los números
asignados a las caras adyacentes a la arista deben ser de la misma paridad y los
números asignados a los vértices que determinan la arista también deben ser de la
misma paridad. Pero si un vértice es par (impar) lo serán también los demás vértices,
esto mismo sucede con las caras.
Los conjuntos de cuatro números con la misma paridad, cuya suma sea 30 y sean ele-
mentos de {1, 2, . . . , 14}, son los siguientes ocho conjuntos: {1, 5, 11, 13}, {1, 7, 9, 13},
{3, 5, 9, 13}, {3, 7, 9, 11}, {2, 4, 10, 14}, {2, 6, 8, 14}, {2, 6, 10, 12}, {4, 6, 8, 12}.
Uno de estos conjuntos debe ser usado para numerar las caras y otro para numerar
los vértices. Puesto que ningún promedio debe ser elemento del conjunto, los úni-
cos conjuntos que satisfacen son: {1, 5, 11, 13} y {2, 4, 10, 14}. Ambos dan el mismo
conjunto para numerar las aristas: {3, 6, 7, 8, 9, 12}.
En conclusión, lo pedido se puede realizar de dos maneras, dependiendo de qué con-
junto se use para numerar las caras y qué conjunto para numerar los vértices.
19. Si N es el punto medio de C D se tiene que los triángulos rectángulos AMO y OND
son congruentes (tienen lados de medidas 3, 4 y 5). Luego
1 1
m∠AQB = m∠AOB + m∠C OD = m∠AOM + m∠DOM = 90◦ ,
2 2
178

entonces ABQ es un triángulo rectángulo.


Ahora, si M es el punto medio de AB, se tiene que m∠QAM = m∠MQA. Veamos
←→
ahora que los triángulos C QD y C LQ son semejantes (con L la intersección de QM
y C D), lo que implicará que QL⊥C D. Son semejantes pues tienen el ángulo C en
común y porque m∠C DQ = m∠C QL. En efecto m∠C DQ = m∠QAB (puesto que
abren el mismo arco de cı́rculo) y m∠QAB = m∠QAM = m∠MQA = m∠C QL por
ser opuestos por el vértice.

D
N L
C
O Q

A B
M

20. Como 2x + 3y no es múltiplo de 3, entonces z tampoco es múltiplo de 3. Esto significa


que z es de la forma 3k + 1 ó 3k + 2, en ambos casos, al elevar al cuadrado, se
obtiene un número que es un múltiplo de 3 más 1. Es decir, z 2 es un múltiplo de 3
más 1. Luego, también 2x debe ser un múltiplo de 3 más 1, por lo tanto, x debe ser
par: x = 2b (1). Reemplazando (1) en la ecuación original:
  
3y = z 2 − 22b = z + 2b z − 2b .

Pero los factores z + 2b y z − 2b no son simultáneamente múltiplos de 3, ya que


z + 2b − z − 2b = 2b+1 no es múltiplo de 3. Entonces, necesariamente
z − 2b = 1 (2)
z + 2b = 3y (3)

De (2) y (3) se tiene 2b+1 = 3y − 1 (4). De (4) vemos que 2b+1 debe ser un múltiplo
de 3 más 2; por esta razón, b debe ser par: b = 2c (5).
Caso 1: Si b = 0, obtenemos y = 1. Con esto, x = 0, z = 2.
Caso 2: Si b ≥ 2, entonces 3y debe ser un múltiplo de 4 más uno, por lo que y debe
ser par: y = 2d (6). Reemplazando (6) en (4):
  
2b+1 = 3d + 1 3d − 1 . (7)
 
Vemos que ambos 3d + 1 y 3d − 1 deben ser potencias de dos que se diferencian
en dos. Por lo tanto
 
3d + 1 = 4 (8)
 
3d − 1 = 2 (9)
179

De (8) y (9) resulta d = 1. Luego, b = 2, y = 2, x = 4, z = 5. En conclusión los


tripletes de enteros no negativos (x, y, z) que resuelven la ecuación dada son (0, 1, 2)
y (4, 2, 5).

21. Tomemos el trapecio ABC D inscrito en la circunferencia y tracemos otro trapecio


ABC F congruente al ABC D, como se muestra en la siguiente figura.

B C
s
s s s
A D
s P r
s r
r
F r r E
O

Llamemos P al punto de intersección de AD y C F . Como ABC P resulta ser un


paralelogramo, AP = PC = s. Y, entonces, F P = PD = r.
Si P resulta el centro de la circunferencia entonces se tendrı́a que s = r, lo cual
no es posible por hipótesis del problema. Por lo tanto, al unir los puntos F y D al
centro O de la circunferencia, obtenemos un paralelogramo F PDO. Al prolongar F O
hasta cortar nuevamente la circunferencia en E, se obtiene que DE = AF = s, por
ser AD y EF cuerdas paralelas. Como EF es un diámetro tenemos que la figura
ABC DEF es la mitad de un decágono regular de lado s. Ası́ que bastará encontrar
los ángulos internos del decágono, lo que nos da m∠ABC = m∠BC D = 144◦ y
m∠BAD = m∠ADC = 36◦ .

22. Observemos que si n es un número natural entonces

1 1 1 1 1 1
− = ⇒ = + (*)
n n+1 n(n + 1) n n + 1 n(n + 1

Usando esto y que 12 + 13 + 61 = 1, podemos hacer las sustituciones 12 por 13 + 61 , 13 por


1 1 1 1 1
4 + 12 y 6 por 7 + 42 , para obtener una nueva expresión en la que los denominadores son
más grandes. Luego podemos repetir el procedimiento hasta lograr que el denominador
más pequeño sea 5 y no haya repetidos:

1

1 1 1
1 1

1= 3 + 6 + 4 + 12 + 7 + 42 ,
1 1
 1 1
 1 1
 1 1
 1 1
 1 1

1= 4 + 12 + 7 + 42 + 5 + 20 + 13 + 156 + 8 + 56 + 43 + 1806
180

Ahora solo sustituimos 41 , 1


5 y 1
20 y los ordenamos:

1 = 51 + 61 + 17 + 81 + 1 1 1 1
12 + 13 + 20 + 21
1 1 1 1 1 1 1
+ 30 + 42 + 43 + 56 + 156 + 420 + 1806 .

Ası́ logramos una expresión de las requeridas. Usando (*) podemos descomponer el
término de denominador mayor y se obtienen denominadores aún mayores. A lo que
resulta se le puede aplicar el mismo procedimiento y ası́ sucesivamente, se pueden
obtener infinitas expresiones como las indicadas.

23. Claramente 10 es suertudo y como 12 + 32 = 32 + 12 = 10, tenemos que 13 y 31


también lo son. Observe que 32 + 22 = 13, por lo que 32 es suertudo. Entonces 31 y
32 son dos suertudos consecutivos y por lo tanto, para cualquier N ≥ 1,

|111{z
. . . }1 000
| {z. . . }0 y 111
| {z. . . }1 000
| {z. . . }0 1
31 unos N ceros 31 unos N−1 ceros

son dos enteros suertudos consecutivos.


También, si n y n + 1 son ambos suertudos consecutivos, también lo son

111
| {z. . . }1 0 y 111
| {z. . . }1
n unos n+1 unos

Repitiendo la construcción determinamos una infinidad de parejas de enteros suertudos


consecutivos.

24. Numeramos las casillas del tablero de la siguiente manera:

1 2 1 2 1 2 1 2 1
2 3 2 3 2 3 2 3 2
1 2 1 2 1 2 1 2 1
2 3 2 3 2 3 2 3 2
1 2 1 2 1 2 1 2 1

Tenemos 15 casillas numeradas con 1, 22 casillas numeradas con 2 y 8 casillas nume-


radas con 3. Si inicialmente tenemos a fichas sobre casillas numeradas con 1, b fichas
sobre casillas nunmeradas con 2, c fichas sobre casillas numeradas con 3, entonces
c ≤ 8 (solo hay 8 casillas numeradas con 3). Ahora observamos que después de dos
movimientos, todas las fichas que se encontraban sobre casillas 1 pasan a estar sobre
casillas 3, por lo tanto a ≤ 8. También, después del primer movimiento, todas las
fichas sobre casillas 2 pasan a ubicarse sobre casillas 1 y 3, entonces b ≤ a + c.
Ası́, b ≤ 16 y a + b + c ≤ 32. Con esto se demuestra que si el proceso continúa
indefinidamente entonces no puede haber más de 32 fichas en el tablero; esto es, si
hay 33 fichas el juego finalizará en algún momento.
181

Con exactamente 32 fichas en el tablero, sı́ es posible continuar el proceso indefini-


damente, para ello se pueden ubicar las fichas en las casillas marcadas con X:

X X X X X X X X .
X X X X X X X X
X X X X X X X X
X X X X X X X X

Moviéndose todas primero a la derecha, luego arriba, luego a la izquierda, luego abajo
y reiniciando el ciclo indefinidamente.

25. (a) Tenemos

cos 3x = cos 2x cos x − sin 2x sin x


= (2 cos2 x − 1) cos x − 2 sin2 x cos x
= 2 cos3 x − cos x − 2 cos x + 2 cos3 x = 4 cos3 x − 3 cos x.

(b) Sea x = 18◦ , entonces:


5x = 90◦ y 3x = 90◦ − 2x. Luego

cos 3x = cos(90◦ − 2x)


4 cos3 x − 3 cos x = sin 2x
4 cos3 x − 3 cos x = 2 sin x cos x
4 cos2 x − 3 = 2 sin x
4 sin2 x + 2 sin x − 1 = 0

La última ecuación tiene por soluciones



−1 ± 5
sin x = .
4

Como sin x > 0, entonces sin 18◦ = sin x = (−1 + 5)/4.

26. Como 240 = 24 · 3 · 5, hay que analizar la divisibilidad de a4 − 1 por 16, por 3 y por
5. Tenemos a4 − 1 = (a − 1)(a + 1)(a2 + 1). Por otra parte, como a no tiene factores
comunes con 240 entonces a debe ser impar; digamos a = 2k + 1. Entonces

a4 − 1 = 8k(k + 1)(2k 2 + 2k + 1). (*)

Como el producto de dos números consecutivos k y k +1 es divisible por 2, la igualdad


anterior dice que a4 − 1 es divisible por 16.
182

Si k = 3p + 1, a = 2k + 1 = 6p + 3 y se sabe que a no es divisible por 3 (a no


tiene factores comunes con 240), luego k tiene la forma 3p ó 3p + 2 y esto significa
que k(k + 1) es divisible por 3. Ası́, por (*), a4 − 1 es divisible por 3.
Como a no es divisible por 5, entonces k es de alguna de las formas: 5p, 5p + 1, 5p + 3
ó 5p + 4. En el primero y el último caso tendrı́amos que k(k + 1) es divisible por 5.
Por otra parte
2(5p + 1)2 + 2(5p + 1) + 1 = 50p2 + 30p + 5 (divisible por 5)
2(5p + 3)2 + 2(5p + 3) + 1 = 50p2 + 70p + 25 (divisible por 5)
Ası́, por (*), en todos los casos a4 − 1 es divisible por 5.
1 1 1 1
27. De a + b + c = a+b+c se sigue que

(bc + ac + ab)(a + b + c) = abc

y de aquı́: (b + c)(a + b)(a + c) = 0.


Sin perder generalidad podemos suponer que b + c = 0 y, por lo tanto c = −b. De
1
aquı́ se sigue que c1999 = −b1999 . De esto último se tiene que c1999 = b−1
1999 y que

c 1999 +b1999 = 0, de modo que


1 1 1 1 1
+ + = 1999 = 1999
a1999 b1999 c1999 a a + b1999 + c1999

28. Si las fracciones indicadas son enteras entonces su producto debe de ser un entero:
xy + 1 yz + 1 zx + 1 1 1 1 1
· · = xyz + x + y + z + + + +
z x y x y z xyz

Como x, y, z, xyz son enteros, entonces


1 1 1 1
+ + + (*)
x y z xyz
debe de ser un entero.
1 1 1
Caso 1: x = 1. Si en (*) hacemos x = 1 obtenemos 1 + y + z + yz . Como esto debe
1 1 1
ser un entero, entonces M = + + y debe ser un entero. Para y = 2, z = 3 se
z yz
tiene M = 1, entero. Para otros valores con 1 < y < z se tiene M < 1. Para este
caso la única solución al problema es x = 1, y = 2, z = 3.
Caso 2: x = 2. Reemplazando en (*) tenemos que N = 12 + y1 + z1 + 2yz 1
debe ser
entero. Probando con y = 3, z = 4 no se obtiene un entero; con y = 3, z = 5 tampoco
se obtiene un entero; con y = 3, z = 7 se obtiene N = 1, entero. Ası́, otra solución
del problema es x = 2, y = 3, z = 7. En todos los demás casos se puede ver que
N < 1.
Finalmente, si x ≥ 3, y ≥ 4, z ≥ 5, la expresión en (*) resulta menor que 1 y no hay
ninguna solución más.
183

29. Observe que para que las raı́ces estén bien definidas debemos tener yz ≥ 0, xz ≥ 0,
xy ≥ 0. Además

x = 42 + yz > 0

y = 6 + xz > 0

Esto es, x > 0, y > 0, y, entonces z > 0. Pongamos x = a2 , y = b2 , z = c2 , con


a, b, c > 0. Obtenemos un nuevo sistema
a2 − bc = 42 (1)
2
b − ac = 6 (2)
2
c − ab = −30 (3)

Restando (1) y (2):


(a − b)(a + b + c) = 36.
Restando (2) y (3):
(b − c)(a + b + c) = 36,
de donde se sigue que b = (a + c)/2. Sustituyendo esto en (2) se obtiene que
(a − c)2 = 24. (*)
Restando (1) y (3) obtenemos
(a − c)(a + b + c) = 72, (**)
como a + b + c > 0, entonces a − c > 0 y, de (*):
√ √
a − c = 24 = 2 6.
Sustituyendo en (**) obtenemos que
√ √
a + b + c = 36/ 6 = 6 6.
√ √ √ √
Concluimos
√ que a − b = 6, b − c = 6. De todo esto, a = 3 6, b = 2 6 y
c = 6. Ası́, x = 54, y = 24, z = 6.
30. Consideremos la figura:

β β
ββ a
c

A C
N L M k
k
184

Sean AM = MC = k, AB = c, BC = a. En 4BMC , por la ley de senos:

a k a cos 2β
= ⇒ = (1)
sin(90 + 2β) sin β k sin β

En 4ABC :
a 2k a 2 cos β
= ⇒ = (2)
sin(90 − β) sin 4β k sin 4β

De (1) y (2) podemos escribir cos 2β 2 cos β


sin β = sin 4β ⇒ 2 cos β sin β = cos 2β sin 4β, de donde
sin 2β = cos 2β · 2 cos 2β · sin 2β. Simplificando

(ya que sin 2β debe ser diferente de
2 1 2
0) obtenemos cos 2β = 2 y cos 2β = 2 (la solución negativa no procede en este
caso). Tenemos que 2β = 45◦ y β = 22, 5◦ . Los ángulos internos del triángulo miden:
m]A = 67, 5◦ , m]B = 90◦ y m]C = 22, 5◦ .

31. Como 324 = 182 necesitamos una factorización para la expresión a4 + 182 . Tenemos
a4 + 182 = (a2 + 182 ) − 36a2 = (a2 + 18 + 6a)(a2 + 18 − 6a) (*) Ası́:

4
Y (10 + 12k)4 + 182
N=
(4 + 12k)4 + 182
k=0
4
Y [(10+12k)2 +18+6(10+12k)][(10+12k)2 +18−6(10+12k)]
= [(4+12k)2 +18+6(4+12k)][(4+12k)2 +18−6(4+12k)]
k=0
4
Y (144k 2 +312k+178)(144k 2 +168k+58)
= (144k 2 +168k+58)(144k 2 +24k+10)
k=0
4
Y 144k 2 + 312k + 178
=
144k 2 + 24k + 10
k=0

Sean nk = 144k 2 + 312k + 178 y dk = 144k 2 + 24k + 10. Entonces podemos observar
que
dk+1 = 144(k + 1)2 + 24(k + 1) + 10 = 144k 2 + 312k + 178 = nk (0 ≤ k ≤ 3)
En conclusión:
n0 n1 n2 n3 n4 n0 n1 n2 n3 n4
N= = =
d0 d1 d2 d3 d4 d0 n1 n2 n3 n4
n4 144 · 42 + 312 · 4 + 178
= = 373.
d0 10

32. Tracemos AE de manera que m]EAC = 20◦ y BD ⊥ AE. Tenemos, por ángulo–ángulo,
que 4C AE ∼ 4ABC (pues tienen ]BC A en común y ]EAC ∼= ]C BA).
185

20

20

a2
De la semejanza: CE
a = a
b ⇒ CE = b.
Luego BE = b − EC = b − a2 /b. Como 4ABC es isósceles, ]BAC = 80◦ y

m]BAE = 60◦ ; ası́, en el triángulo rectángulo 4ABD se tiene BD = 23 b, AD = b2 y,
puesto que AE = AC y como AC = a, entonces AE = a y ED = AD − AE = b2 − a.
Como 4BDE es rectángulo se tiene

BE 2 = BD 2 + ED 2 ⇒
q
2
BE = 34 b2 + b2 − a

Por consiguiente, al ser BE = b − a2 /b, se tiene


q
a2 3 2
2
b− b = 4b + b
2 −a

Elevando al cuadrado y simplificando se obtiene lo pedido.

33. Considere la figura:

F E
H
B M
C
186

Primero recuerde que m∠FAH = m∠OAC (puesto que si AA0 es el diámetro por A
entonces los triángulos rectángulos AF H y AC A0 son semejantes). Por lo tanto
m∠FAM = m∠HAE y entonces los triángulos rectángulos AF M y AEH son seme-
jantes y como los catetos F M y EH son iguales, tenemos que los triángulos son
congruentes y entonces AE = FA. Por otro lado, los triángulos rectángulos ABE y
AC F son semejantes por tener el ángulo en común, y como AE = FA, resulta que son
congruentes, por tanto, tenemos que AB = AC .
34. Usando la tercera ecuación tenemos que xy + yz + xz = −z 2 ⇒ (y + z)(x + z) = 0.
Luego, z = −y ó z = −x.
Supongamos primero que z = −y, sustituyendo en la primera ecuación se tiene
x 3 = 8, y en la segunda ecuación se tiene x 2 + 2y2 = 22, luego x = 2 (pues x 3 = 8) y,
entonces 22 + 2y2 = 22 ⇒ y2 = 9 ⇒ y = 3, y = −3; como z = −y, entonces, para
y = 3, se tiene z = −3 y, para y = −3, se tiene z = 3; de aquı́ salen dos soluciones
(2, 3, −3) y (2, −3, 3).
Supongamos ahora que z = −x, sustituyendo en la primera y en la segunda ecuación
se obtiene que y3 = 8 y 2x 2 + y2 = 22; procediendo de modo análogo al caso
anterior se obtienen las soluciones (−3, 2, 3) y (3, 2, −3). Estas cuatro tripletas son
las soluciones del sistema.
2x+29
35. Observe que si x−2 = 3, entonces
2x + 29 = 3x − 6 ⇒ x = 35.
De modo que si en  
2x + 29
2f (x) + 3f = 100x + 80
x−2
sustituimos x por 3 tenemos
2f (3) + 3f (35) = 380. (†)
Si sustituimos x por 35 tenemos
2f (35) + 3f (3) = 3580 (‡)

Multiplicando (†) por −2 y (‡) por 3 y sumando lo que resulta, obtenemos 5f (3) = 9980
y, entonces f (3) = 1996.
x
36. Sean x1 < x2 < ... < x7 los enteros seleccionados y supongamos que xji > 2 para todo
j > i. Resulta xi+1
xi > 2 para todo i = 1, ..., 6. Entonces xi+1 ≥ 2xi + 1 ∀i = 1, ..., 6 y,
reemplazando hacia atrás tenemos xi+1 ≥ 2i x1 + 2i−1 + ... + 2 + 1 ∀i = 1, ..., 6. Para
i = 6 resulta
x7 ≥ 26 x1 + 25 + ... + 2 + 1
≥ 26 + 25 + ... + 2 + 1 = 127
Esto no puede ser puesto que todos los enteros seleccionados son ≤ 126.
187

37. Dado que sin 2θ = 2 sin θ cos θ y cos 2θ = cos2 θ − sin2 θ, el sistema se convierte en

x sin θ + y cos θ = 4a sin θ cos θ
x cos θ − y sin θ = a cos2 θ − a sin2 θ

Multiplique la primera ecuación por sin θ, la segunda por cos θ y luego súmelas; se
obtiene  
x = a 3 sin2 θ cos θ + cos3 θ (*)

Ahora multiplique la primera ecuación por cos θ y la segunda por − sin θ y súmelas;
se obtiene  
y = a 3 cos2 θ sin θ + sin3 θ . (**)

Luego, de (*) y (**) se tiene que

x + y = a(cos θ + sin θ)3


x − y = a (cos θ − sin θ)3

Ası́, obteniendo raı́z cúbica y luego elevando al cuadrado en la primera de estas dos
últimas ecuaciones, tenemos

(x + y)2/3 = a2/3 (cos θ + sin θ)2


= a2/3 (cos2 θ + 2 cos θ sin θ + sin2 θ)
= a2/3 (1 + sin 2θ) ,

es decir
(x + y)2/3 = a2/3 (1 + sin 2θ) .

Del mismo modo


(x − y)2/3 = a2/3 (1 − sin 2θ) .

Sumando estos dos últimos resultados tenemos

(x + y)2/3 + (x − y)2/3 = 2a2/3 ,

que es una relación entre x e y, en la que no aparece θ.

38. De (b) se tiene f (1) = f (1 · 1) = kf (1)f (1) ⇒ f (1) = 0 ó f (1) = k1 . Pero si f (1) = 0,
entonces f (x) = f (x · 1) = kf (x)f (1) = 0 ⇒ f (x) = 0, ∀x; pero en este caso no se
satisface (a); de modo que f (1) = k1 .
De (b) se tiene f (0) = f (0 · 0) = kf (0)f (0) ⇒ f (0) = 0 ó f (0) = k1 . Pero si f (0) = k1 ,
entonces k1 = f (x · 0) = kf (x)f (0) = f (x) ⇒ f (x) = k1 , ∀x; pero en este caso no se
satisface (a); de modo que f (0) = 0.
188

De (a),
4 4
f (2) = f (2) − 0 = f (2) − f (0) = f (1 + 1) − f (1 − 1) = ·1·1= .
k k
Ahora, sea x un número real cualquiera, entonces, por (a):
4 4
f (2x) = f (x + x) − f (x − x) = · x · x = x 2;
k k
pero también, por (b),
4
f (2x) = kf (2)f (x) = k · · f (x) = 4f (x).
k
De esto se tiene que k4 x 2 = 4f (x) ⇒ f (x) = k1 x 2 .
39. Tenemos
sin3 x + cos3 x = (sin x + cos x)(sin2 x − sin x cos x + cos2 x)
= (sin x + cos x)(1 − sin x cos x)
3
y 2 sin 2x = 3 sin x cos; luego
3
1 + sin3 x + cos3 x = sin 2x ⇒
2
1 + (sin x + cos x)(1 − sin x cos x) = 3 sin x cos x.

(sin x+cos x)2 −1


A la vez, sin x cos x = 2 ,
de modo que la ecuación se convierte en
  !
3 − (sin x + cos x)2 (sin x + cos x)2 − 1
1 + (sin x + cos x) =3
2 2

Si denotamos y = sin x + cos x, entonces tendremos


   2 
3 − y2 y −1
1+y =3 ⇒ (y + 1)(y2 + 2y − 5) = 0.
2 2
Esta última ecuación tiene tres soluciones:
√ √
y1 = −1, y2 = −1 + 6, y3 = −1 − 6.

Para y1 = −1, se tiene sin x + cos x = −1, pero


  
√ 1 1 √ π
sin x + cos x = 2 √ sin x + √ cos x = 2 sin x + ,
2 2 4
√  
de modo que 2 sin x + π4 = −1 y, por lo tanto sin x + π4 = √
−1
2
y de aquı́ se
obtienen las soluciones x = −π +(−1)n+1 π4 +nπ, con n ∈ Z. Las √raı́ces y2 y y 3 tienen
√4 √
valor absoluto mayor que 2, mientras que |sin x + sin y| = 2 sin(x + π4 ) ≤ 2,
por lo tanto ellas no aportan más soluciones de la ecuación inicial.
189

40. Si y = 0: f (x) + f (x) = f (3x) y ası́


2f (x) = f (3x) (1)
Si y = x: f (2x) + f (0) = f (3x) (2)
Si x = 2y: f (3y) + f (y) = f (6y) (3)
Haciendo x = 0 en (1): 2f (0) = f (0) ⇒ f (0) = 0. Sustituyendo en (2): f (2x) = f (3x) =
2f (x) y, por lo tanto, f (2x) = 2f (x) (4).
En (3): f (3y) + f (y) = f (2 · 3y) (4) 2f (3y), de donde se concluye que f (3y) = f (y).
=
Ası́, en (1): 2f (x) = f (3x) ⇒ 2f (x) = f (x) ⇒ f (x) = 0 para todo x y, entonces f es la
función constante idénticamente nula.
41. Supongamos que x1 < x2 ; restando las dos primeras ecuaciones tenemos
√ √
x1 − x2 = x2 − x3 < 0 ⇒ x2 < x3 .

Procediendo de modo similar obtenemos sucesivamente x3 < x4 < . . . < xn−1 < xn <
x1 , lo cual es una contradicción. Si suponemos que x1 > x2 , se obtiene igualmente
una contradicción. Por lo tanto x1 = x2 y, sucesivamente, x1 = x2 = · · · =√
xn−1 = xn .
Llamando√t = x1 = x2 = · · · = xn−1 = xn , el sistema se convierte en t − t = a ⇒
t−a= t ⇒
t 2 − (1 + 2a)t + a2 = 0 ⇒

1+2a+ 1+4a
t= 2 como única solución positiva. Es decir, la solución es

1+2a+ 1+4a
x1 = x2 = · · · = xn−1 = xn = 2 .
42. Consideremos el polinomio
p(x) = f (x) − 5;
tenemos que p(a) = p(b) = p(c) = p(d) = 0, por lo tanto, existe un polinomio q(x)
tal que p(x) = (x − a)(x − b)(x − c)(x − d)q(x).
Dado que p(x) es de coeficientes enteros y a, b, c, d son enteros, entonces q(x) es
de coeficientes enteros. Si k es un entero tal que f (k) = 8, tendrı́amos que p(k) =
f (k) − 5 = 8 − 5 = 3 y, por lo tanto:

3 = (k − a)(k − b)(k − c)(k − d)q(k) (\)

Puesto que los coeficientes de q(x) son enteros y k es entero, se tiene que q(k) ∈ Z
y (\) significa que (k − a)|3, (k − b)|3, (k − c)|3, (k − d)|3. Pero como a, b, c, d son
diferentes, entonces k − a, k − b, k − c, k − d son diferentes. Dado que 3 tiene cuatro
divisores: −3, −1, 1 y 3, entonces cada uno de los números k − a, k − b, k − c, k − d
es igual a uno de esos divisores y tendrı́amos

3 = 1 · −1 · 3 · −3 · q(k) ⇒ 1 = 3 · q(k),

pero esto no puede ser puesto que q(k) es un entero.


190

43. Consideremos la figura:

O
Q
I

B C

Sea I el centro del cı́rculo inscrito al triángulo ABC , O el centro del cı́rculo circunscrito,
AP el diámetro de éste y Q el punto de tangencia de AC con el cı́rculo inscrito.
Recordemos que I es la intersección de las bisectrices y O la de las mediatrices;
por ser AB = AC , la mediatriz y la bisectriz coinciden. Sea d la distancia pedida.
4AIQ ∼ 4APC (par de ángulos congruentes; el ángulo A es común y ambos tienen
un ángulo recto, uno, ángulo inscrito en una semicircunferencia y el otro, el radio es
IQ
perpendicular a la tangente en el punto de tangencia). Ası́, AP AI
= PC , pero AI = 9 + d,
9+d 4
AP = 18, IQ = 4; entonces 18 = PC (†). Ahora veamos que PC = IP. Tenemos
∠BC P ∼ = ∠BAP pues abarcan el mismo arco del cı́rculo. Además ∠BAP = ∠PAC ,

por ser AP bisectriz de ∠BAC . Por otro lado, m∠AIC + m∠C IP = 180◦ y también
m∠AIC + m∠C AI + m∠IC A = 180◦ (esto último porque son ángulos internos del
triángulo AC I); además ∠C AI ∼ ∼
= ∠PBC y ∠IC A = BC I, ası́ que, comparando, tenemos
m∠C IP = m∠C AI + m∠IC A = m∠PC B + m∠BC I = m∠PC I, de donde 4PC I es
isósceles con lados congruentes PI = 9 − d y PC . Regresando a (†), sustituyendo
PC por 9 − d y despejando d:
9+d 4
= ⇒ 81 − d2 = 72 ⇒ d = 3 cm.
18 9−d
44. En primer lugar observamos que el dominio de f es el conjunto de los números reales
positivos, exceptuando el 1, pues logx 5 solo está definido si x > 0, x 6= 1. Ahora,
f (x) ≤ 0 ⇒ 125x 4 ≤ 5logx 5 . Ambos miembros de esta desigualdad son positivos de
manera que podemos aplicar a ambos miembros el logaritmo en base 5 y preservamos
log 5
la desigualdad: 4 log5 x + 3 ≤ logx 5. Además logx 5 = log5 x = log1 x . Si denotamos
5 5
y = log5 x, la desigualdad se convierte en 4y + 3 ≤ y1 , que se puede escribir como
(y+1)(y− 14 )
y ≤ 0. Resolviendo esta desigualdad se tiene que y ≤ −1 ó 0 < y ≤ 41 . Es

decir log5 x ≤ −1 ó 0 < log5 x ≤ 41 . Por lo tanto, f (x) ≤ 0 si 0 < x ≤ 15 ó 1 < x ≤ 4 5.
191

45. Los enteros de esa forma son los del 5 al 45. Con a1 = a2 = . . . = a9 = 1, obtenemos
el mayor de ellos, 45. Con a1 = a2 = . . . = a9 = 9, obtenemos el menor, 5. Como
1 2 7 28
7 + 7 + · · · + 7 = 7 = 4, se pueden escribir 6, 7, 8 y 9 ası́:

1 2 7 8 9
+ + ··· + + +
7 7 7 a8 a9


 6 si a8 = 8, a9 = 9

7 si a8 = 4, a9 = 9
=

 8 si a8 = 8, a9 = 3

9 si a8 = 2, a9 = 9

Si tomamos cada ai = i, cada fracción es 1 y obtenemos 9. Si cambiamos alguna


ai de i a 1, la suma aumenta 1i − ii = i − 1, entonces podemos conseguir aumentos
(independientes) de 1, 2, 3, . . ., 8 y ası́ formamos los números del 10 al 45.

46. Se tiene que

q
1
f (x + 2a) = f ((x + a) + a) = + f (x + a) − [f (x + a)]2
2 p
= 12 + f (x + a)(1 − f (x + a))
r q q
= 2 + ( 12 + f (x) − [f (x)]2 )( 12 − f (x) − [f (x)]2 )
1

r
1 1
= + − f (x) + [f (x)]2
2 4
1 1
= + f (x) −
2 2

Pero, por hipótesis,


q
1
f (x) = f (x − a + a) = + f (x − a) − [f (x − a)]2
2

y por lo tanto, f (x) ≥ 12 , luego f (x) − 21 = f (x) − 12 y, entonces

1 1 1 1
f (x + 2a) = + f (x) − = + f (x) − = f (x).
2 2 2 2

47. Es claro que todas las tripletas (a, a, b), (a, b, a), (b, a, a) son soluciones de la ecua-
ción, para cualesquiera a, b ∈ R (incluyendo el caso a = b). Estas son las únicas
soluciones. En efecto, supongamos que hay una solución (p, q, r), con todos ellos
diferentes. La ecuación se puede reescribir como

x n (y − z) + yn (z − x) + z n (x − y) = 0
192

y, como (p, q, r) es solución, tendrı́amos

pn (q − r) + qn (r − p) + r n (p − q) = 0.

Como r − p = (r − q) + (q − p), entonces se tiene

pn (q − r) + qn (r − q) + qn (q − p) + r n (p − q) = 0

⇒ (pn − qn )(q − r) = (qn − r n )(p − q).


Como q − r y p − q son diferentes de cero, concluimos que

pn − q n qn − r n
= . (*)
p−q q−r

Ahora, si consideramos en el plano los puntos (p, pn ), (q, qn ), (r, r n ), la igualdad (*)
significa que ellos son colineales. Pero estos tres puntos pertenecen a la gráfica de
f (x) = x n y, como n es par, no puede tener tres puntos alineados. Es decir, p, q, r no
pueden ser todos diferentes.

48. Consideremos la figura:

K
A

L E P

S
I
Q B
G H
R
J

C
D

Denotemos los puntos medios de EA, ED, EC y EB como F , G, H, I respectivamente.


Sabemos que F S = 31 F D y F P = 31 F B; además el ∠SF P es común, entonces
4F PS ∼ 4F BD, ası́ PS = 13 BD (*). También se sabe que RH = 13 DH y QH = 31 HB
y ∠RHQ es común, entonces 4RHQ ∼ 4DHB, con lo que RQ = 13 BD (**); de (*)
y (**) vemos que PS = RQ (?). Como PI = 13 AI y QI = 31 IC y ∠PIQ es común,
entonces 4PQI ∼ 4AC I, con lo que PQ = 13 AC (***); de modo parecido tenemos
que RS = 13 AC (****). De (***) y (****) se tiene que PQ = RS y, por (?), se tiene que
PQRS es un paralelogramo. Tracemos las paralelas a AC por B y por D; también las
193

paralelas a BD por A y C . Estas 4 paralelas se cortan en los puntos J, K , L y M


(vea la figura), de tal forma que K LMN es un paralelogramo de lados AC = 3PQ y
BD = 3RQ ⇒ 4PQR ∼ 4K JM en razón 1 : 3 y 4PSR ∼ 4K LM en razón 1 : 3.
(JK LM)
Observamos que 2(ABC D) = (JK LM) y teniendo en cuenta que (PQRS) = 19 , entonces
2(ABC D) 9
(PQRS) = 1 ⇒
2
(PQRS) = (ABC D).
9
49. Consideremos la figura:

C2 E C1
F
Q P
B
C

Notemos que m∠AQB = 90◦ pues está inscrito en una semicircunferencia. De igual
forma m∠C FA = m∠C PA = m∠AEB = 90◦ . Tenemos que EP es la altura de
4C PA, recto en P y QF es la altura de 4AQB, recto en Q. De aquı́ se tiene que
AP 2 = AC · AE (*) y AQ 2 = AB · AF (**). Además, los triángulos rectángulo ABE y
AC F son semejantes, por lo tanto AF · AB = AE · AC . Luego por esto último y por (*)
y (**) se tiene AP 2 = AQ 2 ⇒ AP = AQ.

50. Trace desde Q la tangente a la circunferencia de centro A y radio 1, y sea P 0 el punto


de intersección de ésta con BC .
Veamos que m∠P 0 AQ = 45◦ , lo que implicarı́a que P 0 = P, con lo que se justificará la
parte (a).

Q
D C

P0
β
βα
A α B

Sea T el punto de tangencia, como T Q = QD se tiene que los triángulos rectángulos


AQD y AQT son congruentes, por lo que m∠TAQ = ∠DAQ = β. Análogamente,
194

m∠BAP 0 = m∠TAP 0 = α y como 2α + 2β = 90◦ tenemos que m∠P 0 AQ = α + β =


45◦ . Del triángulo rectángulo AT E tenemos que AE = cos12α . Del triángulo rectángulo
AT F tenemos que AF = cos12β .
Luego,
1 1 cos 2α + cos 2β
AE + AF = + = .
cos 2α cos 2β cos 2α cos 2β
2 1 4
Como ab ≤ a+b 2 tenemos que ab ≥ (a+b)2
, esta desigualdad aplicada a la identidad
anterior nos da:
4
AE + AF ≥ .
cos 2α + cos 2β
Como la función cos x satisface que
 x + y  cos x + cos y
cos ≥ ,
2 2
tenemos
4 2 2 √
AE + AF ≥ ≥  = = 2 2.
cos 2α + cos 2β cos 2α+2β cos 45◦
2

51. Prolonguemos el lado BC y tracemos el segmento OP que va del centro O perpen-


dicularmente a la prolongación del lado BC . Si la longitud de este segmento es x
entonces el lado del cuadrado es 2x.

B 2x C P
x
O

A
D

Sea y = C P. Tenemos:
√ 2
Por Pitágoras en 4OBP : (1 + 2) = x 2 + (2x + y)2 (1)
Por Pitágoras en 4OC P : 12 = x 2 + y2 (2)
Entonces, por (1) y (2):

2 + 2 2 = (2x + y)2 − y2 ⇒

2 + 2 2 = 2x(2x + 2y) ⇒

1+ 2
− x = y.
2x
195

 √ 2 √ 2 √
(1+ 2)
Sustituyendo en (2): 1 = x 2 + 1+2x 2 − x ⇒ 1 = x 2 + 4x 2 − (1 + 2) + x 2 ⇒
√ √
4x 2 = 8x 4 − 4x 2 (1 + 2) + (1 + 2)2 ⇒
√ √
8x 4 − 4x 2 (2 + 2) + (1 + 2)2 = 0 (3).

Como el lado del cuadrado es 2x, entonces el área es 4x 2 . Pongamos A = 4x 2 , entonces


(3) se convierte en
1 2 √ √
A − A(2 + 2) + (1 + 2)2 = 0 ⇒
2 √ √
A2 − 2A(2 + 2) + 2(1 + 2)2 = 0 ⇒
√ √
A2 − 2A(2 + 2) + (2 + 2)2 = 0 ⇒
h √ i2
A − (2 + 2) = 0


De donde se obtiene que A = 2 + 2.

52. Sea N = abcde el número buscado; este deberá ser igual a la suma

abc + abd + · · · + cde;

esta suma tendrá 3 · 4 · 5 = 60 sumandos. Como primera cifra de los sumandos


aparecerá igual número de veces las cinco cifras a, b, c, d, e, es decir, habrá 60 ÷ 5 =
12 sumandos cuya primera cifras es a, 12 cuya primera cifra es b, etc. Lo mismo
sucederá con las segundas y terceras cifras de los sumandos. Ası́, tendremos

N = (100a + 10b + c) + (100a + 10b + d) + · · · + (100c + 10d + e)


= (1200a + 120a + 12a) + (1200b + 120b + 12b) + · · · + (1200e + 120e + 12e)
= (1200 + 120 + 12)(a + b + c + d + e) = 12 · 111(a + b + c + d + e)

Si S = a + b + c + d + e, entonces N = 12 · 111 · S. S será máximo cuando las cifras


de N sean 9, 8, 7, 6 y 5, entonces S = 35. S será mı́nimo cuando las cifras de N sean
1, 2, 3, 4 y 5, entonces S = 15. De manera que S debe estar comprendido entre 15 y
35, pero como N = 12 · 111 · S, entonces N debe ser múltiplo de 9 ya que 12 y 111 son
múltiplos de 3. De esta forma, también debe ser múltiplo de 9 la suma S de las cifras
de N. Los únicos múltiplos de 9 comprendidos entre 15 y 35 son 18 y 27. Para S = 18
resulta N = 23976 que no vale pues S = 27 6= 18. Para S = 27 resulta N = 35964
que es la solución buscada pues S = 27.

53. Dividamos a las personas en dos grupos: en el primero ubicamos a las personas de
pesos a1 ,..., a79 , en el segundo a las restantes n − 79 personas. La personas del primer
grupo deberán viajar solas en el bote porque si lo hicieran acompañadas superarı́an
la capacidad de peso del bote; cada una de las personas del segundo grupo, pueden
viajar con la persona de peso an .
196

La persona de peso an debe llevar a todas las otras del segundo grupo (una a la
vez) a la otra orilla y regresarse, ası́, atraviesa el rı́o 2(n − 80) veces. Luego de esto
las personas a80 ,... , an−1 se encuentran del otro lado y a1 ,..., a79 y an del lado en
que se encontraban inicialmente (an tuvo que volver la última vez para traer el bote).
Ahora, a1 atraviesa el rı́o y se queda del otro lado, uno de los que estaban allá debe
regresarse con el bote, digamos el an−1 , entonces an debe llevar de nuevo a an−1
al otro lado (no puede llevarse a uno de los que se encuentran de este lado porque
se superarı́a la capacidad de peso del bote); luego an regresa; ası́ se han realizado
cuatro viajes más. Esto debe repetirse hasta que se vaya a78 , entonces deben sumarse
4 · 78 = 312 viajes a los 2(n − 80) que ya tenı́amos. A este momento solo tenemos de
este lado a an y a79 , entonces a79 se va (1 viaje más), an−1 regresa (1 más) y luego
cruzan an−1 con an (1 más), ası́, hay tres travesı́as más. Tenemos en total

312 + 2(n − 80) + 3 = 355


2n + 155 = 355
n = 100.

54. Numeramos las columnas de izquierda a derecha del 1 al 8 y las filas de arriba a
abajo del 1 al 8. Observamos que el número que se encuentra en la fila i y columna
j, se puede escribir como 8(i − 1) + j. En cada fila hay cuatro números negativos y
cuatro números positivos; supongamos que nos ubicamos en la fila i y digamos que
los positivos de esa fila son los números que pertenecen a las columnas a, b, c, d y
los negativos son los que pertenecen a las columnas w, x, y, z. Al sumar los números
en esa fila tendremos entonces

[8(i − 1) + a] + [8(i − 1) + b] + [8(i − 1) + c] + [8(i − 1) + d]


−[8(i − 1) + w] − [8(i − 1) + x] − [8(i − 1) + y] − [8(i − 1) + z]
= a + b + c + d − (w + x + y + z)

Es decir, el resultado de la suma en cada fila depende únicamente de las posiciones


que ocupen los positivos y los negativos. Como hay cuatro números con signo más y
cuatro con signo menos en cada columna entonces cada número a, b, c, d, w, x, y,
z aparecerá cuatro veces con signo positivo y cuatro con signo negativo en la suma
total, de modo que esta suma es siempre igual a 0.

55. Si trazamos la diagonal BD nos damos cuenta que m∠MBD > m∠MBC = 90◦ y,
por lo tanto MD > BM. Ası́, por la semejanza de los paralelogramos se tiene

BM MD
= . (1)
DC BC
197

B K
C

A
D

Sea K el punto intersección de MD y BC ; de la semejanza de los triángulos rectángu-


los MBK y K C D se deduce que

BM MK
= (2)
DC KC

Dado que MD = MK + K D, BC = BK + K C y de acuerdo con (1) y (2), se tiene que

MK MK + K D KD
= = (3)
KC BK + K C BK

Pero, de los triángulos semejantes MBK y K DC se deduce que

MK BK
= . (4)
KC KD

De acuerdo con (3) y (4) se concluye que MK = K C y entonces, por (2), se deduce
que BM = DC = 1 y de (1) se tiene que MD = BC = 2. Ası́ resulta que los
paralelogramos ABC D y BMDN son congruentes.
Puesto que MK = K C , del triángulo rectángulo MBK , según el teorema de Pitágoras,
obtenemos MK 2 = 1 + (2 − MK )2 , es decir MK = 54 . Además, sin (∠BMK ) = MK BK
=
2−K C 2−MK 3
MK = MK = 5 . Ası́, concluimos que el área del paralelogramo es

6
(ABC D) = AB · BC · sin (∠BMK ) = .
5

56. Si 4URT esta inscrito en C y es recto en R. entonces como ∠ARB es recto debe
existir un cı́rculo C 0 de diámetro |AB| que se inteseque con C .
De hecho esa es la condición necesaria y suficiente. Sea r el radio de C , d la distancia
del centro de C al punto medio de AB, la condición necesaria se reduce a:

|AB|
≥r−d
2
198

57. Designemos p(x) = f (x) − 5, entonces a, b, c, d, e son ceros de p(x) y por el teorema
del factor se obtiene que:

p(x) = (x − a)(x − b)(x − c)(x − d)(x − e) (1)

Ahora, como f (0) = −7, entonces

p(0) = f (0) − 5 = −7 − 5 = −12 (2)

Y evaluando en x = 0 en (1) resulta que

p(0) = −a · −b · −c · −d · −e = −abcde (3)

De (2) y (3) se deduce que abcde = 12. Con lo cual se concluye que:

166(abcde) + 8 = 166 · 12 + 8 = 2000.

58. Para n = 1 es evidente. Si n = 2k con k ≥ 1; los divisores de n son 1, 2, ..., 2k y su


suma es

S = 1 + 2 + · · · + 2k = 2k+1 − 1
√ k
≤ 2k 2k = 2k+ 2 si k ≥ 2

y S = 1 + 2 = 3 ≤ 2 2 si k = 1.
En general, si n = pk con k ≥ 1, los divisores de n son 1, p, ..., pk y su suma es

pk+1 − 1
S = 1 + p + · · · + pk =
p−1
q
k
≤ pk pk = pk+ 2 si k ≥ 2

y S = 1 + p ≤ p p si k = 1.
Si n y m son primos relativos la suma de los divisores de nm se obtiene al multiplicar
la suma de los divisores de n por la suma de los divisores de m y el resultado se
obtiene en el caso general usando la descomposición de n en factores primos.

59. Sea n ∈ N. Entonces

9n + 9 > 6n − 1; 9n > 6n − 4 ⇒
√ √ √ √
3 n + 1 > 6n − 1; 3 n > 6n − 4.
Sumando miembro a miembro las desigualdades anteriores se obtiene que:
√ √ √ √
3 n + 1 + 3 n > 6n − 1 + 6n − 4
199

Racionalizando se sigue que:

1 1
√ √ >√ √
n+1− n 6n − 1 − 6n − 4
√ √ √ √
6n − 1 − 6n − 4 > n + 1 − n (1)

Ahora, evaluando para n = 1, 2, 3, ..., 100 en la desigualdad (1) resulta la siguiente


secuencia de desigualdades:
√ √ √
5− 2> 2−1
√ √ √ √
11 − 8 > 3 − 2
√ √ √ √
17 − 14 > 4 − 3
···
√ √ √
599 − 596 > 101 − 10

Sumando miembro a miembro las 100 desigualdades se obtiene que:


√ √ √ √ √ √
101 − 1 < 5 − 2 + 11 − 8 + 17
√ √ √
− 14 + · · · + 599 − 596

Racionalizando en parejas el primer miembro de (2) resulta que:

√ 3 3
101 − 1 < √ √ +√ √ (3)
5+ 2 11 + 8
3
+··· + √ √
599 + 596

Luego, multiplicando ambos miembros de (3) por 13 y además como


√ √
101 − 1 > 100 − 1 = 9, entonces se concluye que:

1 1 1 1
3< √ √ +√ √ +√ √ + ··· + √ √ .
2+ 5 8 + 11 14 + 17 596 + 599

60. Primero veamos que si


P + Q + R = 0◦ ,

entonces
P Q R
sin P + sin Q + sin R = −4 sin sin sin (2)
2 2 2
200
 
En efecto, P +Q +R = 0◦ ⇒ R = −(Q +P) ⇒ sin R2 = sin − Q+P
2 = − sin Q+P
2 =
− sin Q2 cos P2 − sin P2 cos Q2 ⇒
 
R Q P Q P P Q Q P
− sin sin sin = sin cos + sin cos sin sin
2 2 2 2 2 2 2 2 2
Q P P P Q Q
= sin2 cos sin + sin2 cos sin
2 2 2 2 2 2
Q 1 P 1
= sin2 · sin P + sin2 · sin Q ⇒
2 2 2 2
R Q P 1 − cos Q 1 − cos P
−2 sin sin sin = · sin P + sin Q
2 2 2 2 2
1 1 1
= sin P + sin Q − (cos Q sin P + cos P sin Q)
2 2 2
1
= (sin P + sin Q − sin(P + Q))
2
1
= (sin P + sin Q + sin[−(P + Q)])
2
1
= (sin P + sin Q + sin R) ⇒
2
R Q P
−4 sin sin sin = sin P + sin Q + sin R.
2 2 2
√ sin 60◦
Como 3= cos 60◦ , si se cumple (1) tendremos
(sin A cos 60◦ − cos A sin 60◦ )+(sin B cos 60◦ − cos B sin 60◦ )+(sin C cos 60◦ − cos C sin 60◦ ) =
0⇒
sin(A − 60◦ ) + sin(B − 60◦ ) + sin(C − 60◦ ) = 0.
Como A + B + C = 180◦ , entonces

(A − 60◦ ) + (B − 60◦ ) + (C − 60◦ ) = 0

y se aplica (2), entonces

A − 60◦ B − 60◦ C − 60◦


sin · sin · sin =0
2 2 2

De aquı́ se tiene que al menos uno de los ángulos A − 60◦ , B − 60◦ , C − 60◦ es igual
a 0 y entonces A o B o C es igual a 60◦ .

61. Si hacemos x1 = . . . = x2000 = 0 tenemos que 2000 = 2000 · f (0), entonces f (0) = 0
1999
√ f (0)
1999
ó f (0) = 2000 y, en este caso, f (0) = 2000.
Si hacemos x1 = . . . = x1999 = 0 y x2000 = x arbitrario en R, entonces f (0)1999 · f (x) =
1999f (0)
1999 · f (0) + f (x) ⇒ f (x)[f (0)1999 − 1] = 1999 · f (0) ⇒ f (x) = . Para el
f (0)1999 − 1
201

caso f (0) = 0√se tiene f (x) = 0 para todo x. Para el caso f (0) = 1999 2000 se tiene
1999 1999 2000 √
f (x) = = 1999 2000. Solo hay dos funciones que cumplen con la condición
2000 − 1 √
dada y son las funciones constantes: f (x) = 0 y f (x) = 1999 2000.

62. Vemos que, para todo x:

f (x + 1) = f (1x) = f (x) (*) y


f (x + 2) = f ((x + 1) + 1) = f (1(x + 1) = f (x + 1) = f (x), (por (*))

2
∀x real positivo . Si x > 2 real, cualquiera, entonces existe y real tal que x = y y+1
(y es solución de y2 − xy + 1 = 0, que existe puesto que 4 = x 2 − 4 > 0). En ese
caso:
 2     
y +1 1 1
f (x) = f =f y+ = f y( ) = f (1)
y y y

Si 0 < x < 2 entonces 2 + x > 2 y se tiene f (2 + x) = 1, pero sabemos que


f (2 + x) = f (x) y, entonces, se tiene

f (x) = f (x + 2) = f (1).

63. Sin perder generalidad, sean P1 , . . . Pn las vértices del polı́gono regular de n lados,
de lado l. Sea P un punto sobre el lado P1 P2 demostraremos que la suma de las
distancias de P a cada uno de los lados es constante.
Sean h2,3 , h3,4 , . . . hn,1 las distancias desde P al resto de los lados. Si se trazan los
segmentos que unen a P con cada uno de los vértices P3 , . . . Pn se forman n − 1
triángulos 4PP2 P3 , 4PP3 P4 , . . . 4PPn P1 , que juntos forman el polı́gono. Dado que
las distancias h2,3 , h3,4 , . . . hn,1 coinciden con las alturas de los triángulos, respecti-
vamente; y que el área, A, del polı́gono es la suma de las áreas de los triángulos en
que se descompone.
l
A = (h2,3 + h3,4 + · · · + hn,1 )
2
es decir,
2A
h2,3 + h3,4 + · · · + hn,1 = .
l

64. Sea D el pie de la altura sobre BC . Llame m∠BAC = α y m∠AC B = θ. Trace el


circuncı́rculo, Γ del 4ABC y sean P 0 intersección de la prolongación de AO con Γ y
sea Q 0 intersección de la prolongación de BO con Γ. Tenemos que m∠C AP 0 = 90 − θ,
y m∠C BQ 0 = 90 − θ, y como m∠C AP 0 = m∠C BP 0 , pues inscriben el mismo ángulo,
se concluye
m∠OBD = m∠P 0 BD.
202

Esta última relación, la perpendicularidad de OD con BC , y el hecho de compartir


el lado BD llevan a la congruencia entre 4ODB y 4P 0 DB de donde obtiene que
OD = DP 0 es decir P 0 coincide con P. Un razonamiento similar establecer que Q y
R también están sobre Γ.
Para concluir el ejercicio se debe observar que m∠APR = m∠AC R = 90−α. Entonces
m∠APQ = m∠ABQ = 90 − α, es decir m∠APR = m∠APQ, PO es la bisectriz del
∠RPQ. Un razonamiento similar permite obtener que OR y OQ son las bisectrices
del ∠PRQ y de ∠PQR, repectivamente. Por lo tanto O es el incentro del 4PQR.

R0 A

Q0
O

D
B C

P0

65. Para m = n = 1, se tiene f (2) − 2f (1) = 0 ó 1 ⇒ −2f (1) = 0 ó 1. Como f (1) es no


negativo, entonces f (1) = 0.
Para m = 2, n = 1 se tiene

f (3) − f (2) − f (1) = 0 ó 1,

es decir f (3) = 0 ó 1, pero f (3) > 0, luego, f (3) = 1.


Para m = 3 y cualquier n, se tiene

f (n + 3) − f (3) − f (n) = 0 ó 1,

es decir, f (n + 3) = f (n) + 1 ó f (n) + 2.


Como f (3) = 1 y f (n + 3) = f (n) + 1 ó f (n) + 2, entonces f (n + 3) > f (n) y f (6) =
f (3 + 3) = 2 ó 3, f (9) = f (6 + 3) = 3 ó 4, ..., f (3k) = k ó k + 1. Pero como
f (3 · 3333) = 3333, entonces f (3 · 3332) = 3332 (la otra posibilidad es 3333, pero,
como f (n + 3) > f (n), esta no se puede dar), siguiendo de esta manera hacia atrás,
obtenemos que f (3k) = k, para 1 ≤ k ≤ 3333.

66. Si a = 0 ó b = 0 la afrimación es evidente. Supongamos entonces que a > 0 y b > 0


y, sin perder generalidad, 0 < a ≤ b. Entonces 0 < ab ≤ 1 y como p > q, entonces
 a p  a q  a p  a q
0< ≤ ⇒1+ ≤1+
b b b b
203

De aquı́ se tiene que


h  a p i1/q h  a q i1/q
1+ ≤ 1+ .
b b
 1
Luego, ya que 1 + a p
b ≥1y0< p < q1 , obtenemos que
h  a p i1/p h  a p i1/q
1+ ≤ 1+
b b
De estas dos última desigualdades se tiene que
h  a p i1/p h  a q i1/q
1+ ≤ 1+
b b
Por lo tanto  p   q 
a + bp 1/p a + bq 1/q
≤ ,
bp bq
es decir, puesto que b > 0,
(ap + bp )1/p ≤ (aq + bq )1/q .

67. Hay que probar que 4ABC y 4DEF son equiláteros.


I. 4ABC .

O1
F A E

B C
O2 O3

Tenemos O1 B = O1 C = O2 A = O2 C = O3 A = O3 B = r ⇒
4O1 C O2 ∼ ∼
= 4O2 AO3 = 4O1 BO3 (por L–L–L) ⇒
m∠BO1 O3 = m∠C O1 O2 = α.
Sea m∠BO1 C = β ⇒ α + α − β = 60◦ ⇒
2α − β = 60◦ ⇒ 2α − 60◦ = β.
Como m∠C O2 O1 = m∠AO2 O3 = α, llegamos a que m∠AO2 C = β; análogamente
m∠AO3 B = β ⇒ 4AO2 C ∼ ∼
= 4AO3 B = 4BO1 C (por L–A–L) ⇒
AB = AC = BC ⇒ 4ABC ∼ 4O1 O2 O3 .
II. 4DEF .
204

O1
F A E

B C
O2 O3

Tenemos que
4O1 EO3 ∼ ∼
= 4O3 DO2 = 4O2 F O1 (por L–L–L) ⇒
m∠EO1 O3 = m∠F O1 O2 = α.
Sea m∠F O1 E = β ⇒ α + α + 60◦ = β ⇒
β = 2α + 60◦ . Análogamente a I, concluimos que
m∠F O1 E = m∠EO3 D = m∠DO2 F = β ⇒ 4F O1 E ∼ ∼
= 4EO3 D = 4DO2 F (por
L–A–L) ⇒
F E = DE = DF ⇒ 4DEF ∼ 4O1 O2 O3 .
Solución de las eliminatorias del año
2000

Primera eliminatoria
III ciclo
1. (b) Los enunciados de los cofres de oro y plata dicen lo contrario, entonces uno de
ellos es verdadero. Si lo que dice el de oro fuera verdadero, entonces lo que dice el
de plata también serı́a verdadero, como solo uno dice la verdad, entonces lo que dice
el cofre de oro es falso, es decir, el retrato no puede estar en el cofre de oro. Por lo
dicho antes, entonces el que dice la verdad es el cofre de plomo, ası́, lo que dice el
de plata es falso, como el de plata dice que el retrato no está en él, entonces, al ser
esto falso es porque el retrato está en el cofre de plata.
2. (b) En cada viaje de ida van dos botes y de regreso va uno. Son necesarios tres viajes
de ida y dos de vuelta.
Ocho y Cuatro deben hacer juntos el viaje de ida, si no se tardará por lo menos
8 + 4 + 2 horas de ida y 1 + 1 horas en los regresos; en total 16 horas.
El viaje de ida de 8 y 4 no puede ser el primero porque si no uno de ellos debe hacer
un viaje de vuelta y un segundo viaje de ida (tardando más de 8 + 4 + 4 = 16 horas).
Tampoco puede ser el tercer viaje, porque entonces uno de ellos cruzará dos veces de
ida y una vez de vuelta. Ası́, los cruces de ida son Uno y Dos, Cuatro y Ocho, Uno y
Dos y los cruces de vuelta son uno con Uno y el otro con Dos (en cualquier orden).
Se tardan 2 + 8 + 2 + 1 + 2 = 15 horas. Una secuencia posible de viajes tardando 15
horas es:
I viaje de ida: Uno y Dos 2h
I viaje de vuelta: Uno 1h
II viaje de ida: Cuatro y Ocho 8 h
II viaje de vuelta: Dos 2h
III viaje de ida: Uno y Dos 2h
1 1
3. (d) Dado que 2a = 10 se tiene que 10 a = 2, similarmente 10 a = 5. Luego, multipli-
1 1 1 1
cando, término a térnino se tiene 10 a + b = 10 de donde, + = 1
a b

205
206

4. (c) Cuando todos los ladrones tienen 6 piezas, sobran 5. Como para lograr que todos
tengan 7 piezas faltarı́an 8 piezas se concluye que son 13 ladrones. La cantidad de
piezas robadas fue 6 ∗ 13 + 5 = 83.

5. (c) Si n es la edad que tiene ahora Ana, cuando Marı́a tenı́a n años, eso hace 24 − n
años, Ana tenı́a n − (24 − n) = 2n − 24 años. Luego, dos veces 2n − 24 debe ser 24
es decir n = 18.

6. (a) Si S es el total de asambleistas entonces entre estudiantes y administrativos


forman un 16 + 15 = 30
11
del total de la asamblea. Ası́, los profesores representan 19
30 de
30∗570
S y entonces, S = 19 = 900.
33
7. (b) 3333 = (3,11)33 = 333 ∗ 1133 < 333 ∗ 2733 = 333 ∗ 399 = 3132 < 33 .
Además tanto 333 como 333 son menores que 333 por lo tanto el mayor de todos los
33
números es 33 .

8. (d) Por ángulos complementarios

m∠EGF = 180◦ − m∠BGE

además por ángulos entre paralelas

m∠EF G = 180◦ − m∠ABG


= 180◦ − m∠BGE.

9. (a) Como 555555 = 3 ∗ 5 ∗ 7 ∗ 11 ∗ 13 ∗ 37, la única manera de combinar los factores para
lograr expresarlo como producto de dos números de tres cifras es (3 ∗ 7 ∗ 37)(5 ∗ 11 ∗ 13).

10. (a) Como a + b + c + d − 5 debe ser múltiplo de 11, las opciones son

a + b + c + d = 5 lo que resulta imposible en vista de que los números a, b, c, d


son distintos.
a + b + c + d = 16, este caso produce las opciones a = 1, b = 2, c = 4, d = 9;
a = 1, b = 2, c = 5, d = 8; a = 1, b = 2, c = 6, d = 7; a = 1, b =
3, c = 5, d = 7; a = 1, b = 3, c = 4, d = 8; a = 1, b = 4, c = 5, d = 6;
a = 2, b = 3, c = 4, d = 7 y a = 2, b = 3, c = 5, d = 6
a + b + c + d = 27, este caso produce las opciones a = 3, b = 7, c = 8, d = 9
y a = 4, b = 6, c = 8, d = 9

En total hay 10 posibilidades diferentes, pero, además, cada combinación de 4 números


puede escribirse en 24 formas diferentes; lo que da un total de 240 combinaciones de
valores para a, b, c, y d.

11. (b) a+b+c


3 = 85, de donde a + b + c = 3 ∗ 85, d+l
2 = 95, de donde d + l = 2 ∗ 95.
Entonces a+b+c+d+l
5 = 3∗85+2∗95
5 = 89.
207

12. (d) En el primer movimiento cambiamos la segunda y la tercera flechas, por lo que
tendremos ↑↓↓↓↓↓.
En el segundo movimiento cambiamos la tercera y cuarta flechas para obtener ↑↓↑↑↓↓,
y finalmente cambiamos la cuarta y la quinta flecha con lo que nos queda la confi-
guración deseada. Ası́ el número mı́nimo de movimientos es menor o igual a 3. Desde
luego 2 movimientos no son suficientes ya que para llegar a la configuración deseada
hay que voltear la segunda y la quinta flechas, por las condiciones del problema estos
movimientos son independientes. Al hacer estos cambios, dos de las flechas restantes
se voltean por lo que es indispensable hacer un tercer movimiento.
13. (a) Veamos como son las terminaciones de las potencias de 7:
70 = 1, 71 = 7, 72 = 49, 73 = ..,43, 74 = ..,01, 75 = ..,07, 76 = ..,49, 77 = ..,43 de
aquı́ podemos ver que cada cuatro veces se empiezan a repetir las terminaciones y
observamos que 7 elevado a una potencia que es múltiplo de 4 termina en 01. Como
2000 es múltipo de 4 entonces 72000 termina en 01; la suma de los dos últimos dı́gitos
es 1.
4
14. (b) 101 = 0,03960. Después del punto decimal va un dı́gito, quedan 1999 por analizar
en el perı́odo (3960) que tiene 4 dı́gitos.
Ahora 1999 = 449 ∗ 4 + 3, es decir hay 499 perı́odos completos y luego debemos
considerar 3 dı́gitos del perı́odo siguiente, esto corresponde al 6.
15. (c) Si pinto con tres colores, tengo una sola manera:

3
1 2
2 1

Ası́ obtengo 4 maneras de pintar la pieza.


Si pinto con los cuatro colores, elijo tres para las varillas exteriores 1, 2 y 3:

3 3 4
1 2 1 2 1 2
4 1 2 4 2 1

3 3 3
y tengo tres maneras de completar el trabajo. Pero los colores de las tres varillas
exteriores se pueden elegir de cuatro formas diferentes, asi que con 4 colores hay
4 ∗ 3 = 12 maneras de pintar la pieza. En total 4 + 12 = 16 maneras de pintarla.
208

16. (b) Si tomamos las potencias de 2, tenemos únicamente 8 números: 22 , 23 , . . . , 29 .


Entre los números formados por las potencias de 3 tenemos: 32 , 33 , 34 , 35 , 36 es decir
5 números más. Las potencias de 4 ya están consideradas como subconjunto de las
potencias de 2. Como potencias de 5 tenemos sólo 3: 52 , 53 , 54 . Como potencias de
6 se tiene sólo 2: 62 , 63 . Las potencias de 7 que se deben considerar son: 72 , 73 , es
decir, dos números más. Las potencias de 8 y de 9 ya están consideradas, pues estos
números son potencias de 2 y 3, respectivamente. Números formados como potencias
de 10 tenemos: 102 , 103 y a partir del 10 y hasta el 31 solo aparece una potencia por
número que son 112 , 122 , . . . 312 , con excepción de los números 16, 25 y 27 que ha
están considerados. En total hay 40 de tales números
17. (c) Se construye la siguente tabla en la cual se indica los dı́as en que mienten M o
dicen la verdad V

Lun Mar Mier Juev Vie Sab Dom


Hiena M M M V V V V
Zorra V V V M M M V

Ahora se construye una tabla similar y se denota con S en caso de que estén diciendo
lo indicado ese dı́a y por N en caso de que no esté diciendo lo indicado, tenemos que

Lun Mar Mier Juev Vie Sab Dom


Hiena S N N S N N N
Zorra N N N S N N S

De las tablas se desprende que el dı́a que sucedió este encuentro fue el jueves.
18. (c) Como el niño 1 dijo no, los colores 2, 3 y 4 no pueden ser iguales, luego él ve dos de
un color y otro del otro. El segundo niño, si ve el tercero y el cuarto del mismo color
sabrá que su color es diferente al de 3 y al de 4, si los ve diferentes deberá contestar
que no. El tercer niño sabe ahora, debido a la respuesta no del segundo, que su gorro
es de color diferente al del cuarto, por lo que deberá contestar que si.
19. (a) De acuerdo con la tabla, son los impares:

R R R R R R R R R R
R V R V R V R V R V
R V R V R R R V R V
R V R V R R R V R V
R V R V R R R V R R
R V R V R V R V R R
R V R V R V R V R R
R V R V R V R V R R
R V R V R V R V R R
R V R V R V R V R V
209

20. (d) Las reglas para construir los triángulos son:

Los ángulos internos en un triángulo deben sumar 180◦


Los ángulos de la base de un triángulo isósceles deben ser congruentes.
el ángulo llano mide 180◦ .
Por lo tanto sólo se pueden construir 5 triángulos, como lo muestra la figura.

68
68
34 78
10
a partir de aquı́ no se puede
34
146 44
112 51 51
17 17

2000
21. (d) Para que sea entero n + 19 debe dividir a 2000. Ahora, como 2000 = 24 ∗ 53 ,
n + 19
tiene 20 divisores. Como n es positivo, debemos desechar para n+19 todos los divisores
menores que 20; estos son 1, 2, 4, 5, 8, 10, 16 ası́ que n + 19 puede tomar cualquiera
de los otros 13 valores posibles.

22. (b) Vemos que 1993 es impar, estos quedan en la columnas A, C o E. Los números que
caen en A son aquellos x tales que x − 1 es múltipo de 8, los que caen en C son tales
que x + 1 es múltiplo de 4 y los que caen en E son tales que x + 3 es múltiplo de 8.
Como 1993 − 1 = 249 ∗ 8 entonces 1993 cae en la columna A.

23. (b) Sea x el ancho de cada rectángulo y y su largo, entonces 3x = 2y que equivale a
4y = 6x, y el perı́metro es 5x + 4y = 176, que, sustituyendo, equivale a 5x + 6x = 176
de donde 11x = 176 y x = 16. De aquı́ se obtiene también que y = 24. Ası́ el
perı́metro de cada rectángulo congruente es 2x + 2y = 80.

24. (a) Sea x la edad de la madre, dado que al dividirse por 2, 3, 4, 6 y 8 deja residuo 1,
entonces x − 1 es divisible por 24. Los números menores que 100 con esta propiedad
son 25, 49, 73 y 97. De estos sólo el 25 cumple con la condición de ser múltiplo de 5.
Ası́ que la madre de Susana tiene 25 años y Susana tiene 5 años. Suman 30 años.
x
25. (c) De x 2 − + 7 = y se tiene que 2x 2 − x + 14 = 2y y como 2x 2 − x + 14 = 20
2
entonces 2y = 20 es decir y = 10 y ası́ 5y2 − 3 = 497.

26. (c) OE es bisectriz de ∠BOC ⇒ ∠C OE = ∠EOB = x. OD es bisectriz de ∠AOB ⇒


∠AOD = ∠DOB = y. OC ⊥ OA ⇒ ∠AOC = 90, de donde 2x = 2y − 90 es decir
y − x = 45◦ . Por lo tanto

∠DOE = ∠DOB − ∠EOB = 45◦


210

27. (b) La primera fila suma 10, la segunda fila agrega 4 a cada término de la primera;
suma 10 + 16, la tercera agrega 4 a cada término de la segunda; suma 10 + 16 + 16
y ası́ sucesivamente. Entonces la fila i suma 10 + 16(i − 1). La fila 50 suma, entonces
10 + 16 ∗ 49 = 794

28. (d) Como enero tiene 31 dı́as , si el primero de enero cae domingo, lunes o martes,
entonces habrá 5 martes durante el mes; si el primero de enero cae jueves viernes o
sábado, habrá 5 sábados durante el mes. La única manera en que haya exactamente
cuatro martes y cuatro sábados es que el primero de enero sea miércoles, en cuyo
caso el 23 de enero caerá en jueves.

29. (a) Como M es cuadrado de un cuadrado entonces M = x 4 con x entero. Como 6 es


4
factor de M, entonces M = 6k con k entero. Por lo tanto 6k = x 4 ⇒ M6 = k = x6 .
El menor valor de x, diferente de cero, que hace a k entero es x = 6 y, en este caso
4
k = 66 = 216.

30. (d) Se puede agrupar la suma en la siguiente forma:


       
1 1 2 1 3 1 1999 1
+ + + + + + ··· + + =1 + 1 + · · · + 1 = 1999
2 2 3 3 4 4 2000 2000

IV ciclo
1. (b) De a
b = c
d se tiene que ad = bc (1). Multiplicando (1) por bc se obtiene

adbc = b2 c2 ⇒ adbc = bc (2).


Multiplicando (1) por ad se obtiene: adbc = a2 d2 ⇒ adbc = ad (3).
√ √ bc + ad
Sumando (2) y (3): 2 adbc = bc + ad ⇒ adbc = .
2
2. (b) Los enunciados de los cofres de oro y plata dicen lo contrario, entonces uno de
ellos es verdadero. Si lo que dice el de oro fuera verdadero, entonces lo que dice el
de plata también serı́a verdadero, como solo uno dice la verdad, entonces lo que dice
el cofre de oro es falso, es decir, el retrato no puede estar en el cofre de oro. Por lo
dicho antes, entonces el que dice la verdad es el cofre de plomo, ası́, lo que dice el
de plata es falso, como el de plata dice que el retrato no está en él, entonces, al ser
esto falso es porque el retrato está en el cofre de plata.

3. (a) (ABC D) = (F ED) + (F BE) + (EC D) + (FAD) ⇒ 16 = 7 + 2·(4−FA)


2 + 4·2 4·FA
2 + 2 ⇒
9 = 4 − FA + 4 + 2FA ⇒ FA = 1.

4. (d) De 2a = 10 se sigue 2 = 101/a y de 5b = 10 se tiene 5 = 101/b . Luego, multiplicando


miembro a miembro: 2 · 5 = 101/a · 101/b , es decir 10 = 101/a+1/b y ası́, a1 + b1 = 1.
211

5. (b) Como 1998


2 = 999, uno de los sumandos de cada pareja es menor que 1000. Los
impares menores que 1000 son 1, 3, ..., 999 y éstos son 500. Además a cada impar
2k + 1 le podemos asociar el impar 1998 − (2k + 1) que cumple que la suma es 1998.
Por lo que hay exactamente 500 parejas.
6. (c) El mayor número divisible por 30, de cuatro cifras, es 9990, hasta ahı́ hay 9990÷30 =
333 números divisibles por 30. El mayor número divisible por 30 de menos de cuatro
cifras es 990, entonces la cantidad de números de menos de 4 cifras, divisibles por 30,
es 990 ÷ 30 = 33. Luego hay 333 − 33 = 300 números de 4 cifras divisibles por 30.
7. (d) 67 + a + 43 = a + c + 73, de donde c = 37. Luego, e + 37 + 43 = e + 73 + x,
por lo tanto x = 7.
8. (a) Tenemos que
1 n2 − 1 (n − 1)(n + 1)
1− = = ,
n2 n2 n2
entonces
    
1 1 1 1·3 2·4 3·5 (n − 1)(n + 1) n+1
1− 1− ··· 1 − 2 = · · ··· = .
4 9 n 2·2 3·3 4·4 n·n 2n
n+1 1001
Entonces = 2−1 + 2000−1 = ⇒ n = 1000.
2n 2000
9. (d) Como la razón del lado del menor al del mayor es 52 , digamos que el lado mayor
mide 5x y el menor mide 2x. El área del cuadrado mayor es 25x 2 y el área del
2 2
cuadrado menor es 4x 2 . Vemos que el área sombreada es 25x 4−4x = 214 x 2 . La razón
21 2
x 21
pedida es 4 2 = .
25x 100
4
10. (b) Tenemos = 0, 0396. Como el perı́odo (0396) tiene cuatro dı́gitos y 2000 es
101
múltiplo de 4, entonces la cifra que ocupa el lugar 2000 después de la coma es 6.
11. (d) Las reglas para construir los triángulos son
1. Los ángulos internos deben sumar 180◦
2. Los ángulos de la base de un triángulo isósceles deben ser congruentes.
3. El ángulo llano mide 180◦ .
Por lo tanto solo se pueden construir cinco, como lo muestra la figura.

68
68
34 78
10
a partir de aquı́ no se puede
34
146 44
112 51 51
17 17
212

12. (a) Los dı́gitos a, b, c pueden tomar los valores: 1, 2, 3, 5, 7. Para que sea divisible
por 2 o 5 estas cifras deben ir al final, mientras que las otras pueden ocupar otras
posiciones; verificando las posibilidades obtenemos que los números son: 132, 135, 175,
312, 315, 735. En total son seis.
13. (b) En cada viaje de ida van dos botes y de regreso va uno. Son necesarios tres viajes
de ida y dos de vuelta.
Ocho y Cuatro deben hacer juntos el viaje de ida, si no se tardará por lo menos
8 + 4 + 2 horas de ida y 1 + 1 horas en los regresos; en total 16 horas.
El viaje de ida de 8 y 4 no puede ser el primero porque si no uno de ellos debe hacer
un viaje de vuelta y un segundo viaje de ida (tardando más de 8 + 4 + 4 = 16 horas).
Tampoco puede ser el tercer viaje, porque entonces uno de ellos cruzará dos veces de
ida y una vez de vuelta. Ası́, los cruces de ida son Uno y Dos, Cuatro y Ocho, Uno y
Dos y los cruces de vuelta son uno con Uno y el otro con Dos (en cualquier orden).
Se tardan 2 + 8 + 2 + 1 + 2 = 15 horas. Una secuencia posible de viajes tardando 15
horas es:
I viaje de ida: Uno y Dos 2h
I viaje de vuelta: Uno 1h
II viaje de ida: Cuatro y Ocho 8 h
II viaje de vuelta: Dos 2h
III viaje de ida: Uno y Dos 2h

14. (a) Prolonguemos el segmento AC hasta que corte a BD y llamemos F al punto de


intersección.

30 30
C
4 4

3 3

D B
2 F 2

4ABD es equilátero, ya que AD = AB y m]DAB = 60◦ . Como DC = BC , ]C DB ≈


]C BD, por lo que ]ADC ≈ ]ABC . Luego 4ADC ≈ 4ABC . Entonces m ]DAC =
−→
m]BAC = 30◦ , es decir AC es bisectriz de ]DAB y √
AF es√altura de 4ABD, de
donde DF = BF = 2. Por lo tanto AC = AF − F C = 2 3 − 5.
15. (b) Puesto que F , H, J están alineados, entonces los triángulos JIH y HGF son
semejantes; entonces
FG HI x−7 3 49
= ⇒ = ⇒x= .
GH IJ 7 4 4
213

16. (d) Tenemos


1 1 1
+ = ⇒ 3(m + n) = nm ⇒ 3|n o 3|m.
n m 3
Si 3|n entonces n = 3k (k entero) y tendrı́amos m1 = 13 − 3k1 = k−1 3k ⇒ m = k−1 .
3k
3k
Como k − 1 y k son primos relativos, k−1 es entero positivo si y solo si (k − 1)|3, con
k ≥ 1, es decir k − 1 = 1 o k − 1 = 3, esto es k = 2 o k = 4. Tenemos entonces
que n = 6 o n = 12 y entonces m = 6 (cuando n = 6) y m = 4 (cuando n = 12).
Aquı́ se obtienen dos parejas (6, 6) y (12, 4). La simetrı́a del problema proporciona un
par más: (4, 12) (que se obtiene considerando el caso 3|m). Hay en total tres parejas
que cumplen lo pedido: (6, 6), (12, 4) y (4, 12).
17. (d) De acuerdo con la figura tenemos que
AD = AM + MD = MD + DN = MN.

Además C M = NB por ser lados opuestos de un paralelogramo.

C B
D

18. (a) Tenemos que el dı́gito de las unidades para cada uno de los sumandos es
(12 + 1) = . . . 2, (72 + 7) = . . . 6,
(22 + 2) = . . . 6, (82 + 8) = . . . 2,
(32 + 3) = . . . 2, (92 + 9) = . . . 0,
(42 + 4) = . . . 0, (102 + 10) = . . . 0
(52 + 5) = . . . 0, (112 + 11) = . . . 2
(62 + 6) = . . . 2,

Podemos observar que cada 5 sumandos los dı́gitos de las unidades se repiten y su
suma es 0. Como 1995 es un múltiplo de 5, el dı́gito de las unidades de la suma de
los sumandos hasta 1995 es 0 y si aumentamos los sumandos
(19962 + 1996) = . . . 2
(19972 + 1997) = . . . 6
(19982 + 1998) = . . . 2

Tenemos que el dı́gito de las unidades de la suma A es . . .+0+. . . 2+. . . 6+. . .+2 =
. . . 0.
214

19. (a) Cada una de las distancias entre los dı́gitos sucesivos del número telefónico es
la hipotenusa de un triángulo rectángulo con lados enteros. Usando
√ el√Teorema
√ √de
Pitágoras,
√ √ las distancias pueden fácilmente ser calculadas y son 2, 5, 5, 2,
10 y 5, respectivamente. La distancia total recorrida por el dedo es
√ √ √ √ √ √ √ √ √ √
2 2 + 3 5 + 10 = 2 2 + 3 5 + 2 5 = 5(3 + 2) + 2 2.

20. (c) Sea n = pα1 1 · · · pαk k . El número de divisores positivos es (α1 + 1) · · · (αk + 1). Para
que este número sea impar, α1 , . . ., αk deben ser pares y n es un cuadrado perfecto.
Como 442 = 1936 < 1998 < 2025 = 452 hay 44 cuadrados perfectos entre 1 y 1998.
Habrá 44 números naturales menores que 1998 con un número impar de divisores
positivos.

21. (a) Sea P = abc, Q = acb, entonces

P − Q = 100a + 10b + c − (100a + 10c + b) = 9(b − c).

Entonces b − c debe ser cuadrado perfecto no cero. Ası́, b − c = 1 o b − c = 4 o


b − c = 9. Las posibilidades se dan en la tabla:

b: 9 9 9 8 8 7 7 6 6 5 5
c: 0 5 8 4 7 3 6 2 5 1 4

b: 4 4 3 2 1
c: 0 3 2 1 0

En total hay 16 posibilidades, además en cada caso hay 9 posibilidades para selec-
cionar el dı́gito a (no puede ser 0, pues el número es de tres cifras), de modo que la
cantidad de valores posibles para P es 9 · 16 = 144.

EC b/2
22. (c) 4DEC ∼ 4ABC , en razón = , luego, los perı́metros guardan esa misma
BC a
proporción. Como 4ABC

es triángulo rectángulo 30◦ − 60◦ y se tiene que BC = a,
entonces AC = b = 23 a y AB = c = a2 . Luego
√ √
3
b/2 4 a 3
= = .
a a 4

23. (c) Puesto que n2 = p2α1 2α2 2


1 p2 . El número de divisores positivos de n es

(2α1 + 1)(2α2 + 1) = 143 = 13 · 11,

de donde 2α1 + 1 = 13, 2α2 + 1 = 11 y α1 = 6, α2 = 5. Por lo tanto el número de


divisores positivos de n3 = p3α1 3α2
1 p2 es (3α1 + 1)(3α2 + 1) = (3 · 6 + 1)(3 · 5 + 1) = 304.
215

24. (d) La distancia máxima está dada desde uno de los extremos de MN a uno de los
extremos de PQ. Ya que para cada punto fijo R sobre PQ, la distancia máxima de
R a los puntos de MN se alcanzará en M y N. Consideremos los puntos N y P y
dibujemos un triángulo con estos puntos y el vértice A. La√distancia de N a A es 1 y
de A a B es 2, por lo tanto la hipotenusa del 4ABP √ es 5. Obsérvese que 4NAP
es rectángulo, luego la hipotenusa del 4NAP es 6.

M N
A

B
P

25. (c) 2x + 5y = 2000 ⇒ 5y = 2000 − 2x = 2(1000 − x) ⇒ y es divisible por 2 y 1000 − x


es divisible por 5. Si 1000 − x = 5k, entonces 5y = 2 · 5k ⇒ y = 2k, x = 1000 − 5k,
estos pares son las soluciones. Pere queremos que tanto x como y sean positivos,
entonces tendremos k > 0 y 1000 − 5k > 0 ⇒ k < 200. Las soluciones que buscamos
son (1000 − 5k, 2k), con k = 1, ..., 199. En total hay 199 soluciones.
26. (d) Cada pieza tiene área 6. Podemos calcular el área de cada cuadrado formado, de
dos maneras: número de piezas por 6 o lado por lado (y los lados son de longitud
entera). Entonces el número de piezas por 6 es un cuadrado perfecto, ası́ que el número
de piezas en cada cuadrado es 6 por un cuadrado perfecto. Si formamos los cuadrados
más pequeños usamos

6 · 12 = 6, 6 · 22 = 24, 6 · 32 = 54,
2
6 · 4 = 96, 6 · 52 = 150, 6 · 62 = 216,
6 · 72 = 294, 6 · 82 = 384, 6 · 92 = 486,
6 · 102 = 600,

Para ver cuántas piezas se gastaron en total: hasta 9 cuadrados: 6 + 24 + 54 + 96 +


150 + 216 + 294 + 384 + 486 = 1710; hasta 10 cuadrados: 2310 (es más que 2000).
Entonces se pueden formar 9 cuadrados y sobran 290 piezas.
27. (b) Sean:
a : número de dulces que tomó A
b : número de dulces que tomó B
c : número de dulces que tomó C

Según lo que dijeron tenemos a = b + 2; b = a


2 y b = c − 5 y c es número par.
216

Si A dice la verdad, como el número de dulces es 13, 13 = a + b + c = (b + 2) + b + c


y ası́, 11 − c = 2b, luego 11 − c deberá ser número par y entonces c no puede ser
par, luego C miente.
Si B dice la verdad, 13 = a + b + c = 2b + b + c = 3b + c = 3(c − 5) + c, de donde
c = 7 y no puede ser par, por lo tanto C miente.
Como no puede haber dos mentirosos uno de A y B dice la verdad, esto nos lleva a
que C miente.

28. (a) Supongamos que se separa el total de piedras en dos grupos. La suma de los
pesos de un grupo A es a y la suma de los pesos del otro grupo B es b; ası́, el peso
total de las piedras es a + b. Al menos uno de esos valores es menor o igual que 1,
digamos que a ≤ 1. Ahora supongamos que se van quitando piedras del grupo B y
agregando al grupo A. Podrı́a suceder que el grupo A se mantenga con peso menor o
igual que 1 hasta el final y ası́ el peso total de las piedras no sobrepasarı́a a 1. Pero
la otra posibilidad es que en algún momento al quitar una piedra de B y ponerla en
A el peso de A sobrepase 1, en este caso, el peso de B tendrı́a que ser menor o igual
que 1 (esto por las condiciones del problema). Digamos que lo anterior sucede con la
piedra n que se pasa de B a A. Sean x1 , x2 , . . ., xn los pesos de las piedras que se
han pasado, hasta xn−1 tendrı́amos a + x1 + ... + xn−1 ≤ 1 (*), mientras que

b − (x1 + x2 + . . . + xn−1 ) − xn ≤ 1,

en cuyo caso
b − (x1 + x2 + . . . + xn−1 ) ≤ 1 + xn .
Como cada piedra pesa 1 kg o menos, xn ≤ 1, concluyéndose que b − (x1 + x2 + . . . +
xn−1 ) ≤ 2 (**). De (*) y (**) se tiene que a + (x1 + x2 + . . . + xn−1 ) + b − (x1 + x2 +
. . . + xn−1 ) ≤ 1 + 2 ⇒ a + b ≤ 3. Es decir el peso máximo no es mayor que 3.
Por otra parte, lo que enuncia el problema puede lograrse con un peso total de 3 kg;
en efecto, tome tres piedras de 1 kg de peso cada una (se cumple la condición de que
cada piedra no pesa más de 1 kg). Al dividirse en dos grupos, siempre queda un grupo
de una piedra, este grupo no pesa más de 1 kg. De esto y lo anterior se concluye que
el peso máximo es 3.

29. (b) Como m]APB + m]DPC = 90◦ tenemos que 4ABP ∼ 4PC D, por lo que
BP · PC = C D · AB = 60 y como BP + PC = 16 se tiene que BP = 10 y PC = 6 o
BP = 6 y PC = 10.
√ √ √ √
AP = 52 + 102 = 5 5 y PD = 122 + 62 = 6 5 y (APD) =
En el primer caso √
30·5
2 =√ 75; AD = 305 y perı́metro 4APD es mayor que 41. En el segundo caso
√ √ √ √
AP = 52 + 62 = 61 y PD = 122 + 102 = 2 61 y (APD) = 61, AD = 305 y
perı́metro 4APD es menor que 41.

30. (c) Primero debemos llegar a un múltiplo de tres mayor que 97 y de ahı́ bajar, apretando
B consecutivamente hasta 97. El menor múltiplo de 3 mayor que 97 es 99 y como
217

97 = 99 − 2 = 11 · 32 − 2, debemos ver la forma de llegar a 11 más rápido. Esta forma


es
A B A B B A B
1 → 3 →2 → 6 → 5 → 4 → 12 → 11
A A B B
y de ahı́ 11 → 33 → 99 → 98 → 97. Ası́, la secuencia más corta requiere de 11 teclas.

Segunda eliminatoria
III ciclo
SELECCIÓN

1. (b) La afirmación 1 no puede ser verdadera porque en ese caso las otras serı́an falsas
contradiciendo lo que dice esa afirmación, lo mismo sucederı́a si la 2 o la 4 fueran
verdaderas. La única que puede ser verdadera es la 3 porque en ese caso las otras
tres son falsas y habrı́a exactamente tres falsas.

2. (b) Para que un número sea divisible por 9, la suma de sus dı́gitos tiene que ser
múltiplo de 9. Si el número tiene 10 dı́gitos y está formado por solo ceros y unos,
entonces, para ser divisible por nueve debe tener 9 nueves (y solo un 0), el primer
dı́gito no puede ser 0 porque en ese caso el número se considera de nueve dı́gitos.
Considerando un cero en cada una de las demás posiciones vemos que hay nueve
números que cumplen las codiciones pedidas.
5p 10p
3. (d) 2 = 3u ⇒ 5p = 6u ⇒ 10p = 12u = 3(4u) ⇒ 3 = 4u.

4. (b) Hay 2 tipos de ensalada, 4 de carne y 3 de postre. Juan solo escogerá entre 1
ensalada, 3 carnes y 3 postres, el número de posibilidades que tiene para formar su
almuerzo es 1 · 3 · 3 = 9. De modo análogo, el número de posibilidades para Julia es
1 · 4 · 2 = 8; para Jorge 2 · 3 · 2 = 12 y para Jacinta 2 · 2 · 3 = 12. Quien tiene menos
posibilidades es Julia.

5. (c) m]AC P = 90◦ − α ⇒ m]QHC = 90◦ − (90◦ − α) = α ⇒ m]BHC = 180◦ − α.

6. (d) A cada uno le corresponden 8/3 de litro de leche. Ası́, Daniel aporta un tercio de
litro y Raquel siete tercios de litro. A Daniel le tocan 100 colones y a Raquel 700.

7. (b) Basta con usar 7 veces la pesa de tres kilos para obtener 21 kilos, luego la pesa
de 19 kilos para que queden dos kilos y luego 4 veces la pesa de tres kilos. En total
se gastarán 16 colones.

8. (d) Sean x, y, z los números, x + y + z = 205, x


y = 25 , y
z = 34 , entonces z
y = 4
3 y
x 2
3
y
z = 5
4 ⇒ x
z = 10 ⇒ x
z + y
z = 34 + 103 ⇒ x+y
z = 21
20 . Pero z = 205 − (x + y), entonces
y 3
x+y 21
205−(x+y) = 20 ⇒ x + y = 105.
218

9. (d) Tenemos

1·2·4+2·4·8+3·6·12+···+100·200·400 2·4(1·1·1+2·2·2+3·3·3+···+100·100·100) 8
1·3·9+2·6·18+3·9·27+···+100·300·900 = 3·9(1·1·1+2·2·2+3·3·3+···+100·100·100) = .
27

10. (a) m]DFA = m]EF C (opuestos por el vértice) y m]F EC = m]F DA (ambos
rectos), entonces 4ADF ∼ 4EC F (por A–A). Luego

EC FC 2a FC 2a a
= ⇒ = ⇒ FC = ⇒ DF =
AD DF a a − FC 3 3
a
3 ·a a2
y, por lo tanto, (ADF ) = 2 = 6 .
1a
b+b − b = b ⇒ 2b = 2 b ⇒
11. (b) Se tiene a+b a a a+b a a
2b + 2b
b
= 2 ab ⇒ 2b + 21 = 2 ab ⇒ 3a
2b = 1
2
⇒ ab = 13 . Es decir, 14 < ab < 12 .

12. (c) Tenemos que

102000 − 2000 = 1 0| . . . {z
000000} − 2000 = 9| . . . {z
999999} 8000
2000 ceros 1996 nueves

De manera que la suma de las cifras del número es 1996 · 9 + 8 = 17972.

13. (a) Dado que AB = BC , entonces el triángulo ABC es isósceles y, puesto que

m]BAC = 30◦ y AC = 4 3, entonces AB = BC = 4. Por ángulos alternos internos
entre paralelas tenemos que m]C AD = 30◦ . Tracemos ahora la altura C M.

B C
30

30
30
A D
M

√ √ √
El 4AC M √es 30 − 60 − 90 y su hipotenusa mide 4 3, por lo que AM = 4 3 · 23 = 6
√ √
y, C M = 4 2 3 = 2 3. También 4C DM es 30 − 60 − 90 y, como C M = 2 3, entonces
√ √
MD = 2. Ası́, el área del trapecio es 8+4
2 · 2 3 = 12 3.

14. (a) Podemos escribir

x 3 − y3 (x − y)(x 2 + y2 + xy)
= .
xy xy

Si este es un número entero entonces xy divide a (x −y)(x 2 +y2 +xy) y, en particular,


x|(x − y)(x 2 + y2 + xy). Como x es primo entonces x|(x − y) ó x|(x 2 + y2 + xy).
219

– Si x|(x − y), entonces x − y = lx (con l entero) ⇒ y = x(1 − l) ⇒ x|y, pero esto


no es posible pues y es primo, x 6= y y x 6= 1 (por ser x primo).
– Si x|(x 2 + y2 + xy) entonces x 2 + y2 + xy = mx (con m entero) ⇒ y2 = x(m − x − y)
⇒ x|y2 y como x es primo, entonces x|y, pero ya vimos que esto no es posible.
Concluimos que no hay ningún par (x, y) con las condiciones pedidas por el enunciado.

15. (a) Observemos la siguiente figura, en ella se ha trazado el segmento DE que une los
pies de ambas medianas.

E D

B A

Puesto que DE es paralelo a AB, entonces 4ABC ∼ 4DEC y como AC = 2DC ,


entonces el área de 4DEC es la cuarta parte del área de 4ABC . De aquı́ se tiene que
el área del trapecio ADEB es igual a 18 − 184 = 272 . Por otra parte, por propiedades de
la mediana, AF = 2EF y BF = 2DF y, puesto que las medianas son perpendiculares,
(ADEB) = AE·BD 2 = 3DF2·3EF = 92 DF · EF = 272 ⇒ DF · EF = 3 (*).
Además, por Pitágoras en 4ABF y puesto que AB = 5, se tiene 25 = AF 2 + BF 2 =
4EF 2 + 4DF 2 ⇒ EF 2 + DF 2 = 254 (**). De (*) y (**) se obtiene que los valores de
EF y DF son 2 (el mayor) y 32 (el menor). Digamos que DF = 2 y EF = 23 . Por
√ √
Pitágotas
√ en 4ADF : AD = 13 √ y, entonces AC = 2 13. Por Pitágotas en 4C EF :
C E = 12 73 y, entonces BC = 73. Si consideramos DF = 23 y EF = 2, obtenemos
√ √ √ √
AC = 73 y BC = 2 13. En todo caso, AC + BC = 73 + 2 13.

DESARROLLO

1. Se puede llegar a la posición 25413 en cinco jugadas, ası́:


A C C A C
12345 → 51234 → 51423 → 51342 → 25134 → 25413

Veamos que no es posible llegar a esa posición en 4 jugadas o menos. En la siguiente


tabla se presentan todas las posiciones que se pueden lograr con cuatro jugadas o
menos; en la primera columna aparece la posición inicial, en la segunda columna apa-
recen las dos posiciones que se pueden obtener a partir de la inicial, en la tercera las
cuatro posiciones que se pueden obtener a partir de las dos anteriores, estas producen
220

8 posibilidades para la tercera jugada y a la vez estas 8 producen 16 posibilidades


para la cuarta jugada. Se puede observar que en ningún momento aparece la posición
25413.

23451
34512
34251
45123
24531
45312
45231
51234
23514
35142
35214
51423
25134
51342
51234
12345
53412
34125
34512
41253
54132
41325
41532
12534
53124
31245
31524
12453
51234
12345
12534

2. Como las cifras son múltiplos de tres, entonces x es divisible por 3, este será uno de
sus tres divisores primos, entonces x = 3a pb qc donde p y q son primos, p < q y
mayores que 5 (no pueden ser 2 porque el número serı́a par, ni 5 porque terminarı́a
en 5 y 5 no es divisible por 3). Además, b = c = 1, pues, para b > 1, el número
más pequeño serı́a x = 3 · 72 · 11 = 1617 > 1000. Ası́, las posibilidades son x = 3pq
ó x = 9pq. Como x es de tres cifras entonces 100 ≤ x ≤ 1000. Caso x = 9pq,
solamente 9 · 7 · 11 = 693 sirve. Caso x = 3pq: • Si p = 7, entonces x = 21q y
como x ≤ 1000 ⇒ 21q ≤ 1000 ⇒ q ≤ 47. La última cifra de x será igual a la última
de q, de modo que los posibles son 21 · 13 = 273 (no sirve), 21 · 19 = 399 (sirve),
21 · 23 = 483 (no sirve), 21 · 29 = 609 (sirve), 21 · 43 = 903 (sirve). • Si p = 11, entonce
x = 33q ⇒ q < 31. Si q termina en 5, 7 ó 9, la última cifra de x no serı́a múltiplo de
3; como q debe ser primo menor que 31, mayor que 11 y terminado en 1 ó 3, entonces
las únicas posibilidades son 33 · 13 = 429 (no sirve) y 33 · 23 = 759 (no sirve). • Si
p = 13, entonces x = 39q ⇒ q < 26; las posibilidades son 39 · 17 = 663 (sirve),
39 · 19 = 741 (no sirve) y 39 · 23 = 897 (no sirve). • Si p = 17, entonces x = 51q ⇒
q < 20. La única posibilidad es 51 · 19 = 969 (sirve). De modo que hay seis números
con las condiciones pedidas: 399, 609, 663, 693, 903, 969.

3. Tracemos por P una recta paralela a AB que corta a AD y BC en X e Y respectivamente


(vea la figura).
221

A B
X P Y

D C

Los triángulos APX y AC D son semejantes y, por lo tanto, AD


AX
= DC
XP
. Además, los
triángulos BPY y BDC son semejantes y, por lo tanto, BC = DC . Pero como AB,
BY YP

X Y y DC son paralelas y por el Teorema de Thales podemos decir que AD AX


= BCBY
.
Entonces DC = DC , de donde X P = Y P. De aquı́ es evidente que PQ corta a AB y
XP YP

C D en sus puntos medios.

IV ciclo
SELECCIÓN

1. (d) Si se toma el diámetro como base constante, entonces el área será máxima cuando
la altura lo sea, en ese caso cuando la altura es r, por lo que el área máxima es r 2 .
1 1 1
2. (b) Observamos que k(k+1) = k − k+1 , entonces
 
4999 1 1 1
2000 − + + ··· +
2·3 3·4 1999 · 2000
 
1 1 1 1 1 1
= 4999
2000 − − + − + · · · + −
2 3 3 4 1999 2000
 
4999 1 1 4999 999
= − − = − = 2.
2000 2 2000 2000 2000

3. (b) Tenemos
q q q
√ √ √ √ √
y + xy + 2y x = y 1 + x + 2 x = y (1 + x)2
√ √ √ √
= y(1 + x) = y + xy ⇒
√ √ q √
√ √
y + xy + x = x + y + xy + 2y x

4. (d) Observamos que el k−ésimo término de la fila número n es 3k−1 · 2n−1 . Por otra
parte, la fila n tiene 101 − n términos (la primera tiene 101 − 1 términos, la segunda
101 − 2 = 99, etc.), de manera que la fila número 51 tiene 101 − 51 = 50 términos.
Entonces el término número 50 de la fila 51 es 350−1 · 251−1 = 349 · 250 .
222

5. (c) Como AB es diámetro, entonces


m]ADB = 90◦ , m]C DB = 75◦ − β (pues subtiende el mismo arco que ]BAC y
m]BAC = 75◦ − β). Entonces

90◦ = m]ADC + m]C DB = α + 75◦ − β ⇒


α − β = 15◦ ⇒ β = α − 15◦ .

6. (c) Dado que p tiene cuatro cifras se tiene que 59p = 10000n + r (con r el número
formado por las últimas cuatro cifras de 59p) ⇒ 3r = 10000k + d (con d el número
formado por las últimas cuatro cifras de 3r) ⇒ 113d = 10000e + f (con f el número
formado por las últimas cuatro cifras de 113d).
Ası́:

f = 113d − 10000e = 113(3r − 10000k) − 10000e


= (113)(3)r − (113k + e)10000
= (113)(3)(59p − 10000n) − (113k + e)10000
= (113)(3)(59)p − (113k + e + (3)(113)n)1000
= p + (2p − (113k + e + (3)(113)n)10000

Dado que f tiene cuatro cifras y p también la parte derecha en la expresión final debe
ser cero y f debe coincidir con p.

7. (d) Al comienzo lo conoce una persona, esta se lo cuenta a p personas, cada una de
estas a p + 1 personas, ası́ que hay otras p(p + 1) que lo conocen y cada una de
estas, a su vez, se lo cuentan a p + 2; ası́, habrá otras p(p + 1)(p + 2) que lo conocen,
esto da un total de

1 + p + p(p + 1) + p(p + 1)(p + 2) = (p + 1)(p2 + 3p + 1).

personas que conocen el secreto.

8. (d) Como cada uno debe recibir al menos un confite y Elizabeth debe recibir tres
entonces el problema se reduce a repartir cuatro confites entre Hugo, Mario y Eduar-
do, sabiendo que Eduardo puede recibir uno o ninguno de esos cuatro confites. Las
opciones son, en el orden Eduardo, Hugo y Mario.

(1, 0, 3), (1, 1, 2), (1, 2, 1), (1, 3, 0), (0, 4, 0), (0, 3, 1), (0, 2, 2), (0, 1, 3) y (0, 0, 4)

en total 9 formas.

9. (d) Como x, y son impares, entonces existen enteros k y l tales que x = 2k + 1,


y = 2l + 1, entonces

T = 32k+1 + (2l + 1 − 1)2 z = 3 · (3k )2 + 4l2 z.


223


Luego, T − 3 = 3 (3k )2 − 1 + 4l2 z = 3(3k + 1)(3k − 1) + 4l2 z. Como 3k es impar,
entonces 3k + 1 y 3k − 1 son pares y, por lo tanto, (3k + 1)(3k − 1) es múltiplo de 4.
entonces T − 3 = 4S, con S entero. Es decir, T = 4S + 3 y, entonces, al dividir T
entre 4 se obtiene como residuo 3.

10. (a) Observe que


1 1 1
(1 − )(1 − ) . . . (1 − 2 ) = (1 − 12 )(1 + 21 )(1 − 13 )(1 + 31 )...(1 − k1 )(1 + k1 )
4 9 k
1 3 2 4 k −1 k +1 1 k +1 k +1
= · · · · ... · · = · =
2 2 3 3 k k 2 k 2k
Si k es par entonces la fracción es canónica y m = k + 1, n = 2k y, entonces,
m + n = 3k + 1.

11. (c) Sean x1 , . . . , x10 los números a colocar en los triangulitos, del modo como se
muestra en la siguiente figura.

x3 x4 x5
x2 x6
x1 x7
x10 x8
x9

Tenemos que x1 + · · · + x10 = 66 y x2 + x4 + x6 + x8 + x10 = 42. S = x1 + x2 + x3 =


x3 + x4 + x5 = x5 + x6 + x7 = x7 + x8 + x9 = x9 + x10 + x1 . Entonces

5S = (x1 + x2 + x3 ) + (x3 + x4 + x5 ) + (x5 + x6 + x7 ) + (x7 + x8 + x9 ) + (x9 + x10 + x1 )


= (x1 + · · · + x10 ) + x1 + x3 + x5 + x7 + x9 = 66 + 24 = 90 ⇒ S = 18.

(pues x1 + x3 + x5 + x7 + x9 = 66 − (x2 + x4 + x6 + x8 + x10 ) = 66 − 42). Entonces


5S = 90 y, por lo tanto, S = 18.

12. (b) Se prolonga C M hasta el punto L sobre AB. Los triángulos DC M y ALM son
congruentes y por lo tanto AL = 6; ası́, si h es la altura trazada desde el vértice C
entonces sen 30◦ = 14
h
, es decir, h = 14 sen 30◦ = 7. El área es 8+6
2 · 7 = 49.

6
D 30
C
7
M h

L B
A 8
224

13. (c) Digamos que, para llegar al lugar en que el perro alcanza al conejo, el perro
necesita y de sus saltos y el conejo x de los suyos adicionales a los que le lleva de
ventaja al perro, entonces x5 = y2 (el tiempo que tarda el perro en dar 2 saltos es el
mismo que tarda el conejo en dar 5); además, 50+x 8
y = 3 (la distancia recorrida por el
perro corresponde a y de sus saltos, la recorrida por el conejo es 50 + x, la misma
distancia recorre el conejo en 8 saltos que el perro en 3). De estas dos ecuaciones se
tiene que y = 300.
14. (a) Observemos la siguiente figura.

M
L
P

A B
N


En primer lugar, como 4ABC es equilátero de lado 2 3, entonces su altura es
√ 2 3 √
3 · 2 = 3. Por otra parte, se tiene que
(ABC ) = (APC ) + (APB) + (C PB),
es decir
√ √ √ √ √
2 3·3 2 3 · PN 2 3 · PL 2 3 · PM 2 3 · (PN + PL + PM)
= + + = .
2 2 2 2 2
Entonces PN + PL + PM = 3.
√ √ √ √ √ 2 √
15. (b) Observe que n + 1 > n, entonces n + 1 + n > 2 n ⇒ √n+1+ < √1 .
n n
Racionalizando el lado izquierdo de la última desigualdad se tiene que
√ √ 1
2( n + 1 − n) < √ (*)
n
√ √
De n − 1 < n, procediendo como antes se obtiene que
√ √ 1
2( n − n − 1) > √ (**)
n

De modo que
1 1 1
√ + √ + ... √
1 2 10000
√ √ √ √ √ √
>2( 2 − 1 + 3 − 2 + · · · + 10001 − 10000) (por (*))

=2( 10001 − 1) > 2(100 − 1) = 2 · 99 = 198.
225

Por otra lado,


1 1 1
√ + √ + ... √
1 2 10000
√ √ √ √ √ √
<1 + 2( 2 − 1 + 3 − 2 + · · · + 10000 − 9999) (por (**))

=1 + 2( 10000 − 1) = 1 + 2(100 − 1) = 1 + 2 · 99 = 199.

De manera que 198 < M < 199.

DESARROLLO

1. En primer lugar observamos que ningún par (0, y) ó (x, 0) puede ser solución; tampoco
1 1
pares (x, x) ó (x, −x). De la primera ecuación tenemos x+y
x2y
= −12 ⇒ xy + x 2 = 2
−1

1 1 15
(*). Procediendo de modo análogo con la segunda ecuación tenemos y2
− xy = 4.
1 1 13
Sumando estas dos nuevas ecuaciones tenemos x2
+ y2
= 4. Sea a = x1 , b = 1
y.
13 1
Tenemos a2 + b2 = (**) y, en (*), ab + a2 = −1
4 2 ⇒ b = −a − 2a . Sustituyendo esto
1
 2
en (**) tenemos a2 + −a − 2a = 134 ⇒ 8a4 − 9a2 + 1 = 0 ⇒ (8a2 − 1)(a2 − 1) = 0
⇒ a = ± 2√1 2 , a = ±1.
Si a = 1, entonces b = 2 ;
−3
si a = −1, entonces b = 32 ; si a = 1

2 2
, entonces
√ √
−5 2 5 2 1
b= 4 ; si a = √ ,
−1
2 2
entonces b = 4 . y b = y1 , entonces, las parejas
Como a = x
√ √ √ 2√2
(x, y) que resuelven la ecuación son (−1, 32 ), (1, −2 −2 2
3 ), (2 2, 5 ) y (−2 2, 5 ).

2. Puesto que las medianas AQ y BP son congruentes, entonces los triángulo 4DEF y
4AOB son isósceles (O es el punto donde concurren las medianas) y, por propiedad
de la mediana, OR = 12 ÷ 3 = 4.

F
P Q

D E
x
A y T R B

Tracemos el segmento DT , perpendicular a AB, entonces los triángulos AOR y ADT


son semejantes; ası́, si DT = x y AT = y, tenemos 43 = yx y, aplicando Pitágoras
en 4ADT , x 2 + y2 = 4. De estas dos ecuaciones se obtiene que x = 58 , y = 65 .
De este modo, la base de 4DEF es 6 − 2y = 6 − 125 = 185 cm y la altura es
12 − 2 − x = 10 − 58 = 425 cm y el área buscada es 378 2
25 cm .
226

3. (a − 1)(b − 1) divide a ab − 1 si

ab − 1
= k,
(a − 1)(b − 1)

con k entero. Se dividirá el análisis en dos casos:


 Si a = b :
ab − 1 a2 − 1 a+1
k= = 2
=
(a − 1)(b − 1) (a − 1) a−1
lo cual solo es posible si a + 1 = kl y
a−1 = l, restando estas dos expresiones se obtiene 2 = l(k −1), esta última expresión
solo se cumple en los casos l = 2, k = 2 ó l = 1, k = 3. El primero de ellos lleva al
caso a = 3, b = 3; el segundo al caso a = 2, b = 2.
 Si a 6= b :

ab − 1 ab − 1
k= =
(a − 1)(b − 1) ab − (a + b) + 1
a+b−2 a−1 b−1
=1+ =1+ +
ab − (a + b) + 1 (a − 1)(b − 1) (a − 1)(b − 1)
1 1
=1+ + .
b−1 a−1
1 1
Si a = 2 entonces k = 2 + b−1 esto llevarı́a a que b−1 es entero lo cual lleva a
b = 2 que no aporta ninguna solución.
Si a = 3 entonces k = 23 + 1
b−1 , esto llevarı́a a que 1
b−1 debe ser un múltiplo impar
de 21 lo cual no es posible.
Finalmente si a ≥ 4 no aparecen casos nuevos pues

ab − 1 1 1 1 1
=1+ + <1+ + < 2.
(a − 1)(b − 1) b−1 a−1 3 b−1

Tercera eliminatoria (final)


III ciclo
Primer dı́a
1. Como Dora no fue a la panaderı́a y a este lugar fueron tres de ellas, entonces a la
panaderı́a fueron Ester, Margarita y Lucı́a. Además, Ester fue a la carnicerı́a. Ası́, Ester
fue a la carnicerı́a y a la panaderı́a. Margarita fue al Banco y a la panaderı́a. Como
la señora Pérez no fue a ningún lugar a donde fueron Lucı́a y la sra. Torres, entonces,
necesariamente Dora es Pérez. Lucı́a no es ni Torres ni Pérez, entonces es Martı́nez
o Gómes.
227

Si el apellido de Lucı́a fuera Martı́nez, entonces, tendrı́amos que Ester serı́a Torres (no
es Pérez, no es Martı́nez y tampoco puede ser Gómez por el segundo punto), entonces
Margarita serı́a Gómez, pero esto no puede suceder, porque entonces habrı́a ido a tres
lugares (la panaderı́a, el banco y la carnicerı́a). Luego, el apellido de Lucı́a es Gómez,
entonces Lucı́a fue a la carnicerı́a y a la panaderı́a. En este caso, Ester es Torres y
Margarita es Martı́nez.

2. Sea m]DAB = y◦ , como ABC D es paralelogramo entonces m]DC B = y◦ . Por


la misma razón, m]ABC = 3x ◦ + 15◦ . Entonces 2y + 6x + 30 = 360 (*). Por otra
parte, puesto que BC = C E, entonces m]C BE = m]C EB, además, m]C BE =
m]BC D = y, puesto que son alternos internos entre paralelas; de este modo, en el
cuadrilátero ADC E se tiene que

y + 3x + 15 + y + x + y = 360 ⇒

4x + 3y + 15 = 360 (**). De (*) y (**) se obtiene que m]DAB = y◦ = 75◦ .

3. Sea abc el número dado (a, b, c diferentes entre sı́). Los otros tres números que se
obtienen son, entonces, ab, ac y bc. La suma de ellos tres es la mitad del número
inicial, esto es ab + ac + bc = 21 abc ⇒

1
10a + b + 10a + c + 10b + c = (100a + 10b + c).
2
Luego 40a + 22b + 4c = 100a + 10b + c, es decir 12b + 3c = 60a ⇒ 4b + c = 20a.
Puesto que a, b, c son cifras, entonces el valor máximo de 4b + c es 45, por lo que
a = 1 ó a = 2.
Si a = 1, tenemos 4b + c = 20 y los posibles valores de b y c son b = 3 y c = 8
ó b = 4 y c = 4 ó b = 5 y c = 0. De aquı́ se obtienen tres números con las
condiciones pedidas: 138, 144, 150.
Si a = 2, tenemos 4b + c = 40 y los posibles valores de b y c son b = 8 y c = 8
ó b = 9 y c = 4. Se obtienen ası́ otros dos números con dicha propiedad: 288 y 294.

Segundo dı́a
4. Trace una paralela a AB por C que se interseca con la prolongación de AM en N.
Los triángulos 4ABM 4C MN son semejantes, con razón de semejanza ρ. Si se traza
una perpendicular a AB por M, que interseca a C N en P y a AB en Q entonces si
|MP| = h1 y |MQ| = h2 , h2 /h1 = ρ y h1 + h2 = h donde h es la altura del 4ABC
ρ
es decir h2 = ρ+1 h

|AB|h2 |AB|h ρ ρ
Area(4ABM) = = = Area(4ABC )
2 2 ρ+1 ρ+1
228

5. Sea y la altura de 4PC D y h la altura de 4C PQ.

Q
h

B C
P
y

A
D

(AD+BP)y
De la información dada se tiene que AD · y = 37, 2 = 29. Por lo tanto
PB · y = 21.
Puesto que m]QPB = m]C PD (opuestos por el vértice) y m]PC D = m]QBP
(alternos internos entre paralelas), entonces 4BPQ ∼ 4C PD.
h y BP · y
Por lo tanto = ⇒h= . Luego,
BP PC PC

PC · h PC BP · y BP · y 21
(C PQ) = = · = = = 10, 5.
2 2 PC 2 2

6. Observamos que 693 = 9 · 7 · 11, de manera que P tiene que ser múltiplo de 9, de 7
y de 11.
Para que sea múltiplo de 9, la suma de sus dı́gitos tiene que ser múltiplo de 9:

x + y + z + 19 = 9k (*)

Usando la regla de divisibilidad por 11, tenemos que

x − y + z − 5 = 11m (**)

Puesto que x, y, z son dı́gitos entonces son enteros entre 0 y 9. Ası́, el mayor valor
de la suma x + y + z + 19 es 46 y como k es entero, entonces k no puede ser mayor
que 5. El menor valor de esa suma es 19, entonces k no puede ser menor que 3, por
lo que k = 3 ó 4 ó 5.
Restando (**) de (*) se tiene 2y+24 = 9k −11m (***). Si k = 4, entonces x +y+z = 17.
Por (***), 2y + 24 = 36 − 11m ⇒ 2y = 12 − 11m ⇒ 11m = 2(6 − y). Luego 11 divide
a y − 6, como y es dı́gito, necesariamente y − 6 = 0, es decir, y = 6. De aquı́ se tiene
que m = 0, es decir x − y + z = 5 (por (**)) y, entonces, x + z = 11 ⇒ z = 11 − x.
229

Ahora, tenemos que 33xy49z es múltiplo de 7; como y = 6, entonces 33x649z = 7t.


Es decir:

7t = 33 · 105 + x · 104 + 649 · 10 + z


7t = 3 · 11 · 105 + x · 104 + 11 · 59 · 10 + 11 − x
7t = 3 · 11 · 105 + 11 · 909 · x · 104 + 11 · 59 · 10 + 11
7t = 11(300591 + 909x)
7t = 11 · 9 · (33399 + 101x)

Esto es, 7 divide a 11 · 9 · (33399 + 101x), como 7 es primo con 11 y con 9, entonces 7
tiene que dividir a 33399 + 101x. Probando con valores de x (como x + z = 11 y ellos
son dı́gitos, x no puede ser ni 0 ni 1), se tiene, para x = 4, que

33399 + 101x = 33399 + 101 · 4 = 33803

que es múltiplo de 7. En este caso, z = 11 − x = 11 − 4 = 7. El número P = 3346497


es un múltiplo de 693.

IV ciclo

Primer dı́a

1. Basta con observar que m∠EOA = π/6, luego EA = 1/ 3. Aplicando el teorema de
Pitágoras se tiene que

q r
p √ 40 √
BH = 4 + AH 2 = 4 + (3 − 1/ 3)2 = − 2 3.
3

Si x = BH se tiene
r
40 √
x= −2 3
3
40 √
x2 = −2 3
3 √
2
3x − 40 = −6 3
9x 4 − 240x 2 + 1492 = 0
230

C D

B
O A

2. La suma total de los 16 números es 136, de modo que en cada grupo la suma debe ser
68. Además, la suma total de los cuadrados es 1496, por lo tanto la suma de cuadrados
en cada grupo debe ser 748.
Observemos que los 16 números se puede escribir como

2k1 − 1, 2k1 , 2k2 − 1, 2k2 , ..., 2k8 − 1, 2k8

donde k1 = 1, k2 = 2, ..., k8 = 8.
Entre los 8 números de cada grupo debemos tomar un número par de números impares
puesto que su suma debe ser 68 que es par. Tomemos para el primer grupo los 8
números

2ka , 2kb , 2kc , 2kd , 2ke − 1, 2kf − 1, 2kg − 1, 2kh − 1

Como son 8 números diferentes, entonces los k’s son diferentes, ası́, uno es 1, otro es
2, ..., otro es 8. Por lo tanto ka + kb + · · · + kh = 1 + 2 + · · · + 8 = 36. Por otra parte,
la suma de los cuadrados es 748, es decir
2
748 = (2ka )2 + (2kb )2 + (2kc )2 + (2kd )2 + (2ke − 1)2 + (2kf − 1)2 + 2kg − 1 + (2kh − 1)2 .

Desarrollando los cuadrados y simplificando concluimos que

186 = ka2 + kb2 + kc2 + kd2 + ke2 + kf2 + kg2 + kh2 − (ke + kf + kg + kh ).
231

Pero ka2 + kb2 + kc2 + kd2 + ke2 + kf2 + kg2 + kh2 = 12 + 22 + · · · + 82 = 204 y, por lo tanto,

ke + kf + kg + kh = 18.

Haciendo ke = 2, kf = 3, kg = 5, kg = 8 (suman 18) y ka = 7, kb = 6, kc = 4, kd = 1,


tenemos un primer grupo formado por los números 2, 3, 5, 8, 9, 12, 14, 15 y un segundo
grupo formado por 1, 4, 6, 7, 10, 11, 13, 16. Esta distribución satisface las condiciones
pedidas. Otras distribuciones válidas pueden ser obtenidas dando otros valores.

3. Suponga que no hay ningún Pi , 1 ≤ i ≤ k + 1, cuadrado perfecto entonces, separando


las potencias pares de los factores que sea posible, cada número puede escribirse
como producto de un cuadrado perfecto y un número que no es cuadrado perfecto.

Y
n
I (i)
P1 = C12 ai1
i=1
..
.
Y
n
(i)
2 I
Pk+1 = Ck+1 aik+1
i=1

Donde Ij (i) puede ser 0 o 1 dependiendo de si ai es o no es factor en la parte que no


es cuadrado perfecto en Pj (Ij (i) = 0 significa que ai no aparece en el producto).

Luego se procede como sigue: dado que hay k +1 números y k factores entonces algún
factor debe aparecer en dos o más números; digamos, sin perder generalidad que es
a1 . Nuevamente sin perder generalidad suponga que uno de esos números es P1 . Se
sustituyen cada número Pj donde aparezca a1 por el producto P1 Pj , si ninguno de
los números obtenidos es cuadrado perfecto se repite el procedimiento usando k − 1
factores primos, todos excepto a1 y k números compuestos todos excepto P1 .

En el peor de los casos después de k aplicaciones de este procedimiento se obtiene


un número que es producto de los números iniciales y cuadrado perfecto.

segundo dı́a

4. Consideremos la figura:
232

A
F

O
E

M
B C D

←→
m∠ABD = 90◦ pues BC es tangente al cı́rculo. ∠AEB es recto, pues es un ángulo
inscrito cuyo arco mide 180◦ . Por otra parte m∠BAE = m∠BEC , pues ambos son
ángulos semi-inscritos que comparten el mismo arco (1).
Por otro lado m∠BAD + m∠BDA = 90◦ , pues son ángulos complementarios, y

m∠C ED + m∠BEC = 90◦ , (2)

pues son ángulos complementarios.


Por (2) y por (1), tenemos que

m∠C ED = m∠C DE ⇒ C E = C D. (3)

Además, m∠BEC = m∠C BE, pues son ángulos semi-inscritos que comparten el
mismo arco. Y de aquı́ podemos concluir que BC = C E. (4)
Por (3) y (4) se tiene que C E = C D y que BC = C E ⇒ BC = C D. De esta forma
concluimos la prueba.

5. Despejando f (x + w) se sigue que:

f (x)[f (x + w) − 1] = 2f (x + w) − 3
f (x)f (x + w) − f (x) = 2f (x + w) − 3f (x)f (x + w) − 2f (x + w) = f (x) − 3
f (x + w)(f (x) − 2) = f (x) − 3
f (x) − 3
f (x + w) = (1)
f (x) − 2

Ahora, haciendo las dos evaluaciones siguientes se prueba que f (x + 3w) = f (x).
f (x + w) − 3
f (x + 2w) = f (x + w + w) = (2)
f (x + w) − 2
Sustituyendo (1) en (2) se sigue que
2f (x) − 3
f (x + 2w) = (3)
f (x) − 1
233

f (x + 2w) − 3
f (x + 3w) = f (x + 2w + w) = (4)
f (x + 2w) − 2
Ahora, sustituyendo (3) en (4) se concluye que f (x + 3w) = f (x) con lo cual 3w es
perı́odo de la función f .

6. Dado que a2 = (a2 − a + 1) + (a − 1), se obtiene

(a2 )n+2 = a2n+4 = (a − 1)n+2 + (a2 − a + 1)T1 (a).

Ası́:

a2n+1 + (a − 1)n+2 = a2n+1 + a2n+4 − (a2 − a + 1)T1 (a)


= a2n+1 (a3 + 1) − (a2 − a + 1)T1 (a)
= a2n+1 (a + 1)(a2 − a + 1) − (a2 − a + 1)T1 (a)
= (a2 − a + 1)(a2n+1 (a + 1) − T1 (a))
234
Conceptos y resultados útiles

Presentamos a continuación algunos conceptos y resultados que pueden ser útiles en


la resolución de problemas. En general son bastante básicos y de fácil comprensión pero
algunos de ellos usualmente no se estudian en la enseñanza media. Además del temario
correspondiente sugerimos el estudio de estos resultados.

ALGEBRA
1. (Principio del palomar o de Dirichlet) Si n+1 objetos se distribuyen al azar en n cajas,
entonces alguna caja contendrá por lo menos dos objetos. Una forma más general es:
si nk + 1 objetos se distribuyen al azar en n cajas, entonces alguna caja contendrá al
menos k + 1 objetos.
Ejemplo: Dado cualquier conjunto A de 10 números naturales comprendidos entre 1 y
99 (ambos inclusive), pruebe que existen dos subconjuntos de A, disjuntos y no vacı́os
B y C , tales que la suma de los elementos de B es igual a la suma de los elementos
de C .
Solución: Con los 10 elementos de A se pueden forma 210 − 1 = 1023 subconjuntos
no vacı́os distintos. La suma de los elementos de cada uno de estos subconjuntos es
menor que 1000, porque incluso 90 + 91 + · · · + 99 < 1000.
Ası́, por el principio del palomar, al menos dos subconjuntos P y Q deben tener la
misma suma. Quitando los elementos comunes a ambos se obtendrı́an dos conjuntos
que son disjuntos y que tienen la misma suma. Por otra parte, ninguno de ellos es
vacı́o, porque si uno fuera vacı́o significarı́a que P ⊂ Q o Q ⊂ P lo cual no puede ser
puesto que P y Q son diferentes y tienen la misma suma.

2. Para r 6= 1 y n ∈ N, se tiene

r n+1 − 1
1 + r + r2 + · · · + rn = .
r−1

3. Para n ∈ N, se tiene
n(n + 1)
1 + 2 + 3 + ··· + n = .
2

235
236

4. Para n ∈ N, se tiene

n(n + 1) (2n + 1)
12 + 22 + 32 + · · · + n2 = .
6

5. (Fórmulas de Vieta) Dado el polinomio f (x) = an x n + an−a x n−1 + · · · + a1 x + a0


(con an 6= 0), si las raı́ces de este polinomio son x1 , x2 , ..., xn , entonces se tienen las
siguientes relaciones:

X
n
an−1
xi = − ,
an
i=1
X an−2
xi xj = + ,
i<j
an
X an−3
xi xj xk = − ,
an
i<j<k

...,
a0
x1 x2 · · · xn = (−1)n
an

6. (Media aritmética-media geométrica) Si x1 , x2 , · · · , xn son n números reales positivos,


entonces su media aritmética y media geométrica satisfacen la desigualdad:
x1 + x2 + · · · + xn
≥ (x1 x2 · · · xn )1/n
n
Las igualdades se dan si y solo si x1 = x2 = · · · = xn .
a+b √
Para el caso de dos números a y b, la desigualdad es ≥ ab.
2
Ejemplo: Vea la solución del ejercicio 15 en la página 176.

7. Si n ≥ 1, entonces (1 + x)n ≥ 1 + nx, para x > −1.

8. Si 0 < n < 1, entonces (1 + x)n ≤ 1 + nx, para x > −1.

9. Una secuencia de números

a, a + k, a + 2k, . . . , a + nk, . . .

se llama progresión aritmética de diferencia k.


Ejemplo:
3, 7, 11, 15
es una progresión aritmética de cuatro términos, de diferencia 4.
237

10. Una secuencia de números

a, a · k, a · k 2 , . . . , a · k n , . . .

se llama progresión geométrica de razón k.


Ejemplo:
2, 6, 18, 54, 162
es una progresión geométrica de cinco términos, de razón 3.
11. (Principio de inducción matemática) Sea P(n) una propiedad definida para n ∈ N. Si
la propiedad es verdadera para n = m y si siempre que P(n) es verdadera, también
P(n + 1) lo es, entonces la propiedad se cumple para todo número natural n ≥ m.
Este principio constituye un método de demostración muy útil para propiedades que
involucran los números naturales, en particular igualdades y desigualdades con esos
números.
Ejemplo: Probar que para todo número natural n ≥ 2, se tiene
1 1 1 √
√ + √ + · · · + √ > n.
1 2 n
Solución: Llamemos con P(n) a la propiedad indicada.
i) Como se debe cumplir para todo n ≥ 2, debemos comenzar
√ probando que P(2) es
verdadera. Debemos probar, entonces, que √11 + √12 > 2. Tenemos que

1 1 √ 1 √ √ √ √
√ + √ > 2⇔1+ √ > 2⇔2+ 2>2 2⇔2> 2
1 2 2

Como lo último es verdadero, entonces también es verdadero que √1 + √1 > 2.
1 2
ii) Supongamos que P(n) es verdadera y demostremos que P(n + 1) es verdadera. Es
decir, suponemos que
1 1 1 √
√ + √ + · · · + √ > n.
1 2 n
es verdadero (hipótesis de inducción) y debemos, entonces, probar que se satisface
1 1 1 1 √
√ + √ + ··· + √ + √ > n + 1.
1 2 n n+1
En efecto,
1 1 1 1
√ + √ + ··· + √ + √
1 2 n n+1
 
1 1 1 1
= √ + √ + ··· + √ +√ (1)
1 2 n n+1
√ 1
> n+ √ (por hip. de inducción)
n+1
238

Ahora,
√ 1 √
n+ √ > n+1⇔
n+1
√ √ √ √
n n+1+1> n+1 n+1⇔
p p
n2 + n + 1 > n + 1 ⇔ n2 + n > n.

Si elevamos al cuadrado ambos miembros en la última desigualdad, esta se preserva


pues los dos son positivos, entonces
p
n2 + n > n ⇔ n2 + n > n2 ⇔ n > 0 (restando n2 ).

La última desigualdad (n > 0) es verdadera pues n ≥ 2 y, como en todos los casos


tenemos si y solo si (⇔), entonces, se tiene que

√ 1 √
n+ √ > n + 1.
n+1

Por esta razón y por (1),

1 1 1 1 √
√ + √ + ··· + √ + √ > n + 1,
1 2 n n+1

tal como querı́amos.

TEORÍA DE NÚMEROS
1. (Representación en base b) Si b es un número natural mayor que 1 entonces, para
todo n ∈ N, existen
a0 , a1 , ..., am ∈ {0, 1, ..., b − 1}
tales que
n = am · bm + am−1 · bm−1 + · · · + a1 · b + a0 .
Si b = 10, la representación anterior es la correspondiente a la representación decimal
y los números a0 , a1 , ..., am se llaman dı́gitos o cifras.

2. (Algoritmo de la división) Dados n y b números enteros, existen únicos q y r, con


0 ≤ r < |b| tales que n = qb + r.

3. (Concepto de divisor) Dados a y b números enteros, a divide a b y se denota por a|b


si existe un número entero k tal que b = a · k. Si a divide a b se dice que a es un
divisor de b o que a es es un factor de b o que b es divisible por a o que b es un
múltiplo de a.
239

4. (Propiedades del divisor) Sean a, b y c números enteros entonces:


a) a|a
b) Si a|b y b|c entonces a|c
c) Si a|b y a|c entonces a|(nb + kc), para n y k enteros
d) Si a|b entonces |a| ≤ |b|
e) Si a|b y b|a entonces |a| = |b|
f) Siempre 1|a y a|0
g) Si a|c entonces ab|cb
h) Si ac|bc y c 6= 0 entonces a|b
5. Si a, b, c son enteros tal que a = b + c y si b y c son múltiplos de k entonces a es
múltiplo de k.
6. Dados n números enteros consecutivos
k + 1, k + 2, ..., k + n, uno de ellos, necesariamente, divide a n.
7. (Criterios de divisibilidad)

a) Un número es divisible por 2 si su última cifra es par.


b) Un número es divisible por 3 si la suma de sus cifras es divisible por 3.
c) Un número es divisible por 4 si el número formado por sus dos últimas cifras es
divisible por 4.
d) Un número es divisible por 5 si su última cifra es 0 o 5.
e) Un número es divisible por 7 si, al restar al número que queda suprimiendo la
última cifra el doble de esta última cifra, se obtiene un múltiplo de 7.
f ) Un número es divisible por 11 si la suma de las cifras de las posiciones pares,
menos la suma de las cifras de las posiciones impares es múltiplo de 11.

8. (Máximo divisor común) El máximo divisor común de dos números enteros a y b es el


entero c tal que c > 0, c|a, c|b y si d es otro entero que divide a a y b, entonces
d|c (en otras palabras, c es el mayor de los divisores comunes de a y b). El máximo
divisor común de a y b se denota por mcd(a, b). Si el máximo divisor común de a y
b es 1 se dice que a y b son primos relativos o coprimos.
9. (Mı́nimo múltiplo común) El mı́nimo múltiplo común de dos números enteros a y b es
el entero c tal que c > 0, a|c, b|c y si d es otro entero que es dividido por a y b,
entonces c|d (en otras palabras, c es el menor de los múltiplos comunes positivos de
a y b). El mı́nimo múltiplo común de a y b se denota por mcm(a, b).
10. (Algunas propiedades de mcd y mcm)

a) Existen números enteros tales que


mcd(a, b) = as + bt. En particular, si a y b son primos relativos, existen s y t
enteros tales que 1 = as + bt.
240

b) (Lema de Euclides) Si a|bc y a y c son primos relativos, entonces a|b.


c) Si a = qb + r entonces mcd(a, b) = mcd(b, r)
d) mcm(a, b) · mcd(a, b) = |a| · |b|

11. (Números primos y compuestos) Un número natural p > 1 se llama primo si sus únicos
divisores son 1 y p. Si el número tiene más de dos divisores se llama compuesto.

12. Si p es primo y p|ab entonces p|a ó p|b.

13. (Teorema fundamental de la Aritmética) Sea n ∈ N, sean p1 , p2 , ..., pk todos los


divisores primos de n, entonces existen únicos a1 , a2 , ..., ak ∈ N, tales que n =
pa1 1 · pa2 2 · ... · pak k . Esta se llama la descomposición prima de n.

14. (Número de divisores) Si el número n tiene la descomposición prima del punto anterior,
entonces el número de divisores de n es igual a (a1 + 1)(a2 + 1) · ... · (ak + 1).

15. (Parte entera) Si x es un número real, su parte entera se define como el mayor entero
que
h√ ies menor o igual que x y se denota por [x]. Por ejemplo [5] = 5, [−1, 23] = −2,
 11 
2 = 1, 3 = 3.
Ejemplo: Vea el ejercicio número 1 en la página 38.

GEOMETRÍA
1. En un triángulo:

a) Las tres alturas son concurrentes, su punto de intersección se llama ortocentro.


b) Las tres medianas son concurrentes, su punto de intersección se llama baricentro.
c) Las tres mediatrices son concurrentes, su punto de intersección se llama circun-
centro.
d) Las tres bisectrices son concurrentes, su punto de intersección se llama incentro.

2. (Fórmula de Herón) Si las medidas de los lados de un triángulo son a, b, c. Entonces


el área del triángulo es
p
A = s (s − a) (s − b) (s − c),

donde s = a+b+c
2 (el semiperı́metro del triángulo).

3. (Otra forma para calcular el área de un triángulo) Si dos lados de un triángulo miden
a y b, y el ángulo comprendido entre esos lados es igual a α, entonces el área del
triágulo es igual a
1
A = · a · b · sin α.
2
Ejemplo: Vea la solución del ejercicio 9 en la página 130.
241

4. (Ley de los cosenos) Si a, b, c son las medidas de los lados de un triángulo y α es


el ángulo entre los lados de medidas b y c, se cumple que

a2 = b2 + c2 − 2bc · cos α.

5. (Ley de los senos) Si las medidas de los lados de un triángulo son a, b y c y la


medida del ángulo opuesto al lado de medida a es α, la del ángulo opuesto al lado
de medida b es β y la del ángulo opuesto al lado de medida c es γ, entonces

a b c
= = .
sin α sin β sin γ

Ejemplo: Vea la solución del ejercicio número 30 en la página 184.

6. En dos triángulos semejantes se tiene que la razón de:

a) sus perı́metros es igual a la razón de semejanza.


b) dos de sus alturas correspondientes es igual a la razón de semejanza.
c) sus áreas es igual al cuadrado de la razón de semejanza.
d) las bisectrices de dos ángulos correspondientes es igual a la razón de semejanza.

7. La bisectriz interior de un ángulo de un triángulo, divide al lado opuesto en segmentos


proporcionales a los lados adjuntos. Con la notación de la figura:

DC BD
= .
AC AB

A B

8. El punto de intersección de las tres medianas de un triángulo divide a cada mediana


en la razón 2 : 1. Con la notación de la figura

BO 2
=
OM 1
o, de modo equivalente,
2
BO = BM.
3
242

A C
M

Ejemplo: Vea la solución del ejercicio 2 en la página 225.

9. Las diagonales de un paralelogramo se cortan en sus puntos medios.

10. La suma de los cuadrados de las longitudes de las diagonales de un paralelogramo


es igual a la suma de los cuadrados de las longitudes de sus lados.

11. (Potencia de un punto respecto a un cı́rculo)

Si AB y C D son cuerdas de un cı́rculo que se cortan en un punto P, entonces


AP · PB = DP · PC .
D

B
P

Si P es un punto exterior a un cı́rculo, PB es secante al cı́rculo y lo corta en


los puntos A y B, PD es secante al cı́rculo y lo corta en los puntos C y D y PT
es tangente al cı́rculo en el punto T , entonces

PA · PB = PC · PD = PT 2 .

A D

C
P
T
243

12. Un cuadrilátero es concı́clico si sus cuatro vértices están en un mismo cı́rculo. Esto
sucede si y solo si sus ángulos opuestos son suplementarios (suman 180◦ ).

A C

Ejemplo: Vea la solución del ejercicio número 17 en la página 159.

13. (Teorema de Tolomeo) Si ABC D es un cuadrilátero concı́clico convexo, con diagonales


AC y BD, entonces AC · BD = AB · C D + AD · BC .

A C

B
244
Simbologı́a

4ABC triángulo de vértices A, B, C


(ABC ...) área del polı́gono de vértices A, B, C , ...
4ABC ∼ = 4PQR el triángulo ABC es congruente al triángulo PQR
4ABC ∼ 4PQR el triángulo ABC es semejante al triángulo PQR
AB segmento de extremos A y B
AB medida del segmento de extremos A y B
]ABC ángulo A, B, C
m]ABC medida del ángulo A, B, C
←→
AB recta que contiene los puntos A y B
lkm l es paralelo a m
l⊥m l es perpendicular a m
N conjunto de los números naturales: {1, 2, 3, ...}
Df dominio de la función f
[x] parte entera del número real x.
x>y x es mayor que y
x≥y x es mayor o igual que y
x<y x es menor que y
x≤y x es menor o igual que y
∀ para todo
∃ existe al menos
⇒ implica

245
246
Temario de la Olimpiada

NIVEL A

GEOMETRÍA

Nociones elementales: punto recta plano, segmento, semirrecta, rayo, semiplano. Puntos
colineales. Rectas paralelas y perpendiculares. Ángulos. Clasificación de ángulos. Ángulos
congruentes, consecutivos, adyacentes, opuestos por el vértice, complementarios, suplemen-
tarios, ángulos determinados por una secante a dos rectas paralelas. Polı́gonos, ángulos
externos e internos. Paralelogramos y sus propiedades. Áreas de paralelogramos. Triángulos.
Desigualdad triangular. Área del triángulo. Suma de las medidas de los ángulos internos
de un triángulo. Clasificación de los triángulos según las medidas de sus lados y de sus
ángulos. Propiedades. Rectas notables en un triángulo. Propiedades relativas a las rectas
notables en un triángulo. Teorema de Pitágoras. Fórmula de Herón.

ÁLGEBRA

Nociones elementales de teorı́a de conjuntos. Conjuntos numéricos N, Z, Q, I, R. Ope-


raciones, propiedades y aplicaciones. Potencias y radicales. Razones y proporciones y sus
propiedades. Valor absoluto. Notación cientı́fica. Expresiones algebraicas. Valor numérico
de una expresión algebraica. Polinomios. Fórmulas notales (primera, segunda y tercera).
Ecuaciones de primer grado y problemas de aplicación.

TEORÍA DE NÚMEROS

Concepto de divisibilidad: divisor, múltiplo. Propiedades. Algoritmo de la división. Núme-


ros primos y compuestos. Teorema fundamental de la aritmética. Máximo común divisor.
Mı́nimo común múltiplo. Propiedades. Reglas de divisibilidad. Obtención de los divisores
positivos de un número. Número de divisores positivos de un número. Notación desarrollada
de un número entero positivo.

247
248

NIVEL B

GEOMETRÍA

Nociones elementales de Geometrı́a: punto, recta, plano, segmento, semirrecta, rayo,


semiplano. Puntos colineales. Rectas paralelas y perpendiculares. Angulos. Clasificación de
ángulos por su medida. Angulos congruentes, consecutivos, adyacentes, opuestos por el
vértice, complementarios, suplementarios, ángulos determinados por una secante a dos rectas
paralelas. Polı́gonos. Ángulos internos y externos. Paralelogramos. Propiedades. Triángulos.
Desigualdad triangular. Clasificación de los triángulos según las medidas de sus lados y
de sus ángulos. Propiedades. Rectas notables en un triángulo. Propiedades relativas a las
rectas notables en un triángulo. Semejanza de triángulos y proporcionalidad entre lados
correspondientes, alturas, perı́metros y áreas. Teorema de Thales. Teorema de Pitágoras.
Perı́metros y áreas de figuras planas. Fórmula de Herón. Congruencia de triángulos. Ley de
senos. Ley de cosenos.

ÁLGEBRA

Nociones elementales de la teorı́a de conjuntos. Conjuntos numéricos, N, Z, Q, I, R.


Operaciones, propiedades y aplicaciones. Potenciación y radicación. Razones y proporciones.
Propiedades. Valor absoluto. Notación cientı́fica. Expresiones algebraicas. Valor numérico de
una expresión algebraica. Polinomios. Fórmulas notables (primera, segunda, tercera, cuarta y
quinta). Ecuaciones de primer grado. Problemas de aplicación. Ecuaciones de segundo grado.
Sistemas de ecuaciones. Problemas de aplicación. Factorización de polinomios.Teorema del
factor. Simplificación de expresiones algebraicas. Desigualdades y sus propiedades.

TEORÍA DE NÚMEROS

Concepto de divisibilidad: divisor, múltiplo. Propiedades. El algoritmo de la división.


Números primos y números compuestos. El teorema Fundamental de la Aritmética. Máxi-
mo común divisor. Mı́nimo común múltiplo. Propiedades de estas nociones. Reglas de di-
visibilidad. Obtener los divisores positivos de un número. Número de divisores. Notación
desarrollada de un número entero positivo.

TRIGONOMETRÍA

Razones trigonométricas de un ángulo agudo de un triángulo rectángulo. Razones tri-


gonométricas de los ángulos especiales 30◦ , 60◦ , 45◦ . Problemas de aplicación (ángulos
de elevación y de depresión, entre otros). Funciones trigonométricas. Algunas propiedades
básicas.
249

NIVEL C
GEOMETRÍA
Nociones elementales de Geometrı́a: punto, recta, plano, segmento, semirrecta, rayo,
semiplano. Puntos colineales. Rectas paralelas y perpendiculares. Angulos. Clasificación de
ángulos por su medida. Angulos congruentes, consecutivos, adyacentes, opuestos por el vérti-
ce, complementarios, suplementarios, ángulos determinados por una secante a dos rectas
paralelas. Polı́gonos. Ángulos internos y externos. Paralelogramos. Propiedades. Triángulos.
Desigualdad triangular. Clasificación de los triángulos según las medidas de sus lados y
de sus ángulos. Propiedades. Rectas notables en un triángulo. Propiedades relativas a las
rectas notables en un triángulo. Semejanza de triángulos y proporcionalidad entre lados
correspondientes, alturas, perı́metros y áreas. Teorema de Thales. Teorema de Pitágoras.
Perı́metros y áreas de figuras planas. Fórmula de Herón. Congruencia de triángulos. Ley
de cosenos. Ley de senos. Cı́rculo y circunferencia. Elementos especiales en el cı́rculo: ra-
dio, secante, tangente, cuerda, diámetro. Circunferencias concéntricas, tangentes y secantes.
Concepto de arco. Ángulo centra, inscrito, semiinscrito y sus relaciones con los arcos que
subtienden. Polı́gonos inscritos y circunscritos. Nociones de apotema, ángulo interno y ex-
terno en un polı́gono. Areas y perı́metros de polı́gonos. Cuerpos geométricos básicos: prisma,
cubo, cilindro, cono, pirámide, esfera y sus elementos. Área lateral y volumen de los cuerpos
geométricos.

ÁLGEBRA
Nociones elementales de la teorı́a de conjuntos. Conjuntos numéricos, N, Z, Q, I, R.
Operaciones, propiedades y aplicaciones. Potenciación y radicación. Razones y proporciones.
Propiedades. Valor absoluto. Notación cientı́fica. Expresiones algebraicas. Valor numérico de
una expresión algebraica. Polinomios. Fórmulas notables (primera, segunda, tercera, cuarta
y quinta). Ecuaciones de primer grado. Problemas de aplicación. Ecuaciones de segundo
grado. Sistemas de ecuaciones. Problemas de aplicación. Teorema del factor. Factorización
de polinomios. Simplificación de expresiones algebraicas. Desigualdades y sus propiedades.

TEORÍA DE NÚMEROS
Concepto de divisibilidad: divisor, múltiplo. Propiedades. El algoritmo de la división.
Números primos y números compuestos. El teorema Fundamental de la Aritmética. Máxi-
mo común divisor. Mı́nimo común múltiplo. Propiedades de estas nociones. Reglas de di-
visibilidad. Obtener los divisores positivos de un número. Número de divisores. Notación
desarrollada de un número entero positivo.

TRIGONOMETRÍA
Razones trigonométricas de un ángulo agudo de un triángulo rectángulo. Razones tri-
gonométricas de los ángulos especiales 30◦ , 60◦ , 45◦ . Problemas de aplicación (ángulos de
250

elevación y de depresión, entre otros). Funciones trigonométricas. Identidades trigonométri-


cas. Ecuaciones trigonométricas.

FUNCIONES
Concepto de función. Nociones generales: dominio, codominio, rango (ámbito), imagen,
pre–imagen, gráfico de una función. Tipos especiales de funciones: lineal, cuadrática, sobre-
yectiva, inyectiva, biyectiva, constante. Operaciones con funciones. Composición de funciones.
Inversa de una función. Función exponencial y logarı́tmica. Propiedades de los logaritmos.
Cambio de base. Identidades y ecuaciones logarı́tmicas y exponenciales. Problemas de apli-
cación.

También podría gustarte